C03 Integration

You might also like

Download as pdf or txt
Download as pdf or txt
You are on page 1of 129

Get all the files at: https://bit.

ly/azizhandouts

Integration
REVISION: 3162
31 January 2024

AZIZ MANVA
AZIZMANVA@GMAIL.COM

ALL RIGHTS RESERVED


Get all the files at: https://bit.ly/azizhandouts

Table of contents
2. MORE INTEGRATION ..........................78
1. INTEGRATION ........................................ 3 2.1 Integration by Parts 78
1.1 Indefinite Integration 3 2.2 Trigonometric Integrals 93
1.2 𝒖-substitution 14 2.3 Trigonometric Substitutions 104
1.3 Area under a Curve and Reimann Sums 24 2.4 Partial Fractions 111
1.4 Definite Integral: Reimann Sum Definition 36 2.5 Arc Length 115
1.5 Definite Integrals (FTC) and Areas 41 2.6 Improper Integrals: Type I 118
1.6 Derivatives of Integrals (FTC) 52 2.7 Improper Integrals: Type II 125
1.7 𝒖 −Substitution with Definite Integrals 56
3. FURTHER TOPICS............................. 127
1.8 Average Value, Area between Curves 58
1.9 Volumes with Cross Sections 65 3.1 Hyperbolic Integrals 127
1.10 Volumes with Cylindrical Shells 73 3.2 Leibniz Rule 128
1.11 Kinematics, Centroids, & Biology 74 3.3 Walli’s Theorem 129
Get all the files at: https://bit.ly/azizhandouts

1. INTEGRATION
1.1 Indefinite Integration
A. Antiderivatives
You differentiate a function to find its derivative. If you reverse the process, you are finding an anti-derivative.

1.1: Antiderivative
𝐹(𝑥) is an antiderivative of 𝑓(𝑥) if:
𝐹 ′ (𝑥) = 𝑓(𝑥)

Example 1.2
Find an antiderivative for:
𝑓(𝑥) = 5

𝑓(𝑥) = 5 ⇒ 𝐹(𝑥) = 5𝑥
B. Integration

Example 1.3
If 𝐺(𝑥) is an antiderivative of 𝑓(𝑥), and 𝐹(𝑥) is also an antiderivative of 𝑓(𝑥) then what is the connection
between the two?

∫ 𝑓(𝑥) 𝑑𝑥 = 𝐹(𝑥) ⇒ 𝐹 ′ (𝑥) = 𝑓(𝑥)

∫ 𝑓(𝑥) 𝑑𝑥 = 𝐺(𝑥) ⇒ 𝐺 ′ (𝑥) = 𝑓(𝑥)

𝐹 ′ (𝑥) = 𝐺 ′ (𝑥) = 𝑓(𝑥)

Since the two functions have the same derivative, they can only differ by a constant.
𝐹(𝑥) = 𝐺(𝑥) + 𝐶, 𝑓𝑜𝑟 𝑠𝑜𝑚𝑒 𝑐𝑜𝑛𝑠𝑡𝑎𝑛𝑡 𝐶

1.4: Integration
The collection of all antiderivatives of a function 𝑓(𝑥) is called the indefinite integral of 𝑓(𝑥) and is written
∫ 𝑓(𝑥) 𝑑𝑥 = 𝐹(𝑥) + 𝐶
Where C is a 𝑐𝑜𝑛𝑠𝑡𝑎𝑛𝑡 𝑜𝑓 𝑖𝑛𝑡𝑒𝑔𝑟𝑎𝑡𝑖𝑜𝑛

➢ The symbol 𝑑𝑥 is used to indicate that we are finding the antiderivative with respect to 𝑥.

Example 1.5
2
A. Evaluate ∫ 7 𝑑𝑥
B. In the integral that you evaluated above, what are the values that 𝐶 can take?

Part A
2 2
∫ 𝑑𝑥 = 𝑥 + 𝐶
7 7
Part B
Get all the files at: https://bit.ly/azizhandouts

𝐶 ∈ (−∞, ∞) 𝑂𝑅 𝐶 ∈ ℝ

1.6: Constant Multiple Rule for Integration


For any constant 𝑘:

∫ 𝑘 ∙ 𝑓(𝑥) 𝑑𝑥 = 𝑘 ∫ 𝑓(𝑥) 𝑑𝑥

➢ We can generalize the two examples we have been doing so far in the property above. It lets us move
constants “out” of the integration symbol.
➢ This property is directly related to the constant multiple property for derivatives.

Example 1.7
Find the most general indefinite integral for the exercises below. Do not forget the constant of integration.
∫ −𝜋 𝑑𝑥

Constants
Simply putting an 𝑥 before the given number, and adding a constant of integration works:
∫ −𝜋 𝑑𝑥 = −𝜋𝑥 + 𝐶

C. Power Rule (Algebraic Integrals)

1.8: Power Rule for Integration


𝑥 𝑛+1
∫ 𝑥 𝑛 𝑑𝑥 = + 𝐶, 𝑛 ≠ −1
𝑛+1

We can check this by differentiation:



𝑥 𝑛+1 𝑛 + 1 𝑛+1−1
( ) = 𝑥 = 𝑥𝑛
𝑛+1 𝑛+1

The 𝑛 = −1 case does not work since:


1
∫ 𝑑𝑥 = ln |𝑥|
𝑥

Example 1.9: Integer Exponents


Find the most general indefinite integral for the exercises below. Do not forget the constant of integration.
∫ 𝑥 3 𝑑𝑥

∫ 5𝑥 2 𝑑𝑥
4
∫ − 𝑥 7 𝑑𝑥
3

Here, the formula given for the power rule is useful:


𝑥 3+1 𝑥 4
∫ 𝑥 3 𝑑𝑥 = = +𝐶
3+1 4
𝑥 2+1 5
∫ 5𝑥 2 𝑑𝑥 = 5 ∙ = ( ) 𝑥3 + 𝐶
2+1 3
Get all the files at: https://bit.ly/azizhandouts

4 𝑥 7+1 4 𝑥8 𝑥8
(− ) = (− ) = − + 𝐶
3 7+1 3 8 6

Example 1.10:
Evaluate, if possible, using the power rule. Explain the answer that you get
∫ 𝑥 −1 𝑑𝑥

𝑥0
∫ 𝑥 −1 𝑑𝑥 = ⇒ 𝑁𝑜𝑡 𝑑𝑒𝑓𝑖𝑛𝑒𝑑
0

In the power rule,


𝑛 ≠ −1 𝑓𝑜𝑟 𝑝𝑟𝑒𝑐𝑖𝑠𝑒𝑙𝑦 𝑡ℎ𝑖𝑠 𝑟𝑒𝑎𝑠𝑜𝑛

In fact:
1
∫ 𝑥 −1 𝑑𝑥 = ∫ 𝑑𝑥 = ln|𝑥| + 𝐶
𝑥

1.11: Sum and Difference Property


∫ 𝑓(𝑥) + 𝑔(𝑥) 𝑑𝑥 = ∫ 𝑓(𝑥) 𝑑𝑥 + ∫ 𝑔(𝑥) 𝑑𝑥

∫ 𝑓(𝑥) − 𝑔(𝑥) 𝑑𝑥 = ∫ 𝑓(𝑥) 𝑑𝑥 − ∫ 𝑔(𝑥) 𝑑𝑥

Example 1.12: Fractional Exponents


1 2 1
∫ −𝑥 3 + 𝑥 2 𝑑𝑥
3

The same power rule formula applies, but since we have fractions, we need to be more careful with the
calculations:
1 4
1 𝑥 3+1 𝑥3 3 4
∫ −𝑥 3 𝑑𝑥 =− =− = − 𝑥3
1 4 4
3+1 3
1 3
2 1 2 𝑥 2+1 2 𝑥2 4 3
∫ 𝑥 2 𝑑𝑥 = ( ) =( ) = 𝑥2
3 3 1+1 3 3 9
2 2
Hence, the final answer is (remember to add the constant of integration):
1 2 1 4 3 3 4
∫ −𝑥 3 + 𝑥 2 𝑑𝑥 = 𝑥 2 − 𝑥 3 + 𝐶
3 9 4

Example 1.13: Radicals


73
∫ (√𝑥 − √𝑥 ) 𝑑𝑥
9

Split the integral using the sum and difference rule:


73
∫ √𝑥 𝑑𝑥 + ∫ − √𝑥 𝑑𝑥
9
In the first term, rewrite the radical as an exponent, and then apply the power rule.
Get all the files at: https://bit.ly/azizhandouts

1 3
1 𝑥 2+1 𝑥2 2 3
∫ √𝑥 𝑑𝑥 = ∫ 𝑥 2 𝑑𝑥 = = = 𝑥 2 + 𝐶1
1 3 3
2+1 2
1
3
In the second term, rewrite the radical √𝑥 = 𝑥 3 :
1 4
73 7 𝑥 3+1 7 𝑥3 7 3 4 7 4
∫ − √𝑥 𝑑𝑥 = (− ) ( ) = (− ) ( ) = (− ) ( ) 𝑥 3 = (− ) (𝑥 3 ) + 𝐶2
9 9 1+1 9 4 9 4 12
3 3
Combine the two to get the final answer:
2 3 7 4
𝑥 2 − ( ) (𝑥 3 ) + 𝐶
3 12

Example 1.14: Negative Exponents


1 3
∫( 2
+ 3 ) 𝑑𝑥
𝑥 𝑥

1 3
∫ 𝑑𝑥 + ∫ 𝑑𝑥
𝑥2 𝑥3
1
If a power of 𝑥 is in the denominator, move the variable to the numerator by using the property 𝑎𝑚 = 𝑎−𝑚 .
1 −2
𝑥 −2+1 𝑥 −1 1
∫ 2
𝑑𝑥 = ∫ 𝑥 𝑑𝑥 = = = − + 𝐶1
𝑥 −2 + 1 −1 𝑥
3 −3
3 −2
∫ 3 𝑑𝑥 = ∫ 3𝑥 𝑑𝑥 = ( ) 𝑥 + 𝐶2
𝑥 −2
Combine the two to get the final answer:
1 3
= − − ( 2) + 𝐶
𝑥 2𝑥

Example 1.15: Negative Fractional Exponents


1 3
∫ 1 + 2 𝑑𝑥
𝑥2 4𝑥 3
1 1
1 𝑥 −2+1

1 𝑥 2+1
∫ 1 𝑑𝑥 = ∫ 𝑥 = 2 𝑑𝑥
= = 2√𝑥
1 1
𝑥 2 − 2 + 1 − 2
3 13
3 3 −2 (4) 𝑥 3
9 √𝑥
∫ 2 𝑑𝑥 = ∫ ( ) 𝑥 3 𝑑𝑥 = +𝐶 = +𝐶
4 1 4
4𝑥 3 3
3
1 3 9 √𝑥
∫ 1+ 2 𝑑𝑥 = 2√𝑥 + +𝐶
4
𝑥2 4𝑥 3

Example 1.16: Radicals in the denominator


1
∫ 3 𝑑𝑥
√𝑥

Rewrite the radical as an exponent, and then integrate:


Get all the files at: https://bit.ly/azizhandouts

1 2
1 1 −
1 𝑥 −3+1 𝑥3 3 2
∫ 3 𝑑𝑥 = ∫ 1 𝑑𝑥 = ∫ 𝑥 3 𝑑𝑥 = +𝐶 = + 𝐶 = 𝑥3 + 𝐶
√𝑥 1 2 2
𝑥3 −3 + 1 3

D. Logarithmic and Exponential Integrals

𝟏
1.17: Integral of
𝒙
1
∫ 𝑑𝑥 = ln|𝑥| + 𝐶
𝑥

𝑑𝑦 1
𝑦 = ln 𝑥 ⇒ =
𝑑𝑥 𝑥
➢ Pay attention to the absolute value sign in the integral, which is important since ln 𝑥 is not defined for
𝑥 < 0.

Example 1.18
1
∫ − 𝑑𝑥
𝑥

1
∫ − 𝑑𝑥 = − ln|𝑥| + 𝐶
𝑥
𝑛
Use the power rule in logarithms ln 𝑥 = 𝑛 ln 𝑥:
1
ln|𝑥 −1 | + 𝐶 = ln | | + 𝐶
𝑥

Example 1.19
2
∫ 𝑑𝑥
𝑥
𝑎
∫ 𝑑𝑥
𝑥
1
∫ 𝑑𝑥
2𝑥

2 1
∫ 𝑑𝑥 = 2 ∫ 𝑑𝑥 = 2 ln|𝑥| = ln|𝑥 2 | = ln 𝑥 2 + 𝐶
𝑥 𝑥
𝑎 1
∫ 𝑑𝑥 = 𝑎 ∫ 𝑑𝑥 = 𝑎 ln|𝑥| = ln|𝑥 𝑎 | + 𝐶
𝑥 𝑥
1 1 1 1 1
∫ 𝑑𝑥 = ∫ 𝑑𝑥 = ln|𝑥|𝑥 = ln |𝑥 2 | + 𝐶
2𝑥 2 𝑥 2

Example 1.20
1 5 𝑎
∫ + + 𝑑𝑥
𝑎𝑥 3𝑥 𝑏𝑥

Split the Integral


1 1 5 1 𝑎 1
∫ 𝑑𝑥 + ∫ 𝑑𝑥 + ∫ 𝑑𝑥
𝑎 𝑥 3 𝑥 𝑏 𝑥
Integrate:
Get all the files at: https://bit.ly/azizhandouts

1 5 𝑎
ln 𝑥 + ln 𝑥 + ln 𝑥 + 𝐶
𝑎 3 𝑏
Use the power rule for logarithms:
1 5 𝑎
ln 𝑥 𝑎 + ln 𝑥 3 + ln 𝑥 𝑏 + 𝐶

𝒇′ (𝒙)
1.21: Integral of
𝒇(𝒙)
𝑓′(𝑥)
∫ 𝑑𝑥 = ln|𝑓(𝑥)| + 𝐶
𝑓(𝑥)

𝑑𝑦 1 𝑓′(𝑥)
𝑦 = ln[𝑓(𝑥)] ⇒ = × 𝑓 ′ (𝑥) =
𝑑𝑥 𝑓(𝑥) 𝑓(𝑥)

2𝑥
∫ 𝑑𝑥 = ln 𝑥 2 + 𝐶
𝑥2

Example 1.22
3𝑥 + 1
∫ 𝑑𝑥
6𝑥 2 − 13𝑥 − 5

Factor the denominator:


𝑃𝑟𝑜𝑑𝑢𝑐𝑡 = 6(−5) = −30, 𝑆𝑢𝑚 = −15 + 2 = −13
2
6𝑥 − 15𝑥 + 2𝑥 − 5 = 3𝑥(2𝑥 − 5) + 1(2𝑥 − 5) = (3𝑥 + 1)(2𝑥 − 5)

Substitute the factored version in the denominator:


3𝑥 + 1 1
∫ 𝑑𝑥 = ∫ 𝑑𝑥
(3𝑥 + 1)(2𝑥 − 5) 2𝑥 − 5

Use the constant multiple rule:


1 2 ln|2𝑥 − 5|
∫ 𝑑𝑥 = +𝐶
2 2𝑥 − 5 2

Example 1.23: Simplification


𝑒 3 𝑒 −1 𝑒 3 1 𝑒4 1 𝑒4 − 1
− = − = − =
−2 −2 −2 −2𝑒 −2𝑒 −2𝑒 −2𝑒
4
3 1 𝑒 − 1
𝑒 3 𝑒 −1 𝑒 3 − 𝑒 −1 𝑒 − 𝑒 𝑒 𝑒4 − 1 1 𝑒4 − 1
− = = = = × =
−2 −2 −2 −2 −2 𝑒 −2 −2𝑒

E. Some Further Questions

Example 1.24
∫|𝑥| 𝑑𝑥

The absolute value function is defined piece-wise to be:


𝑥, 𝑥 ≥ 0
|𝑥| = {
−𝑥, 𝑥 < 0
We integrate it using cases.
Get all the files at: https://bit.ly/azizhandouts

For 𝑥 ≥ 0:
𝑥2
∫|𝑥| 𝑑𝑥 = ∫ 𝑥 𝑑𝑥 = +𝐶
2
For 𝑥 < 0:
𝑥2
∫|𝑥| 𝑑𝑥 = ∫ −𝑥 𝑑𝑥 = − +𝐶
2
F. Initial Value Problems
So far we have not paid any attention to the constant of integration. If we have some further data related to
the question, we can get the value of the constant of integration. Such questions are called initial value
problems.

Example 1.25
Determine 𝑔(0) given that:
𝑓(𝑥) = ∫ 𝑥 𝑑𝑥 , 𝑓(2) = 4, 𝑔(𝑥) = ∫ 𝑓(𝑥) 𝑑𝑥 , 𝑔(1) = 1

Find 𝒇(𝒙)
Integrate the first equation to find an expression for 𝑓(𝑥):
𝑥2
𝑓(𝑥) = ∫ 𝑥 𝑑𝑥 = + 𝐶1
2
𝑥2
To find the value of 𝐶1 , use the data given that 𝑓(2) = 4 and substitute it in 2
+ 𝐶1 :
2
2
𝑓(2) = 4 ⇒ + 𝐶1 = 4 ⇒ 𝐶1 = 2
2

Find 𝒈(𝒙)
Now that we have the explicit definition for 𝑓(𝑥), we can integrate it to find 𝑔(𝑥):

𝑥2 1 𝑥3 𝑥3
𝑔(𝑥) = ∫ 𝑓(𝑥) 𝑑𝑥 = ∫ + 2 𝑑𝑥 = + 2𝑥 + 𝐶2 = + 2𝑥 + 𝐶2
2 2 3 6
𝑥3
To find the value of 𝐶2 , use the data given that 𝑔(1) = 1 and substitute it in 6 + 2𝑥 + 𝐶2 :
1 7
𝑔(1) = 1 ⇒ + 2 + 𝐶2 = 1 ⇒ 𝐶2 = −
6 6
𝑥3 7
𝑔(𝑥) = + 2𝑥 −
6 6
Find 𝒈(𝟎)

Substitute 𝑥 = 0 in the above to determine the value asked in the question:


7
𝑔(0) = −
6

Example 1.26
𝑑𝑦
Find 𝑦(2) given that = 5𝑥 − 2, 𝑦(3) = 1.
𝑑𝑥

𝑑𝑦
Integrate both sides of 𝑑𝑥 = 5𝑥 − 2 with respect to 𝑥:
𝑑𝑦
∫ 𝑑𝑥 = ∫ 5𝑥 − 2 𝑑𝑥
𝑑𝑥
Get all the files at: https://bit.ly/azizhandouts

5𝑥 2
∫ 1 𝑑𝑦 = − 2𝑥 + 𝐶
2
5𝑥 2
𝑦= − 2𝑥 + 𝐶
2
Substitute 𝑦 = 3, 𝑥 = 1, and solve for C:
5(1)2 1 5
3= − 2(1) + 𝐶 ⇒ 3 = + 𝐶 ⇒ 𝐶 =
2 2 2
Evaluate 𝑦(2):
5𝑥 2 4 5(2)2 5
𝑦(2) = − 2𝑥 + = − 2(2) + =
2 2 2 2

G. Trigonometric Antiderivatives

1.27: Trigonometric Derivatives: Summary


𝑑 𝑑
(sin 𝑥) = cos 𝑥 , (cos 𝑥) = − sin 𝑥
𝑑𝑥 𝑑𝑥
𝑑 𝑑
(sec 𝑥) = sec 𝑥 tan 𝑥 , (csc 𝑥) = − csc 𝑥 cot 𝑥
𝑑𝑥 𝑑𝑥
𝑑 𝑑
(tan 𝑥) = sec 2 𝑥 , (cot 𝑥) = − csc 2 𝑥
𝑑𝑥 𝑑𝑥

➢ The above are the standard forms of the derivatives of the trigonometric functions.
Note that the functions on the right (the co-functions) each have a negative sign in their derivative.

Example 1.28: Basic Formulas


∫ sec 2 𝑥 + sec 𝑥 tan 𝑥 + csc 𝑥 cot 𝑥 𝑑𝑥

tan 𝑥 + sec 𝑥 − csc 𝑥 + 𝐶

Example 1.29: Constant Multiple Rule


cos 𝜃
∫ 2 sin 𝜃 + 𝑑𝜃
2

Split the integral:


cos 𝜃
∫ 2 sin 𝜃 𝑑𝜃 + ∫ 𝑑𝜃
2
Use the constant multiple rule:
1
2 ∫ sin 𝜃 𝑑𝜃 + ∫ cos 𝜃 𝑑𝜃
2
Integrate:
1
−2 cos 𝜃 + sin 𝜃 + 𝐶
2
Rewrite:
1
sin 𝜃 − 2 cos 𝜃 + 𝐶
2

Example 1.30: Splitting Fractions


Get all the files at: https://bit.ly/azizhandouts

1 + sin2 𝑥
∫ 𝑑𝑥
sin2 𝑥

1
∫( + 1) 𝑑𝑥 = ∫(csc 2 𝑥 + 1) 𝑑𝑥 = − cot 𝑥 + 𝑥 + 𝐶
sin2 𝑥

Example 1.31: Trig Identities


sin2 𝑥 + cos 2 𝑥
∫ 𝑑𝑥
cos2 𝑥

Substitute sin2 𝑥 + cos 2 𝑥 = 1:


1
∫ 𝑑𝑥 = ∫ sec 2 𝑥 𝑑𝑥 = tan 𝑥 + 𝐶
cos 2 𝑥

∫ 1 + tan2 𝜃 𝑑𝜃

∫ 1 + tan2 𝜃 𝑑𝜃 = ∫ sec 2 𝜃 𝑑𝜃 = tan 𝜃 + 𝐶

Example 1.32: Simplification


csc 𝑥
∫ 𝑑𝑥
csc 𝑥 − sin 𝑥

Simplify the integrand:


1 1
sin 𝑥 sin 𝑥 1 1
= 2 = = = sec 2 𝑥
1 1 − sin 𝑥 1 − sin 𝑥 cos2 𝑥
2
sin 𝑥 − sin 𝑥 sin 𝑥

csc 𝑥
∫ 𝑑𝑥 = ∫ sec 2 𝑥 𝑑𝑥 = tan 𝑥 + 𝐶
csc 𝑥 − sin 𝑥

∫ sin 𝑥 (cot 𝑥 + csc 𝑥 + 1) 𝑑𝑥

Multiplying:
∫ sin 𝑥 cot 𝑥 + sin 𝑥 csc 𝑥 + sin 𝑥 𝑑𝑥
cos 𝑥 1
∫ sin 𝑥 + sin 𝑥 + sin 𝑥 𝑑𝑥
sin 𝑥 sin 𝑥
Simplify:
∫ cos 𝑥 + 1 + sin 𝑥 𝑑𝑥
Integrate:
sin 𝑥 + 𝑥 − cos 𝑥 + 𝐶

tan2 𝑥 + sin2 𝑥
∫ 𝑑𝑥
sin2 𝑥
Get all the files at: https://bit.ly/azizhandouts

tan2 𝑥 sin2 𝑥
∫ + 𝑑𝑥
sin2 𝑥 sin2 𝑥
sin2 𝑥
2
∫ cos2 𝑥 + 1 𝑑𝑥
sin 𝑥

sin2 𝑥 1
∫ 2
× 2 + 1 𝑑𝑥
cos 𝑥 sin 𝑥
∫ sec 2 𝑥 + 1 𝑑𝑥
tan 𝑥 + 𝑥 + 𝐶

H. Inverse Trigonometric Functions

1.33: Summary of Inverse Trigonometric Derivatives


𝑑 1 𝑑 1
(sin−1 𝑥) = , (cos−1 𝑥) = −
𝑑𝑥 √1 − 𝑥 2 𝑑𝑥 √1 − 𝑥 2
𝑑 1 𝑑 1
(tan−1 𝑥) = , (cot −1 𝑥) = −
𝑑𝑥 1 + 𝑥2 𝑑𝑥 1 + 𝑥2
𝑑 1 𝑑 1
(sec −1 𝑥) = , (csc −1 𝑥) = −
𝑑𝑥 𝑥√𝑥 2 − 1 𝑑𝑥 |𝑥|√𝑥 2 − 1

➢ All of the cofunction derivatives have a minus sign in front, and the expression is otherwise the same
as the corresponding function.

Example 1.34
1
∫ 𝑑𝑥
√2 − 𝑥 2

1 1 1 1
∫ 𝑑𝑥 = ∫ 𝑑𝑥 = ∫ 𝑑𝑥
𝑥2 2 √2 2
√2 (1 − ) 𝑥 𝑥
2 √2√1 − ( ) √1 − ( )
√2 √2

𝑑 𝑥 1 1 1 1 1
(sin−1 ) = × = = =
𝑑𝑥 √2 √2 𝑥2 √2 − 𝑥 2
𝑥 2 𝑥 2 √2 (1 −
√1 − ( ) √2√1 − ( ) 2)
√2 √2

I. Preparing for 𝒖 substitution


In the next section, we will introduce u-substitution, which lets us run the chain rule in reverse. However, we
look at the idea by doing some questions with trial and error.

1.35: Constant Multiple Rule


∫ 𝑐𝑓(𝑥) 𝑑𝑥 = 𝑐 ∫ 𝑓(𝑥) 𝑑𝑥

➢ We can use the constant multiple to change our guess for an integral if it only differs by a constant
Get all the files at: https://bit.ly/azizhandouts

multiple.

Example 1.36
∫ sin 2𝑥 𝑑𝑥

We know that:
∫ sin 𝑥 𝑑𝑥 = − cos 𝑥 + 𝐶
Suppose we guess the integral as − cos 2𝑥
𝑑
(− cos 2𝑥) = sin(2𝑥) × 2
𝑑𝑥
This is exactly what we wanted, except that it is double of what we need. Hence, we divide both sides of the
above by 2:
𝑑 (− cos 2𝑥) sin(2𝑥) × 2
= = sin(2𝑥)
𝑑𝑥 2 2
Hence:
(− cos 2𝑥)
∫ sin 2𝑥 𝑑𝑥 = +𝐶
2

Example 1.37
𝑥
∫ sec 2 𝑑𝑥
2

𝑥 𝑥
∫ sec 2 𝑑𝑥 = 2 tan + 𝐶
2 2

∫ sec 𝜋𝑥 tan 𝜋𝑥 𝑑𝑥

sec 𝜋𝑥
∫ sec 𝜋𝑥 tan 𝜋𝑥 𝑑𝑥 = +𝐶
𝜋

2 𝑒𝑥
∫ cos 𝑑𝑥
3 𝜋

2 𝑒𝑥 2 𝑒𝑥 2 𝜋 𝑒𝑥
∫ cos 𝑑𝑥 = ∫ cos 𝑑𝑥 = ( ) ( ) sin
3 𝜋 3 𝜋 3 𝑒 𝜋

Example 1.38
∫(2𝑥 + 3)2 𝑑𝑥

Imagine that
𝑢3
𝑢 = 2𝑥 + 3 ⇒ 𝑢2 = (2𝑥 + 3)2 ⇒ ∫ 𝑢2 𝑑𝑢 = +𝐶
3
Based on the above, we can guess that the integral we want is:
𝑢3 (2𝑥 + 3)3
=
3 3
Get all the files at: https://bit.ly/azizhandouts

But, when we differentiate the above, we get:


𝑑 (2𝑥 + 3)3 3(2𝑥 + 3)2
= × 2 = 2(2𝑥 + 3)2
𝑑𝑥 3 3
Divide the above by 2:
𝑑 (2𝑥 + 3)3
𝑑𝑥 3 2(2𝑥 + 3)2
=
2 2
Simplify:
𝑑 (2𝑥 + 3)3
= (2𝑥 + 3)2
𝑑𝑥 6
Hence:
∫(2𝑥 + 3)2 𝑑𝑥 =

Example 1.39
𝑥 𝑥
∫ (cos + sin 𝜋𝑥 + sec 2 ) 𝑑𝑥
𝜋 2

We use a little trial and error


𝑑 𝑥 𝑥 1
𝑇𝑟𝑦 (sin ) = cos ×
𝑑𝑥 𝜋 𝜋 𝜋
1
The answer is off by a multiplicative factor of . To balance it out, we change our original guess to:
𝜋
𝑥
𝜋 sin
𝜋
𝑥 cos 𝜋𝑥 𝑥
𝜋 sin − + 2 tan + 𝐶
𝜋 𝜋 2
1.2 𝒖-substitution
A. Running the Chain Rule in Reverse

1.40: Integration by Substitution


∫ 𝑓(𝑔(𝑥)) 𝑔′ (𝑥) 𝑑𝑥 = ∫ 𝑓(𝑢) 𝑢′ 𝑑𝑥 = ∫ 𝑓(𝑢) 𝑑𝑢

Integration by substitution is based on the using the chain rule in reverse. In the above expression, in the
leftmost integral, let
𝑑𝑢
𝑢 = 𝑔(𝑥) ⇒ = 𝑔′ (𝑥) ⇒ 𝑑𝑢 = 𝑔′ (𝑥) 𝑑𝑥 = 𝑢′ 𝑑𝑥
𝑑𝑥
And then we make the substitution above.

Example 1.41
𝑥 𝑛+1
Use the power rule for integration ∫ 𝑥 𝑛 𝑑𝑥 = + 𝐶, 𝑛 ≠ −1 to integrate:
𝑛+1

∫ 2(2𝑥 + 4)3 𝑑𝑥

Decide the substitution:


𝑑𝑢
𝑢 = 2𝑥 + 4 ⇒ = 2 ⇒ 𝑑𝑢 = 2 𝑑𝑥
𝑑𝑥
Make the substitution:
Get all the files at: https://bit.ly/azizhandouts

(2𝑥 + 4)3 2⏟
∫⏟ 𝑑𝑥 = ∫ 𝑢3 𝑑𝑢
𝒖𝟑 𝒅𝒖
Integrate:
𝑢4
+𝐶
4
Change back to the original variable:
(2𝑥 + 4)4
= +𝐶
4

1.42: Creating the Pattern


If the expression that we have differs from the pattern we need by only a constant, then we can use the
property
1
∫ 𝑓(𝑥) 𝑑𝑥 = ∫ 𝑝 ∙ 𝑓(𝑥) 𝑑𝑥
𝑝

Note:
➢ The chain rule is useful when the derivative of 𝑢 is available to us in the integrand.
➢ Hence, the choice of 𝑢 is critical when using a 𝑢-subsitution.

Example 1.43: Creating the pattern


∫(5𝑥 + 2)9 𝑑𝑥

Use the substitution:


𝑑𝑢 𝑑𝑢
𝑢 = 5𝑥 + 2 ⇒ =5⇒ = 𝑑𝑥
𝑑𝑥 5

1 1 𝑢10 (5𝑥 + 2)10


(5𝑥 + 2)9 𝑑𝑥
∫⏟ ⏟ = ∫ 𝑢9 𝑑𝑢 = ( ) + 𝐶 = +𝐶
9 𝑑𝑢 5 5 10
⏟ ⏟ 50
𝑢
5 𝑰𝒏𝒕𝒆𝒈𝒓𝒂𝒕𝒆 𝑪𝒉𝒂𝒏𝒈𝒆 𝒃𝒂𝒄𝒌 𝒕𝒐
𝒐𝒓𝒊𝒈𝒊𝒏𝒂𝒍 𝒗𝒂𝒓𝒊𝒂𝒃𝒍𝒆

Example 1.44: Creating the pattern


A.
sin 2𝑥
B. ∫ 𝑑𝑥
2
2
sec 𝜋𝑥
C. ∫ 𝑒 𝑑𝑥
D. ∫ cos(𝜋𝑥 + 𝑒) 𝑑𝑥
E. ∫(5𝑥 4 + 4)(𝑥 5 + 4𝑥)8 𝑑𝑥
F. ∫ 𝑥(3𝑥 2 + 4)3 𝑑𝑥

Part A
𝑑𝑢
Use the substitution 𝑢 = 2𝑥 ⇒ 2 = 𝑑𝑥:
sin 2𝑥 1 𝑑𝑢 − cos 𝑢 − cos 2𝑥
∫ 𝑑𝑥 = ∫ sin 𝑢 = +𝐶 = +𝐶
2 2 2 4 4
Part B
Use the substitution 𝑢 = 𝜋𝑥 ⇒ 𝑑𝑢 = 𝜋 𝑑𝑥 to get:
Get all the files at: https://bit.ly/azizhandouts

sec 2 𝜋𝑥 1 1 tan 𝑢 tan 𝜋𝑥


∫ ∙ ∙ 𝜋 𝑑𝑥 = ∫ sec 2 𝑢 𝑑𝑢 = +𝐶 = +𝐶
𝑒 𝜋 𝑒𝜋 𝑒𝜋 𝑒𝜋
Part C
Substitute 𝑢 = 𝜋𝑥 + 𝑒 ⇒ 𝑑𝑢 = 𝜋 𝑑𝑥:
1 1 1
∫ cos(𝜋𝑥 + 𝑒) 𝑑𝑥 = ∫ cos 𝑢 𝑑𝑢 = sin 𝑢 + 𝐶 = sin(𝜋𝑥 + 𝑒) + 𝐶
𝜋 𝜋 𝜋
Part D
Substitute 𝑢 = 𝑥 5 + 4𝑥 ⇒ 𝑑𝑢 = 5𝑥 4 + 4 𝑑𝑥:
8
5
𝑢9 (𝑥 5 + 4𝑥)9
∫ (𝑥 (5𝑥 4 + 4) 𝑑𝑥 = ∫ 𝑢8 𝑑𝑢 =
⏟ + 4𝑥 ) ⏟ +𝐶 = +𝐶
𝑢 𝑑𝑢
9 9
Part E
Substitute 𝑢 = 3𝑥 2 + 4 ⇒ 𝑑𝑢 = 6𝑥 𝑑𝑥:
3
𝟏 2
1 1 𝑢4 (3𝑥 2 + 4)4
∫ (3𝑥
⏟ + 4) 𝟔𝑥 ⏟ 𝑑𝑥 = ∫ 𝑢3 𝑑𝑢 = ∙ +𝐶 = +𝐶
𝟔 𝑢 𝑑𝑢
6 6 4 24

1.45: Square Root in the Denominator


2 3
∫ √𝑢 𝑢′ 𝑑𝑥 = ∫ √𝑢 𝑑𝑢 = 𝑢2 + 𝐶
3

Example 1.46
A. ∫ cos 𝑥 √sin 𝑥 𝑑𝑥
B. ∫ 𝑥 2 √3𝑥 3 + 5 𝑑𝑥

Part A
Let 𝑢 = sin 𝑥 ⇒ 𝑑𝑢 = cos 𝑥 𝑑𝑥
1 2 3 2 3
∫ cos 𝑥 √sin 𝑥 𝑑𝑥 = ∫ √𝑢 𝑑𝑢 = ∫ 𝑢2 𝑑𝑢 = 𝑢2 + 𝐶 = sin2 𝑥 + 𝐶
3 3
Part B
Let 𝑢 = 3𝑥 3 + 5 ⇒ 𝑑𝑢 = 9𝑥 2 𝑑𝑥
1 1 1 2 3 2 3
∫ 𝟗𝒙𝟐 √3𝑥 3 + 5 𝒅𝒙 = ∫ √𝑢 𝑑𝑢 = × 𝑢2 + 𝐶 = (3𝑥 3 + 5)2 + 𝐶
9 9 9 3 27

1.47: Square Root in the Denominator


𝑢′ 1
∫ 𝑑𝑥 = ∫ 𝑑𝑢 = 2√𝑢
√𝑢 √𝑢

Example 1.48:
𝑑𝑥
∫ (𝑪𝑩𝑺𝑬 𝟐𝟎𝟐𝟎)
√𝑥(1 + √𝑥)

1
Let 𝑡 = √𝑥 + 1 ⇒= 𝑑𝑡 = 𝑑𝑥
2√𝑥
𝑑𝑥 𝑑𝑡
2∫ = 2∫ = 2 ln|𝑡| + 𝐶 = 2 ln|√𝑥 + 1| + 𝐶
2√𝑥(1 + √𝑥) 𝑡

𝑑𝑥

√𝑥 + 𝑥
Get all the files at: https://bit.ly/azizhandouts

Factor the denominator to get an expression equivalent to the earlier part of this example:
𝑑𝑥 𝑑𝑥
∫ =∫ = 2 ln|√𝑥 + 1| + 𝐶
√𝑥 + 𝑥 √𝑥(1 + √𝑥)

Example 1.49
𝑥3
∫ 𝑑𝑥
√1 − 2𝑥 2

𝑢 = 1 − 2𝑥 2 ⇒ 𝑑𝑢 = −4𝑥 𝑑𝑥
2𝑥 2 = 1 − 𝑢

1 𝑥2
− ∫ (−4𝑥 𝑑𝑥)
4 √1 − 2𝑥 2

Carry out the substitution:


1−𝑢
1 1 𝑢−1
− ∫ 2 𝑑𝑢 = ∫ 𝑑𝑢
4 √𝑢 8 √𝑢

1 1
= ∫ (√𝑢 − ) 𝑑𝑢
8 √𝑢
1 2 3
= ( 𝑢2 − 2√𝑢) + 𝐶
8 3
1 3 1
= (1 − 2𝑥 2 )2 − √1 − 2𝑥 2 + 𝐶
12 4

Example 1.50: Trigonometric Expressions


A. ∫(𝑥 + 1) sin(𝑥 2 + 2𝑥) 𝑑𝑥
B. ∫ sin3 𝑥 cos 𝑥 𝑑𝑥
C. ∫ tan6 𝑥 ∙ sec 2 𝑥 𝑑𝑥
𝑥 𝑥
D. ∫ sin5 (2) ∙ cos (2) 𝑑𝑥 (CBSE 2020)

Part A
Let 𝑢 = 𝑥 2 + 2𝑥 ⇒ 𝑑𝑢 = 2𝑥 + 2 𝑑𝑥 = 2(𝑥 + 1) 𝑑𝑥
1 1 1 1
⏟ 2 + 2𝑥) 2(𝑥
∫ sin(𝑥 ⏟ + 1) 𝑑𝑥 = − ∫ − sin 𝑢 𝑑𝑢 = − cos 𝑢 + 𝐶 = − cos(𝑥 2 + 2𝑥) + 𝐶
2 sin 𝑢 𝑑𝑢
2 2 2
Part B
Let 𝑢 = sin 𝑥 ⇒ 𝑑𝑢 = cos 𝑥 𝑑𝑥
1 sin4 𝑥
∫ sin3 𝑥 cos 𝑥 𝑑𝑥 = ∫ 𝑢3 𝑑𝑢 = 𝑢4 + 𝐶 = +𝐶
4 4
Part C
Let 𝑡 = tan 𝑥 ⇒ 𝑑𝑡 = sec 2 𝑥 𝑑𝑥
𝑡7 tan7 𝑥
∫ tan6 𝑥 ∙ sec 2 𝑥 𝑑𝑥 = ∫ 𝑡 6 𝑑𝑡 = +𝐶 = +𝐶
7 7
Part D
𝑥 1 𝑥
Let sin 2 = 𝑡 ⇒ 𝑑𝑡 = 2 cos 2 𝑑𝑥:
Get all the files at: https://bit.ly/azizhandouts

𝑥 𝑥 𝑡6 1 𝑥
∫ sin5 ( ) ∙ cos ( ) 𝑑𝑥 = 2 ∫ 𝑡 5 𝑑𝑡 = 2 ∙ + 𝐶 = (sin6 ) + 𝐶
2 2 6 3 2

B. Exponentials and Logarithms

Example 1.51: Exponentials


Use the pattern ∫ 𝑢′𝑒 𝑢 𝑑𝑥 = ∫ 𝑒 𝑢 𝑑𝑢 = 𝑒 𝑢 + 𝐶
A. ∫ cos 𝑥 𝑒 sin 𝑥 𝑑𝑥
B. ∫ 𝑒 4𝑥 𝑑𝑥
𝑒 √𝜋𝑥
C. ∫ 𝑑𝑥
√𝑒𝑥
sin 𝑥𝑒 √cos 𝑥
D. ∫ 𝑑𝑥
√cos 𝑥
1
E. ∫ [(𝑒 2𝑥 + 2) sec(𝑒 2𝑥 + 𝑥 + 1) tan(𝑒 2𝑥 + 𝑥 + 1)] 𝑑𝑥

Part A
𝑑𝑢
Let 𝑢 = sin 𝑥 ⇒ 𝑑𝑥 = cos 𝑥 ⇒ 𝑑𝑢 = cos 𝑥 𝑑𝑥:

∫ cos 𝑥 𝑒 sin 𝑥 𝑑𝑥 = ∫ 𝑒 𝑢 𝑑𝑢 = 𝑒 𝑢 + 𝐶 = 𝑒 sin 𝑥 + 𝐶


Part B
𝑑𝑢
Let 𝑢 = 4𝑥 ⇒ = 4 ⇒ 𝑑𝑢 = 4 𝑑𝑥
𝑑𝑥
1 1 1 1
∫ 𝑒 4𝑥 𝑑𝑥 = ∫ 𝟒 × 𝑒 4𝑥 𝒅𝒙 = ∫ 𝑒 𝑢 𝑑𝑢 = 𝑒 𝑢 + 𝐶 = 𝑒 4𝑥 + 𝐶
4 4 4 4
Part C
Let 𝑢 = √𝜋𝑥
𝜋 1 √𝜋 √𝜋 1
⇒ 𝑑𝑢 = ∙ = = ( ) ( ) 𝑑𝑥
2 √𝜋𝑥 2√𝑥 2 √𝑥
𝜋𝑥 𝜋𝑥
𝑒 √ 2 √𝜋𝑒 √ 2 2 𝑢 2𝑒 √𝜋𝑥
∫ 𝑑𝑥 = ∫ 𝑑𝑥 = ∫ 𝑒 𝑢 𝑑𝑢 = 𝑒 +𝐶 = +𝐶
√𝑒𝑥 √𝑒𝜋 2√𝑥 √𝑒𝜋 √𝑒𝜋 √𝑒𝜋
Part D
sin 𝑥
Substitute 𝑢 = √cos 𝑥 ⇒ 𝑑𝑢 = − 2 𝑑𝑥
√cos 𝑥
sin 𝑥 𝑒 √cos 𝑥
−2 ∫ − 𝑑𝑥 = −2 ∫ 𝑒 𝑢 𝑑𝑢 = −2𝑒 𝑢 + 𝐶 = −2𝑒 √cos 𝑥 + 𝐶
2√cos 𝑥
Part E
1
Let 𝑢 = 𝑒 2𝑥 + 𝑥 + 1 ⇒ 𝑑𝑢 = 2𝑒 𝑥 + 1 𝑑𝑥 = 2 (𝑒 2𝑥 + 2) 𝑑𝑥
1 1 1
∫ sec 𝑢 tan 𝑢 𝑑𝑢 = sec 𝑢 + 𝐶 = sec(𝑒 2𝑥 + 𝑥 + 1) + 𝐶
2 2 2

1.52: Integrals leading to Logarithms


If the numerator is the derivative of the denominator, then we get a straight-forward pattern that leads to a
logarithm for the integral:
𝑢′ 1
∫ 𝑑𝑥 = ∫ 𝑑𝑢 = ln|𝑢| + 𝐶
𝑢 𝑢

1
Using ∫ 𝑥 𝑑𝑥 = ln|𝑥| + 𝐶, we can integrate
Get all the files at: https://bit.ly/azizhandouts

4 1
∫ 𝑑𝑥 = 4 ∫ 𝑑𝑥 = 4 ln|𝑥| + 𝐶
𝑥 𝑥

Whenever the numerator is the derivative of the denominator, it will lead to a logarithm in the answer.
Use the pattern
𝑢′ 1
∫ 𝑑𝑥 = ∫ 𝑑𝑢 = ln|𝑢| + 𝐶
𝑢 𝑢

Example 1.53:
6𝑥 2 +3
A. ∫ 2𝑥 3 +3𝑥 𝑑𝑥
1
B. ∫ 3𝑥+2 𝑑𝑥
√𝑥

Part A
Let 𝑢 = 2𝑥 3 + 3𝑥 ⇒ 𝑑𝑢 = 6𝑥 2 + 3 𝑑𝑥
𝟔𝒙𝟐 + 𝟑 1
∫ 3 𝒅𝒙 = ∫ 𝑑𝑢 = ln|𝑢| + 𝐶 = ln|2𝑥 3 + 3𝑥| + 𝐶
2𝑥 + 3𝑥 𝑢
Part B
This doesn’t seem to fit the pattern, but since the question asks us to, it should. Factor out √𝑥:
1
∫ 𝑑𝑥
√𝑥(3√𝑥 + 2)
3 1
Let 𝑢 = 3√𝑥 + 2 ⇒ 𝑑𝑢 = 2 ∙ 𝑑𝑥
√𝑥
Then:
2 1 3 1 2 1 2 2
∫ ∙ ∙ 𝑑𝑥 = ∫ 𝑑𝑢 = ln|𝑢| + 𝐶 = ln|3√𝑥 + 2| + 𝐶
3 ⏟ (3√𝑥 + 2) 2
⏟ √𝑥 3 𝑢 3 3
𝑢 𝑑𝑢

Example 1.54
𝑇𝑟𝑢𝑒 𝑜𝑟 𝐹𝑎𝑙𝑠𝑒
𝑢 1
∫ 𝑑𝑥 = ∫ 𝑑𝑢 = ln|𝑢| + 𝐶
𝑢′ 𝑢

𝑑𝑦 1 𝑑𝑢 1 𝑢′ 𝑢
ln|𝑢| = × = × 𝑢′ = ≠ ′ ⇒ 𝐹𝑎𝑙𝑠𝑒
𝑑𝑥 𝑢 𝑑𝑥 𝑢 𝑢 𝑢

C. Trigonometric Integrals
Integrals of trigonometric functions defined as ratios can lead to logarithms since derivatives of the
trigonometric functions are also trigonometric functions. We now find the integrals of the four trigonometric
functions that we have not yet found.

1.55: Integrals of the Remaining Trigonometric functions1


∫ tan 𝑥 𝑑𝑥 = ln|sec 𝑥| + 𝐶

∫ cot 𝑥 𝑑𝑥 = ln|sin 𝑥| + 𝐶

1
While the integrals are mentioned as a property, the logic behind them is important. Make sure you can derive these
whenever needed.
Get all the files at: https://bit.ly/azizhandouts

∫ sec 𝑥 𝑑𝑥 = ln|sec 𝑥 + tan 𝑥| + 𝐶

Part A
sin 𝑥
∫ tan 𝑥 𝑑𝑥 = ∫ 𝑑𝑥
cos 𝑥
Let 𝑢 = cos 𝑥 ⇒ 𝑑𝑢 = − sin 𝑥 𝑑𝑥:
1 1
= −∫ ∙ (−
⏟ sin 𝑥)𝑑𝑥 = − ∫ 𝑑𝑢 = − ln|𝑢| + 𝐶 = − ln|cos 𝑥| + 𝐶
cos
⏟𝑥 𝑑𝑢
𝑢
𝑢
Using the power rule of logarithms: 𝑎 ln 𝑥 = ln 𝑥 𝑎
1
= ln | | + 𝐶 = ln|sec 𝑥| + 𝐶
cos 𝑥
Part B
cos 𝑥
∫ cot 𝑥 𝑑𝑥 = ∫ 𝑑𝑥
sin 𝑥
Let 𝑢 = sin 𝑥 ⇒ 𝑑𝑢 = cos 𝑥 𝑑𝑥:
1 1
∫ ×⏟cos 𝑥 𝑑𝑥 = ∫ 𝑑𝑢 = ln|𝑢| + 𝐶 = ln|sin 𝑥| + 𝐶 = − ln|csc 𝑥| + 𝐶
sin
⏟𝑥 𝑑𝑢
𝑢
𝑢
Part C
sec 𝑥 + tan 𝑥 sec 2 𝑥 + sec 𝑥 tan 𝑥
∫ sec 𝑥 𝑑𝑥 = ∫ sec 𝑥 𝑑𝑥 = ∫ 𝑑𝑥
sec 𝑥 + tan 𝑥 sec 𝑥 + tan 𝑥
Let 𝑢 = sec 𝑥 + tan 𝑥 ⇒ 𝑑𝑢 = sec 𝑥 tan 𝑥 + sec 2 𝑥 𝑑𝑥 :
1
= ∫ 𝑑𝑢 = ln|𝑢| + 𝐶 = ln|sec 𝑥 + tan 𝑥| + 𝐶
𝑢

Example 1.56: Trigonometric Expressions


1 1
A. ∫ 2 cot ( ) 𝑑𝑥
𝑥 𝑥
𝑥+cos 6𝑥
B. ∫ 3𝑥 2 +sin 6𝑥 𝑑𝑥 (CBSE 2012)

Part A
1 1
Let 𝑢 = 𝑥 ⇒ 𝑑𝑢 = − 𝑥 2 𝑑𝑥
1 1 1
− ∫ cot ( ) (− 2 ) 𝑑𝑥 = − ∫ cot 𝑢 𝑑𝑢 = − ln|sin 𝑢| + 𝐶 = ln |csc | + 𝐶
⏟𝑥 ⏟ 𝑥 𝑥
𝑢 𝑑𝑢
Part B
Let 𝑡 = 3𝑥 2 + sin 6𝑥 = 𝑡 ⇒ 𝑑𝑡 = 6(𝑥 + cos 6𝑥)𝑑𝑥
1 6(𝑥 + cos 6𝑥) 1 1 1 1
∫ 2
𝑑𝑥 = ∫ 𝑑𝑡 = ln|𝑡| + 𝐶 = ln|3𝑥 2 + sin 6𝑥| + 𝐶
6 3𝑥 + sin 6𝑥 6 𝑡 6 6

Example 1.57
𝑑𝑥
Find ∫ 𝑥(1+𝑥2 ) (CBSE 2020)

1 2
Let 𝑡 = + 1 ⇒ 𝑑𝑡 = − 3 𝑑𝑥
𝑥2 𝑥
𝑑𝑥 𝑑𝑥 1 2 𝑑𝑥 1 1 1
∫ =∫ = − ∫− = − ∫ 𝑑𝑡 = − ln|𝑡| + 𝐶
2
𝑥(1 + 𝑥 ) 1 2 1 2 𝑡 2
𝑥 3 ( 2 + 1) 𝑥 3 ( 2 + 1)
𝑥 𝑥
Change back to the original variable:
Get all the files at: https://bit.ly/azizhandouts

1 1 1 𝑥2 + 1
− ln | 2 + 1| + 𝐶 = − ln | 2 | + 𝐶
2 𝑥 2 𝑥
Use the power rule for logarithms:
𝑥2
= ln √ 2 +𝐶
𝑥 +1

Example 1.58

D. Long Division

Example 1.59:Basics
𝑥
∫ 𝑑𝑥
3𝑥 + 5

Divide
Multiply and divide the given expression by 3:
1 3𝑥
( )
3 3𝑥 + 5
Add and subtract 5 in the numerator:
1 3𝑥 + 5 − 5
= ( )
3 3𝑥 + 5
Split the fraction:
1 5
= (1 − )
3 3𝑥 + 5
Integrate:
𝑥 1 5
Substitute = (1 − ) in the given integral:
3𝑥+5 3 3𝑥+5
1 5
∫ (1 − ) 𝑑𝑥
3 3𝑥 + 5
Split the integral:
1 5 1
∫ 1 𝑑𝑥 − ∫ 𝑑𝑥
3 3 3𝑥 + 5
Evaluate each integral:
1 5 1
𝑥 − ∙ ln|3𝑥 + 5| + 𝐶
3 3 3
Simplify:
1 5
𝑥 − ln|3𝑥 + 5| + 𝐶
3 9

Example 1.60: Polynomial Long Division


𝑥 2 + 3𝑥 + 2
∫ 𝑑𝑥
2𝑥 − 1
Get all the files at: https://bit.ly/azizhandouts

Rewrite the expression using polynomial long division:


𝑥 7 15 1
∫ ( + ) 𝑑𝑥 + ∫ 𝑑𝑥
2 4 4 2𝑥 − 1
Integrate:
𝑥 2 7𝑥 15 1
= + + ∙ ln|2𝑥 − 1| + 𝐶
4 4 4 2

Simplify:
𝑥 2 7𝑥 15
= + + ln|2𝑥 − 1| + 𝐶
4 4 8
E. Integration by Completing the Square

1.61: Summary of Inverse Trigonometric Derivatives


𝑑 1 𝑑 1
(sin−1 𝑥) = , (cos−1 𝑥) = −
𝑑𝑥 √1 − 𝑥 2 𝑑𝑥 √1 − 𝑥 2
𝑑 1 𝑑 1
(tan−1 𝑥) = , (cot −1 𝑥) = −
𝑑𝑥 1 + 𝑥2 𝑑𝑥 1 + 𝑥2
𝑑 1 𝑑 1
(sec −1 𝑥) = , (csc −1 𝑥) = −
𝑑𝑥 𝑥√𝑥 2 − 1 𝑑𝑥 |𝑥|√𝑥 2 − 1

1.62: Integrals leading to 𝒕𝒂𝒏−𝟏 𝒙


1 1 𝑥
∫ 𝑑𝑥 = tan−1 ( ) + 𝐶
𝑎2 + 𝑥 2 𝑎 𝑎

Factor out 𝑎2 from the denominator to get a 1:


1
∫ 𝑑𝑥
𝑥 2
𝑎2 [1 + ( ) ]
𝑎
𝑥 1
Substitute 𝑢 = ⇒ 𝑑𝑢 = 𝑑𝑥 ⇒ 𝑑𝑥 = 𝑎 𝑑𝑢:
𝑎 𝑎
𝑎 𝑑𝑢 𝑑𝑢 1
∫ 2 2
=∫ 2
= tan−1 𝑢 + 𝐶
𝑎 (1 + 𝑢 ) 𝑎(1 + 𝑢 ) 𝑎
𝑥
Substitute 𝑢 = 𝑎:
1 𝑥
= tan−1 ( ) + 𝐶
𝑎 𝑎

Example 1.63
1
∫ 𝑑𝑥
𝑥2 + 4𝑥 + 8

Rewrite the expression in the denominator 𝑏 by completing the square:


1 1
∫ 2 𝑑𝑥 = ∫ 𝑑𝑥
𝑥 + 4𝑥 + 4 + 4 (𝑥 + 2)2 + 22
1 1 𝑥
Use the formula ∫ 𝑎2 +𝑥2 𝑑𝑥 = 𝑎 tan−1 (𝑎) + 𝐶 with 𝑎 = 2:
1 𝑥+2
= tan−1 ( )+𝐶
2 2

1.64: Integrals leading to 𝒔𝒊𝒏−𝟏 𝒙


Get all the files at: https://bit.ly/azizhandouts

1 𝑥
∫ 𝑑𝑥 = sin−1 +𝐶
√𝑎2 − 𝑥2 𝑎

1 1 1
∫ 𝑑𝑥 = ∫ 𝑑𝑥 = ∫ 𝑑𝑥
√𝑎2 − 𝑥 2 𝑥2 𝑥 2
√𝑎2 (1 − 2 ) 𝑎√1 − (𝑎)
𝑎
𝑥 1
Substitute 𝑢 = 𝑎 ⇒ 𝑑𝑢 = 𝑎 𝑑𝑥 ⇒ 𝑑𝑥 = 𝑎 𝑑𝑢:
𝑎 𝑑𝑢 𝑑𝑢
∫ =∫
𝑎√1 − 𝑢2 √1 − 𝑢2
Integrate and change back to the original variable:
𝑥
= sin−1 𝑢 + 𝐶 = sin−1 + 𝐶
𝑎

Example 1.65
1
∫ 𝑑𝑥
√−𝑥 2 − 6𝑥 − 4

Complete the square for the quantity inside the square root:
2
−(𝑥 2 + 6𝑥 + 4) = −(𝑥 2 + 6𝑥 + 9 − 5) = −[(𝑥 + 3)2 − 5] = (√5) − (𝑥 + 3)2
Substitute:
1
∫ 𝑑𝑥
2
√(√5) − (𝑥 + 3)2
1 𝑥
Use the formula ∫ 𝑑𝑥 = sin−1 𝑎 + 𝐶 with 𝑎 = √5
√𝑎 2 −𝑥 2
𝑥+3
= sin−1 +𝐶
√5

F. Initial Value Problems

Example 1.66
𝑑𝑦 1
Find the particular solution for 𝑑𝑥 = sin 2𝑥 given the initial value condition 𝑦(0) = 2.

𝑦 = ∫ sin 2𝑥 𝑑𝑥
Let 𝑢 = 2𝑥 ⇒ 𝑑𝑢 = 2 𝑑𝑥:
1 1 1
𝑦 = ∫ sin 𝑢 𝑑𝑢 = (− cos 𝑢) + 𝐶 = (− cos 2𝑥) + 𝐶
2 2 2
1
Substitute 𝑦 = 0, 𝑥 = 2 to find the value of C:
1 1
0 = (− cos 0) + 𝐶 ⇒ 𝐶 =
2 2
The particular solution:
1 1
𝑦 = (− cos 2𝑥) +
2 2

G. Challenging Problems

Example 1.67
Get all the files at: https://bit.ly/azizhandouts

1.3 Area under a Curve and Reimann Sums


A. Area from Rectangles: Partitions and Heights
Numerical analysis is a subject that uses algorithms to calculate approximate values of interest. We look at
some basics related to the subject that lead us to the definition of the integral. These ideas can be extended
into the topic of numerical integration (which we will see later).

For many applications, we want to calculate areas. Euclidean geometry gives us the area for a number of
geometrical shapes:
➢ Square = 𝑆𝑖𝑑𝑒 2
➢ Rectangle = 𝐿𝑒𝑛𝑔𝑡ℎ × 𝑊𝑖𝑑𝑡ℎ
𝑏1 +𝑏2
➢ Trapezoid = ℎ ∙ 2
➢ Circle = 𝜋𝑟 2

We now look at calculating more general areas than the specific shapes listed above.

1.68: “Area under the Curve”


The “area under the curve” of a function is taken to mean the area
between the curve and the 𝑥-axis.

The diagram on the right shows the blue area as the area between
the function 𝑦 = 𝑥 2 and the 𝑥 axis
𝑓𝑟𝑜𝑚 𝑥 = 0 𝑡𝑜 𝑥 = 2

1.69: Partition
To approximate area under the curve, we can divide the length
under consideration into different parts. Each part is called a
partition.

➢ Partitions can have different shapes.


➢ We will look at rectangular and trapezoidal partitions, but these are not the only types of partitions.

1.70: Approximation using Rectangles


Since the area of a rectangle is easy to calculate, we can approximate the area under the curve using
Rectangles. The number of rectangles is given by the number of partitions that we divide the length into.

1.71: Right Endpoints


We use right endpoints when we approximate the area under the curve using rectangles with height defined
at the right endpoint of the interval.

1.72: Upper Sum


An upper sum is an approximation that is larger than the true area.

1.73: Increasing Function


A function 𝑓 is called an increasing function for all numbers 𝑥1 , 𝑥2 in the domain of 𝑓, we have:
Get all the files at: https://bit.ly/azizhandouts

𝑥1 > 𝑥2 ⇒ 𝑓(𝑥1 ) > 𝑓(𝑥2 )

Example 1.74
The area under the curve of 𝑓(𝑥) = 𝑥 2 from 𝑥 = 0 to 𝑥 = 1 has been approximated with 𝑛 rectangles in the
given diagrams using right endpoints.

A. Explain why the diagram shows that right endpoints have been used.
B. Explain why the estimation is an upper sum.
C. Explain the connection between the answers to Parts A and B using the definition of increasing
function to help you.
D. Calculate the area when 𝑛 = 2, 𝑛 = 3, 𝑛 = 4
E. What is the trend in the areas?
F. Without calculating, what will be the area from 𝑛 > 4. Will it less or more than the areas that you
calculated in Part A.
G. As 𝑛 → ∞, what does the width of the rectangles tend to. What does the total area that you calculate
tend to?

𝑛 𝑊𝑖𝑑𝑡ℎ (𝑥, 𝑦) Area


2 1 1 1 1 1 1 1 1 5
( , ) , (1,1) ( ) ( ) + ( ) (1) = + = = 0.625
2 2 4 2 4 2 8 2 8
3 1 1 1 2 4 1 1 1 4 1 1 14 14
( , ) , ( , ) , (1,1) ( ) ( ) + ( ) ( ) + ( ) (1) = ( ) ( ) =
3 3 9 3 9 3 9 3 9 3 3 9 27
4 1 1 1 1 1 3 9 1 1 1 9 15
( , ) , ( , ) , ( , ) , (1,1) ( )( + + + 1) =
4 4 16 2 4 4 16 4 16 4 16 32

Write in summation notation

2
1 𝑡
∑ ∙𝑓( )
2 2
𝑡=1
3
1 𝑡
∑ ∙𝑓( )
3 3
𝑡=1
4
1 𝑡
∑ ∙𝑓( )
4 4
𝑡=1

Example 1.75
Approximate the area under the curve 𝑦 = 𝑥 3 from 𝑥 = 1 to 𝑥 = 3 using right endpoints with 𝑛 rectangles.
A. Calculate the approximation for 𝑛 = 2, 𝑛 = 3, 𝑛 = 4.

𝑛 𝑊𝑖𝑑𝑡ℎ (𝑥, 𝑦) Area


2 1 (2,8), (3,27) 1(8 + 27) = 35
Get all the files at: https://bit.ly/azizhandouts

3 2 5 125 7 343 2 125 343 266


( , ),( , ) , (3,27) ( + + 27) =
3 3 27 3 27 3 27 27 9
4 1 3 27 5 125 1 27 125
( , ) , (2,8), ( , ) , (3,27) ( +8+ + 27)
2 2 8 2 8 2 8 8

1.76: Left Endpoints


We use left endpoints when we approximate the area under the curve using rectangles with height defined at
the left endpoint of the interval.

Example 1.77
Approximate the area under the curve 𝑦 = 𝑥 2 from 𝑥 = 0 to 𝑥 = 1 using left endpoints with 2, 3 and 4
rectangles.

𝑛 𝑊𝑖𝑑𝑡ℎ (𝑥, 𝑦) Area


2 1 1 1 1 1 1 1
(0,0), ( , ) ( ) (0) + ( ) ( ) = = 0.125
2 2 4 2 2 4 8
3 1 1 1 2 4 1 1 1 1 4 1 5 5
(0,0), ( , ) , ( , ) ( ) (0) + ( ) ( ) + ( ) ( ) = ( ) ( ) =
3 3 9 3 9 3 3 9 3 9 3 9 27
4 1 1 1 1 1 3 9 1 1 1 9 7
(0,0), ( , ) , ( , ) , ( , ) ( ) (0 + + + )=
4 4 16 2 4 4 16 4 16 4 16 32

Write in summation notation

1
1 𝑡
∑ ∙𝑓( )
2 2
𝑡=0
2
1 𝑡
∑ ∙𝑓( )
3 3
𝑡=0
3
1 𝑡
∑ ∙𝑓( )
4 4
𝑡=0

Example 1.78
Approximate the area under the curve 𝑦 = 𝑥 3 from 𝑥 = 1 to 𝑥 = 2 using left endpoints with 𝑛 rectangles.
A. Calculate the approximation for 𝑛 = 2, 𝑛 = 3, 𝑛 = 4.

𝑛 𝑊𝑖𝑑𝑡ℎ (𝑥, 𝑦) Area


2 1 3 27 1 27
(1,1), ( , ) (1 + ) =
2 2 8 2 8
3 1 4 64 5 125 1 64 125
(1,1), ( , ) , ( , ) (1 + + )=
3 3 27 3 27 3 27 27
Get all the files at: https://bit.ly/azizhandouts

4 1 5 125
(1,1), ( ,
3 27 7 343
) , (2 , 8 ) , (4 , 64 ) 1 125 27 343 187
464 [1 + + + ]= = 2.921875
4 4 64 8 64 64

1.79: Midpoints
We use midpoints when we approximate the area under the curve using rectangles with height defined at the
midpoint of the interval.

Example 1.80
Approximate the area under the curve 𝑦 = 𝑥 2 from 𝑥 = 0 to 𝑥 = 1 using midpoints with 2, 3 and 4 rectangles.

𝑛 𝑊𝑖𝑑𝑡ℎ (𝑥, 𝑦) Area


2 1 1 1 3 9 1 1 1 9
( , ),( , ) ( )( )+ ( )( )
2 4 16 4 16 2 16 2 16
3 1 1 1 1 1 5 25 1 1 1 25
( , ),( , ),( , ) ( )( + + ) =
3 6 36 2 4 6 36 3 36 4 36
4 1
4

Example 1.81
Approximate the area under the curve 𝑦 = 𝑥 3 from 𝑥 = 1 to 𝑥 = 2 using midpoint with 𝑛 rectangles.
A. Calculate the approximation for 𝑛 = 2, 𝑛 = 3, 𝑛 = 4.

𝑛 𝑊𝑖𝑑𝑡ℎ (𝑥, 𝑦) Area


2 1 5 125 7 343
( , ),( , )
2 4 64 4 64
3 1 7 343 3 27 11 1331
( , ),( , ),( , )
3 6 216 2 8 6 216
4 1 9 729 11 1331 13 15
( , ),( , ),( ,),( ,)
4 8 512 8 512 8 8

1.82 : Trapezoids
We use the trapezoidal rule when we approximate the area under the curve using trapezoids with height at
left and right endpoints of the given interval.

Example 1.83
Approximate the area under the curve 𝑦 = 𝑥 2 from 𝑥 = 0 to 𝑥 = 1 using trapezoids with 2, 3 and 4 trapezoids.
Get all the files at: https://bit.ly/azizhandouts

𝑛 𝑤 𝑏1 + 𝑏2 1
𝐴𝑟𝑒𝑎 = ∙ 𝑤 ∙ (𝑏1 + 𝑏2 )
2
2 1 1 1 1 5 1 1 1 5 6 3
0+ = , +1= ( )( )( + ) = =
2 4 4 4 4 2 2 4 4 16 8
3 1
3
4 1
4

Example 1.84
1
Find the area under the curve 𝑥 over the interval 𝑥 = 1 to 𝑥 = 3 using 4 rectangles.

3−1 2
𝑊𝑖𝑑𝑡ℎ 𝑜𝑓 𝑅𝑒𝑐𝑡𝑎𝑛𝑔𝑙𝑒 = = = 0.5
4 4

1 1.5 2 2.5 3
1 2 1 2 1
3 2 5 3

2 2 1 1 2 2 1
1+3 +2 2 + +
0.5 ∙ + 0.5 ∙ 3 + 0.5 ∙ 5 + 0.5 ∙ 5 3
2 2 2 2

B. Upper and Lower Sums: Darboux

C. Some Online Tools


This integration calculator will let you decide the function, the interval, the number of rectangles, and the
endpoint method.
It is a good way of practicing area under the curve approximation questions.

D. Summation Notation
Get all the files at: https://bit.ly/azizhandouts

1.85: Summation independent of the variable


𝑘

∑ 𝑥 = 𝑘𝑥
𝑛=1

∑𝑥 =⏟
𝑥 + 𝑥 + ⋯ + 𝑥 = 𝑘𝑥
𝑛=1 𝑘 𝑡𝑖𝑚𝑒𝑠

1.86: Property I: Constant Property


A constant can be moved out of, or into the summation sign without changing its value.
𝑛 𝑛

∑ 𝑐𝑥 = 𝑐 ∑ 𝑥
𝑥=1 𝑥=1

For example:
3 3

∑ 2𝑥 = 2(1) + 2(2) + 2(3) = 2(1 + 2 + 3) = 2 ∑ 𝑥


𝑥=1 𝑥=1

1.87: Property II: Distribution of Summation over an Expression


You can distribute the summation operator over individual terms (for addition or subtraction)
𝑛 𝑛 𝑛

∑𝑥 + 𝑦 = ∑𝑥 +∑𝑦
𝑖=1 𝑖=1 𝑖=1
𝑛 𝑛 𝑛

∑𝑥 − 𝑦 = ∑𝑥 −∑𝑦
𝑖=1 𝑖=1 𝑖=1

1.88: Sums of 𝒏𝒕𝒉 powers


𝑛
𝑛(𝑛 + 1)
∑ 𝑥 = 1 + 2+. . +𝑛 =
2
𝑥=1
𝑛
𝑛(𝑛 + 1)(2𝑛 + 1)
∑ 𝑥 2 = 12 + 22 + ⋯ + 𝑛 2 =
6
𝑥=1
𝑛 2
3 3
𝑛(𝑛 + 1)
3 3
∑𝑥 = 1 + 2 + ⋯+ 𝑛 = [ ]
2
𝑥=1

Example 1.89
Suppose the area under the curve 𝑓(𝑥) = 𝑥 2 from 𝑥 = 0 to 𝑥 = 1 is divided
into 𝑛 rectangles of equal width.
𝑖2
A. Show that the area of the 𝑖 𝑡ℎ rectangle is 𝐴(𝑅𝑖 ) = 𝑛3
B. using right endpoint rectangles that the area under the curve with 𝑛
(𝑛+1)(2𝑛+1)
rectangles is 6𝑛2
1
C. Show that the limit of the expression as 𝑛 → ∞ is 3.

Part A starting from left to right:


Divide the area into 𝑛 rectangles of equal width, 𝑅1 , 𝑅2 , … , 𝑅𝑛
Get all the files at: https://bit.ly/azizhandouts

Width of each rectangle is: 𝐴𝑟𝑒𝑎 ≈ ∑ 𝐴(𝑅𝑖 )


𝐿𝑒𝑛𝑔𝑡ℎ 𝑜𝑓 𝐼𝑛𝑡𝑒𝑟𝑣𝑎𝑙 1 𝑖=1
=
𝑁𝑜. 𝑜𝑓 𝑅𝑒𝑐𝑡𝑎𝑛𝑔𝑙𝑒𝑠 𝑛
Substitute the result from Part A in the expression
The right endpoint of the rectangles is given by: above:
𝑛
1 2 3 𝑛 𝑖2
, , ,…, =∑
𝑛 𝑛 𝑛 𝑛 𝑛3
𝑖=1

Height of the 𝑖 𝑡ℎ rectangle is given by the function Use the property ∑𝑛𝑥=1 𝑐𝑥 = 𝑐 ∑𝑛𝑥=1 𝑥 to move the
evaluated at the right endpoint of the interval: 1
constant 𝑛3 out of the summation:
𝑖 𝑖 2 𝑛
ℎ(𝑅𝑖 ) = 𝑓 ( ) = ( ) 1
𝑛 𝑛
= 3 ∑ 𝑖2
𝑛
𝑖=1
The area of the 𝑖 𝑡ℎ rectangle is:
1 𝑖 2 𝑖2 Use the formula for the sum of the squares of the
𝐴(𝑅𝑖 ) = 𝑤ℎ = ( ) = 3
⏟ 𝑛 𝑛 𝑛 first 𝑛 natural numbers:
𝑬𝒒𝒖𝒂𝒕𝒊𝒐𝒏 𝑰
1 𝑛(𝑛 + 1)(2𝑛 + 1) (𝑛 + 1)(2𝑛 + 1)
= 3[ ]=
𝑛 6 6𝑛2
Part B
The total area under the curve can be
approximated by: Part C
(𝑛 + 1)(2𝑛 + 1) 2𝑛2 + 3𝑛 + 1 2 1
lim = lim = =
𝑛→∞ 6𝑛2 𝑛→∞ 6𝑛2 6 3

1.90: General Approach

Width of Interval
No. Of Rectangles
Width of each Rectangle = Width of Interval/No. of Rectangles

where x is given by the endpoint rule that you choose. Common endpoint rules include:
Left Endpoint
Right Endpoint
Midpoint

Example 1.91
Approximate under the curve for 𝑓(𝑥) = sin 𝑥 from 𝑥 = 0 to 𝑥 = 𝜋 using summation notation

𝑊𝑖𝑑𝑡ℎ 𝑜𝑓 𝐼𝑛𝑡𝑒𝑟𝑣𝑎𝑙 = 𝜋
𝑁𝑜. 𝑂𝑓 𝑅𝑒𝑐𝑡𝑎𝑛𝑔𝑙𝑒𝑠 = 𝑛
𝑊𝑖𝑑𝑡ℎ 𝑜𝑓𝐼𝑛𝑡𝑒𝑟𝑣𝑎𝑙 𝜋
𝑊𝑖𝑑𝑡ℎ 𝑜𝑓 𝑒𝑎𝑐ℎ 𝑅𝑒𝑐𝑡𝑎𝑛𝑔𝑙𝑒 = =
𝑁𝑜. 𝑜𝑓 𝑅𝑒𝑐𝑡𝑎𝑛𝑔𝑙𝑒𝑠 𝑛

Suppose we choose the right endpoint rule.


𝜋 𝜋 𝜋 2𝜋 𝜋 𝑛𝜋
(sin ) + (sin ) + ⋯ + (sin )

𝑛 ⏟ 𝑛 𝑛 𝑛 𝑛 𝑛
𝑾𝒊𝒅𝒕𝒉 𝑯𝒆𝒊𝒈𝒉𝒕
Write the above in summation notation:
Get all the files at: https://bit.ly/azizhandouts

𝑛
𝜋 𝑘𝜋
∑ (sin )

𝑛 ⏟ 𝑛
𝑘=1
𝑾𝒊𝒅𝒕𝒉 𝑯𝒆𝒊𝒈𝒉𝒕

Suppose we choose the left endpoint rule.


𝜋 𝜋 𝜋 𝜋 (𝑛 − 1)𝜋
(sin
⏟ 0) + (sin ) + ⋯ + (sin )

𝑛 𝑯𝒆𝒊𝒈𝒉𝒕 𝑛 𝑛 𝑛 𝑛
𝑾𝒊𝒅𝒕𝒉
Write the above in summation notation:
𝑛−1
𝜋 𝑘𝜋
∑ (sin )

𝑛 ⏟ 𝑛
𝑘=0
𝑾𝒊𝒅𝒕𝒉 𝑯𝒆𝒊𝒈𝒉𝒕

E. Reimann sums
We have taken earlier taken area approximations with a small number of rectangles (say 2, or 3, or 4
rectangles) and:
➢ rectangles of equal width
➢ Some method of choosing the height of the rectangle (left endpoint, right endpoint, midpoint)

We can generalize to find the area for 𝑛 rectangles. And the sum of these areas is called a Reimann Sum.

1.92: Interval
An interval is a set of real numbers that contains all real numbers lying between any two numbers of the set.

These are the intervals you have seen before, say, when solving inequalities.
𝑥 ∈ (2,4) ⇒ 𝐴𝑙𝑙 𝑛𝑢𝑚𝑏𝑒𝑟𝑠 𝑓𝑟𝑜𝑚 2 𝑡𝑜 4, 𝑏𝑢𝑡 𝑛𝑜𝑡 𝑖𝑛𝑐𝑙𝑢𝑑𝑖𝑛𝑔 2 𝑎𝑛𝑑 4

𝑶𝒑𝒆𝒏 𝑰𝒏𝒕𝒆𝒓𝒗𝒂𝒍
𝑥 ∈ [2,4]
⏟ ⇒ 𝐴𝑙𝑙 𝑛𝑢𝑚𝑏𝑒𝑟𝑠 𝑓𝑟𝑜𝑚 2 𝑡𝑜 4, 𝑖𝑛𝑐𝑙𝑢𝑑𝑖𝑛𝑔 2 𝑎𝑛𝑑 4
𝑪𝒍𝒐𝒔𝒆𝒅 𝑰𝒏𝒕𝒆𝒓𝒗𝒂𝒍

1.93: Partition
A partition {𝑥0 , 𝑥1 , 𝑥2 , … , 𝑥𝑛 } such that:
𝑎 = 𝑥0 < 𝑥1 < 𝑥2 < ⋯ < 𝑥𝑛−1 < 𝑥𝑛 = 𝑏
divides an interval [𝑎, 𝑏] into 𝑛 closed subintervals
[𝑥0 , 𝑥1 ], [𝑥1 , 𝑥2 ], … , [𝑥𝑛−1 , 𝑥𝑛 ]

➢ It is not necessary that the width of each subinterval is equal.

1.94: Rectangles from Partitions


The partition can be used to generate rectangles.
The width of the 𝑘 𝑡ℎ rectangle is given by:
𝑥𝑘 − 𝑥𝑘−1 = Δ𝑥𝑘

Width
The width of each rectangle in the expression above is determined by the partition. Any choice of widths
results in a valid partition. Width can be equal but are not required to be equal.

Height
Height is given by evaluating the function 𝑓 at 𝑐𝑘 .
𝑐𝑘 is any point in the interval [𝑥𝑘−1 , 𝑥𝑘 ]
Get all the files at: https://bit.ly/azizhandouts

1.95: Definition
The Reimann sum for the function 𝑓(𝑥) from 𝑥 = 𝑎 to 𝑥 = 𝑏 is given by the sum of the area of the rectangles
under the curve:
𝑛 𝑛

∑ 𝐴(𝑅𝑘 ) = ∑ ⏟
𝑓(𝑐𝑘 ) Δ𝑥
⏟𝑘
𝑘=1 𝑖=1 𝑯𝒆𝒊𝒈𝒉𝒕 𝑾𝒊𝒅𝒕𝒉

1.96: Norm of a Partition


The norm of a partition is the interval in the partition with the largest width.

Notation: The norm of 𝑃 is written:


‖𝑃‖

Example 1.97
Determine the norm of the partition in each base.
A. A partition over [0,1] is given by 𝑃 = {0,0.2,0.6,0.7,0.8,0.9,1}.

Part A
1 − 0.9 = 0.1
0.9 − 0.8 = 0.1
0.8 − 0.7 = 0.1
0.7 − 0.6 = 0.1
0.6 − 0.2 = 0.4
0.2 − 0 = 0.2
𝑀𝑎𝑥{0.1,0.2,0.4} = 0.4

‖𝑃‖ = 0.4

1.98: Equal Width Intervals


If the intervals have equal width, then the width of each interval
𝐿𝑒𝑛𝑔𝑡ℎ 𝑜𝑓 𝐼𝑛𝑡𝑒𝑟𝑣𝑎𝑙
=
𝑁𝑜. 𝑜𝑓 𝑅𝑒𝑐𝑡𝑎𝑛𝑔𝑙𝑒𝑠

Example 1.99
Find and simplify an expression for the Reimann sum of the area under the curve, using right endpoints, for
the function 𝑓(𝑥) = 𝑥 + 2 from 𝑥 = 1 to 𝑥 = 4 and find the value of that expression as the number of
rectangles approaches infinity.
Get all the files at: https://bit.ly/azizhandouts

Area using Geometry


The shape of the area under the curve is a trapezoid:
3+6 9 27
=3∙ =3∙ = = 13.5
2 2 2
Rectangle Width and Height
𝐿𝑒𝑛𝑔𝑡ℎ 𝑜𝑓 𝐼𝑛𝑡𝑒𝑟𝑣𝑎𝑙 4 − 1 3
𝑊𝑖𝑑𝑡ℎ 𝑜𝑓 𝑒𝑎𝑐ℎ 𝑅𝑒𝑐𝑡𝑎𝑛𝑔𝑙𝑒 = = =
𝑁𝑜. 𝑜𝑓 𝑅𝑒𝑐𝑡𝑎𝑛𝑔𝑙𝑒𝑠 𝑛 𝑛
The right endpoint of the rectangles is:
3 6 9 3𝑛
1 + , 1 + , 1 + , … ,1 +
𝑛 𝑛 𝑛 𝑛
The right endpoint of the 𝑖 𝑡ℎ rectangle:
3𝑖
𝑐𝑖 = 1 +
𝑛
3𝑖 3𝑖
𝐻𝑒𝑖𝑔ℎ𝑡 𝑜𝑓 𝑡ℎ𝑒 𝑖 𝑡ℎ 𝑅𝑒𝑐𝑡𝑎𝑛𝑔𝑙𝑒 = 𝑓(𝑐𝑖 ) = 1 +
+2=3+
𝑛 𝑛
𝑡ℎ
3𝑖 3 9 9𝑖 1 𝑖
𝐴𝑟𝑒𝑎 𝑜𝑓 𝑡ℎ𝑒 𝑖 𝑟𝑒𝑐𝑡𝑎𝑛𝑔𝑙𝑒 = 𝐴(𝑅𝑖 ) = (3 + ) ( ) = + 2 = 9 ( + 2 )
⏟ 𝑛 ⏟ 𝑛 𝑛 𝑛 𝑛 𝑛
𝐻𝑒𝑖𝑔ℎ𝑡 𝑊𝑖𝑑𝑡ℎ
Sum of the areas of all Rectangles
𝑛 𝑛 𝑛
1 𝑖 1 𝑖
∑ 𝐴(𝑅𝑖 ) = ∑ 9 ( + 2 ) = 9 ∑ + 2
𝑛 𝑛 𝑛 𝑛
𝑖=1 𝑖=1 ⏟𝑖=1
𝑪𝒐𝒏𝒔𝒕𝒂𝒏𝒕
𝑴𝒖𝒍𝒕𝒊𝒑𝒍𝒆 𝑹𝒖𝒍𝒆
Split the summation:
𝑛 𝑛 𝑛
1 𝑖 𝑛 1 1 𝑛(𝑛 + 1) 𝑛2 + 𝑛
9 (∑ + ∑ 2 ) = 9 ( + 2 ∑ 𝑖 ) = 9 (1 + 2 ( )) = 9 (1 + )
𝑛 𝑛 𝑛 𝑛 𝑛 2 2𝑛2
𝑖=1 𝑖=1 𝑖=1

Find the limit of the expression as the number of rectangles approaches infinity
𝑛2 +𝑛
Move 9 out of lim 9 (1 + ) using the constant multiple rule:
𝑛→∞ 2𝑛2
𝑛2 + 𝑛
= 9 lim 1 +
𝑛→∞ 2𝑛2
Divide numerator and denominator by 𝑛2 :
𝑛2 𝑛 1
𝑛 2 + 𝑛2 1 +𝑛
= 9 ( lim 1 + ) = 9 ( lim 1 + )
𝑛→∞ 2𝑛2 𝑛→∞ 2
𝑛2
1
Note that as 𝑛 → ∞, → 0:
𝑛
1 +0 3 3 27
= 9 ( lim 1 + ) = 9 ( lim ) = 9 ∙ =
𝑛→∞ 2 𝑛→∞ 2 2 2

Example 1.100
Evaluate the Reimann sum using left endpoints for 𝑓(𝑥) = 𝑥 + 2 from 𝑥 = 1 to 𝑥 = 4 as the number of
rectangles approaches infinity.
Get all the files at: https://bit.ly/azizhandouts

Area using Geometry


The shape of the area under the curve is a trapezoid:
3+6 9 27
=3∙ =3∙ = = 13.5
2 2 2
Rectangle Width and Height
𝐿𝑒𝑛𝑔𝑡ℎ 𝑜𝑓 𝐼𝑛𝑡𝑒𝑟𝑣𝑎𝑙 4 − 1 3
𝑊𝑖𝑑𝑡ℎ 𝑜𝑓 𝑒𝑎𝑐ℎ 𝑅𝑒𝑐𝑡𝑎𝑛𝑔𝑙𝑒 = = =
𝑁𝑜. 𝑜𝑓 𝑅𝑒𝑐𝑡𝑎𝑛𝑔𝑙𝑒𝑠 𝑛 𝑛
The left endpoint of the rectangles is:
3 6 3(𝑛 − 1)
1, 1 + , 1 + , … ,1 +
𝑛 𝑛 𝑛
The left endpoint of the 𝑖 𝑡ℎ rectangle:
3(𝑖 − 1)
𝑐𝑖 = 1 +
𝑛
𝑡ℎ
3(𝑖 − 1) 3(𝑖 − 1) 3𝑖 − 3
𝐻𝑒𝑖𝑔ℎ𝑡 𝑜𝑓 𝑡ℎ𝑒 𝑖 𝑅𝑒𝑐𝑡𝑎𝑛𝑔𝑙𝑒 = 𝑓(𝑐𝑖 ) = 1 + +2=3+ =3+
𝑛 𝑛 𝑛
3𝑖 − 3 3 9 9𝑖 − 9 1 𝑖 − 1
𝐴𝑟𝑒𝑎 𝑜𝑓 𝑡ℎ𝑒 𝑖 𝑡ℎ 𝑟𝑒𝑐𝑡𝑎𝑛𝑔𝑙𝑒 = 𝐴(𝑅𝑖 ) = (3 + ) ( ) = + 2
= 9( + 2 )
⏟ 𝑛 ⏟𝑛 𝑛 𝑛 𝑛 𝑛
𝐻𝑒𝑖𝑔ℎ𝑡 𝑊𝑖𝑑𝑡ℎ
Sum of the areas of all Rectangles
𝑛 𝑛 𝑛
1 𝑖−1 1 𝑖−1
∑ 𝐴(𝑅𝑖 ) = ∑ 9 ( + 2 ) = 9 ∑ + 2
𝑛 𝑛 ⏟ 𝑛 𝑛
𝑖=1 𝑖=1 𝑖=1
𝑪𝒐𝒏𝒔𝒕𝒂𝒏𝒕
𝑴𝒖𝒍𝒕𝒊𝒑𝒍𝒆 𝑹𝒖𝒍𝒆
Split the summation:
𝑛 𝑛 𝑛
1 𝑖−1 𝑛 1 1 𝑛(𝑛 − 1) 𝑛2 − 𝑛
9 (∑ + ∑ 2 ) = 9 ( + 2 ∑ 𝑖 − 1) = 9 (1 + 2 ( )) = 9 (1 + )
𝑛 𝑛 𝑛 𝑛 𝑛 2 2𝑛2
𝑖=1 𝑖=1 𝑖=1

Find the limit of the expression as the number of rectangles approaches infinity
𝑛2 −𝑛
Move 9 out of lim 9 (1 + 2𝑛2
) using the constant multiple rule:
𝑛→∞
𝑛2 − 𝑛
= 9 lim 1 +
𝑛→∞ 2𝑛2
Divide numerator and denominator by 𝑛2 :
𝑛2 𝑛 1
𝑛 2 − 𝑛2 1−𝑛
= 9 ( lim 1 + ) = 9 ( lim 1 + )
𝑛→∞ 2𝑛2 𝑛→∞ 2
𝑛2
1
Note that as 𝑛 → ∞, 𝑛 → 0:
1−0 3 3 27
= 9 ( lim 1 + ) = 9 ( lim ) = 9 ∙ =
𝑛→∞ 2 𝑛→∞ 2 2 2

Example 1.101
Find and simplify an expression for the Reimann sum of the area under the curve, using right endpoints, for
the function 𝑓(𝑥) = 1 − 𝑥 2 from 𝑥 = 0 to 𝑥 = 1 and find the value of that expression as the number of
rectangles approaches infinity.

Width of each rectangle


𝐿𝑒𝑛𝑔𝑡ℎ 𝑜𝑓 𝐼𝑛𝑡𝑒𝑟𝑣𝑎𝑙 1 − 0 1
= = =
𝑁𝑜. 𝑜𝑓 𝑅𝑒𝑐𝑡𝑎𝑛𝑔𝑙𝑒𝑠 𝑛 𝑛
Find the Right Endpoint of each rectangle
Get all the files at: https://bit.ly/azizhandouts

The right endpoint of the rectangles is given by:


1 2 3 𝑛
, , ,…,
𝑛 𝑛 𝑛 𝑛
The right endpoint of the 𝑖 𝑡ℎ rectangle:
𝑖
𝑐𝑖 =
𝑛
Find the Height of each rectangle
The height of the 𝑖 𝑡ℎ rectangle:
𝑓(𝑐𝑖 ) = 1 − 𝑐𝑖2
Find the Area of each rectangle
𝑖 1 𝑐𝑖2
𝐴(𝑅𝑖 ) = (1 − 𝑐𝑖2 )
⏟ ( ) = −
⏟𝑛 𝑛 𝑛
𝐻𝑒𝑖𝑔ℎ𝑡
𝑊𝑖𝑑𝑡ℎ
Find the sum of the areas of all Rectangles
𝑛 𝑛
1 𝑐𝑖2
= ∑ 𝐴(𝑅𝑖 ) = ∑ −
𝑛 𝑛
𝑖=1 𝑖=1
Use the difference property of summation (∑𝑛𝑖=1 𝑥 − 𝑦 = ∑𝑛𝑖=1 𝑥 − ∑𝑛𝑖=1 𝑦) to split the expression:
𝑛 𝑛
1 𝑐𝑖2
= (∑ ) − (∑ )
𝑛 𝑛
𝑖=1 𝑖=1
1
Use the constant multiple property ∑𝑛𝑥=1 𝑐𝑥 = 𝑐 ∑𝑛𝑥=1 𝑥 to move 𝑛 out of the summation sign in the first term:
𝑛 𝑛
1 𝑐𝑖2
= ( ∑ 1) − (∑ )
𝑛 𝑛
𝑖=1 𝑖=1
Use the property ∑𝑘𝑛=1 𝑥 = 𝑘𝑥 to sum the first term:
𝑛 𝑛
1 𝑐𝑖2 𝑐𝑖2
=( (𝑛)) − (∑ ) = 1 − (∑ )
𝑛 𝑛 𝑛
𝑖=1 𝑖=1
𝑖
Substitute 𝑐𝑖 = 𝑛
:
𝑖 2 𝑛 𝑛 𝑖
2
𝑛 2
(𝑛) 2 𝑖
=1−∑ =1−∑ 𝑛 =1−∑ 3
𝑛 𝑛 𝑛
𝑖=1 𝑖=1 𝑖=1
1
Use the constant multiple property ∑𝑛𝑥=1 𝑐𝑥 = 𝑐 ∑𝑛𝑥=1 𝑥 to move 𝑛3 out of the summation sign:
𝑛
1
= 1 − 3 ∑ 𝑖2
𝑛
𝑖=1
𝑛(𝑛+1)(2𝑛+1)
Use the formula for the sum of the squares of the first n natural numbers ∑𝑛𝑥=1 𝑥 2 = 6
:
1 𝑛(𝑛 + 1)(2𝑛 + 1) (𝑛 + 1)(2𝑛 + 1) 2𝑛2 + 3𝑛 + 1
=1− ( ) = 1 − == 1 −
𝑛3 6 6𝑛2 6𝑛2
Find the limit of the expression as the number of rectangles approaches infinity
2𝑛2 + 3𝑛 + 1
lim 1 −
𝑛→∞ 6𝑛2
Divide numerator and denominator in the fraction by the highest power of 𝑛 in the denominator, which is 𝑛2 :
2𝑛2 3𝑛 1 3 1
𝑛 2 + 𝑛2 + 𝑛2 2+𝑛+ 2
𝑛 =1−2+0+0=1−2=1−1=2
lim 1 − 2 = lim 1 −
𝑛→∞ 6𝑛 𝑛→∞ 6 6 6 3 3
𝑛2
Get all the files at: https://bit.ly/azizhandouts

1.4 Definite Integral: Reimann Sum Definition


A. Definition
The definite integral is one of the most powerful tools of Calculus. It lets us calculate the area under the curve
of a function. However, in this section we are not going to focus on calculations, but on the concept, which is
the definition of the definite integral.

1.102: Integral as a Limit of a Reimann Sum


Let 𝑓 be a function defined on a closed interval [𝑎, 𝑏]. 𝐼 is the definite integral over [𝑎, 𝑏] if the limit of the
Reimann sum below exists:
𝑏 𝑛

∫ 𝑓(𝑥) 𝑑𝑥 = lim ∑ 𝑓(𝑐𝑘 )Δ𝑥𝑘


𝑎 ‖𝑃‖→0
𝑘=1

Notation for Definite Integral


𝑎 = 𝐿𝑒𝑓𝑡 𝑒𝑛𝑑𝑝𝑜𝑖𝑛𝑡 𝑜𝑓 𝐼𝑛𝑡𝑒𝑟𝑣𝑎𝑙 = 𝐿𝑜𝑤𝑒𝑟 𝑙𝑖𝑚𝑖𝑡 𝑜𝑓 𝐼𝑛𝑡𝑒𝑔𝑟𝑎𝑡𝑖𝑜𝑛
𝑎 = 𝑅𝑖𝑔ℎ𝑡 𝑒𝑛𝑑𝑝𝑜𝑖𝑛𝑡 𝑜𝑓 𝐼𝑛𝑡𝑒𝑟𝑣𝑎𝑙 = 𝑈𝑝𝑝𝑒𝑟 𝑙𝑖𝑚𝑖𝑡 𝑜𝑓 𝐼𝑛𝑡𝑒𝑔𝑟𝑎𝑡𝑖𝑜𝑛
𝑓(𝑥) = 𝐹𝑢𝑛𝑐𝑡𝑖𝑜𝑛 𝑡𝑜 𝑏𝑒 𝐼𝑛𝑡𝑒𝑔𝑟𝑎𝑡𝑒𝑑 = 𝐼𝑛𝑡𝑒𝑔𝑟𝑎𝑛𝑑
𝑑𝑥 = 𝑉𝑎𝑟𝑖𝑎𝑏𝑙𝑒 𝑤𝑖𝑡ℎ 𝑟𝑒𝑠𝑝𝑒𝑐𝑡 𝑡𝑜 𝑤ℎ𝑖𝑐ℎ 𝑖𝑛𝑡𝑒𝑔𝑟𝑎𝑡𝑖𝑜𝑛 𝑖𝑠 𝑏𝑒𝑖𝑛𝑔 𝑐𝑎𝑟𝑟𝑖𝑒𝑑

Integral as a limit of a Reimann Sum


Since it is a Reimann sum, we can choose:
➢ the partition. That is, we can choose the widths of the rectangles.
➢ The point within the partition (𝑐𝑘 ). This determines 𝑓(𝑐𝑘 )

The limit should exist (and be same) independent of the choices made above.

To understand how a Reimann sum works, we take an example and work through it. The parts of the
definition above are reproduced below.

Let 𝑓 be a function defined on a closed interval [𝑎, 𝑏].


For example, consider the function 𝑓(𝑥) = 𝑥 2 . It is defined over the interval [1,2]. The diagram on the right
shows the graph of the function.

Consider the partition formed by dividing the interval from [𝑎, 𝑏] into 𝑛 parts. Then, find 𝑓(𝑥) for any 𝑥 within
each sub-interval, and that creates 𝑛 rectangles.
For example, the diagrams below show the interval divided into 2 and 5 rectangles respectively.
Get all the files at: https://bit.ly/azizhandouts

The Reimann sum gives an approximation of the area under the curve for this function over the interval [𝑎, 𝑏].
𝑛

𝑅𝑒𝑖𝑚𝑎𝑛𝑛 𝑆𝑢𝑚 = ∑ 𝑓(𝑐𝑘 )Δ𝑥𝑘


𝑘=1

As the number of rectangles increases the approximation becomes better. If the limit of the Reimann sum as
the norm of the partition goes to zero exists, then that is the definition of the definite integral
𝑛

𝑓(𝑐𝑘 ) Δ𝑥
lim ∑ ⏟ ⏟𝑘
‖𝑃‖→0
𝑘=1 𝑯𝒆𝒊𝒈𝒉𝒕 𝑾𝒊𝒅𝒕𝒉

Example 1.103: Converting from Reimann Sums to Integrals


Convert each Reimann sum given below into a definite integral
A. Find the definite integral associated with lim ∑𝑛𝑘=1 𝑐𝑘2 Δ𝑥𝑘 over a partition 𝑃 from 𝑥 = 0 to 𝑥 = 2
‖𝑃‖→0
B. Find the definite integral associated with lim ∑𝑛𝑘=1(𝑐𝑘 + 1)Δ𝑥𝑘 over a partition 𝑃 from 𝑥 = 3 to 𝑥 =
‖𝑃‖→0
7.

Part A Note that we are integrating with respect to 𝑥, and


We need to write lim ∑𝑛𝑘=1 𝑓(𝑐𝑘2 )Δ𝑥𝑘 in the form hence we need a 𝑑𝑥 on the right side of the definite
‖𝑃‖→0
𝑏 integral. Combine everything:
∫𝑎 𝑓(𝑥) 𝑑𝑥. The lower and upper limits of 2
integration are the endpoints of the Partition: ∫ 𝑥 2 𝑑𝑥
0
𝑎
⏟= 0 , 𝑏
⏟= 2 Part B
𝑳𝒆𝒇𝒕 𝒆𝒏𝒅𝒑𝒐𝒊𝒏𝒕 𝑹𝒊𝒈𝒉𝒕 𝑬𝒏𝒅𝒑𝒐𝒊𝒏𝒕 7
𝑳𝒐𝒘𝒆𝒓 𝒍𝒊𝒎𝒊𝒕 𝑼𝒑𝒑𝒆𝒓 𝑳𝒊𝒎𝒊𝒕
∫ (𝑥 + 1) 𝑑𝑥
Identify the height and the width of the rectangles: 3
𝑛

lim ∑ 𝑐⏟𝑘2 ⏟𝑘 ⇒ 𝑓(𝑐𝑘 ) = 𝑐𝑘2 ⇒ 𝑓(𝑥) = 𝑥 2


Δ𝑥
‖𝑃‖→0
𝑘=1 𝑯𝒆𝒊𝒈𝒉𝒕 𝑾𝒊𝒅𝒕𝒉

Example 1.104: Checking for existence of Definite Integrals


For each expression, state whether it corresponds to a definite integral. If it does, state it. If it doesn’t, explain
why not
𝜋 𝜋
A. lim ∑𝑛𝑘=1(csc 𝑐𝑘 )Δ𝑥𝑘 over a partition 𝑃 from 𝑥 = − 2 to 𝑥 = 2
‖𝑃‖→0
𝜋 𝜋
B. lim ∑𝑛𝑘=1(sin 𝑐𝑘 )Δ𝑥𝑘 over a partition 𝑃 from 𝑥 = − 2 to 𝑥 = 2
‖𝑃‖→0

Part A
Get all the files at: https://bit.ly/azizhandouts

The function in the limit expression is:


1
𝑓(𝑥) = csc 𝑥 =
⇒ 0 ∉ 𝐷𝑓
sin 𝑥
And since sin 0 = 0, the number 0 is not in the domain of 𝑓.

𝜋 𝜋
Hence, the expression does not meet the condition that the function is defined over the interval [− 2 , 2 ].
Hence, the definite integral does not exist.

Part B
𝜋
2
∫ sin 𝑥 𝑑𝑥
𝜋

2

Example 1.105: Converting from Integrals to Reimann Sums


7
A. ∫−5 (𝑥 2 + 5𝑥 + 6)𝑑𝑥
𝜋
B. ∫𝜋2 sin2 𝑥 2 𝑑𝑥
4

Part A
7 𝑛
(𝑥 2 2
∫ + 5𝑥 + 6)𝑑𝑥 = lim ∑ (𝑐
‖𝑃‖→0
⏟𝑘 𝑜𝑣𝑒𝑟 𝑎 𝑝𝑎𝑟𝑡𝑖𝑡𝑖𝑜𝑛 𝑜𝑓 𝑡ℎ𝑒 𝑖𝑛𝑡𝑒𝑟𝑣𝑎𝑙 [−5,7]
⏟ 𝑘 + 5𝑐𝑘 + 6) Δ𝑥
−5 𝑘=1 𝑯𝒆𝒊𝒈𝒉𝒕 𝑾𝒊𝒅𝒕𝒉
Part B
𝜋 𝑛
2
2 2 2 2
𝜋 𝜋
∫ sin 𝑥 𝑑𝑥 = lim ∑ sin
⏟ 𝑐𝑘 Δ𝑥⏟𝑘 𝑜𝑣𝑒𝑟 𝑎 𝑝𝑎𝑟𝑡𝑖𝑡𝑖𝑜𝑛 𝑜𝑓 𝑖𝑛𝑡𝑒𝑟𝑣𝑎𝑙 [ , ]
𝜋 ‖𝑃‖→0 4 2
4 𝑘=1 𝑯𝒆𝒊𝒈𝒉𝒕 𝑾𝒊𝒅𝒕𝒉

1.106: Integrals as Area under the Curve


Since a definite integral is the limit of a Reimann sum as the norm of the partition tends to zero, and the
Reimann sum gives you the area under the curve, evaluating the definite integral will give you the area under
the curve.

2 𝑛

∫ 𝑥 𝑑𝑥 2
= lim ∑ 𝑐𝑘2 Δ𝑥𝑘 over a partition 𝑃[0,2] = 𝐴𝑟𝑒𝑎 𝑢𝑛𝑑𝑒𝑟 𝑡ℎ𝑒 𝐶𝑢𝑟𝑣𝑒 𝑜𝑓 𝑥 2 𝑓𝑜𝑟 [0,2]
⏟0 ‖𝑃‖→0
⏟ 𝑘=1
𝑫𝒆𝒇𝒊𝒏𝒊𝒕𝒆 𝑰𝒏𝒕𝒆𝒈𝒓𝒂𝒍 𝑹𝒆𝒊𝒎𝒂𝒏𝒏 𝑺𝒖𝒎

Example 1.107
Find the area between the line 𝑦 = 𝑥, 𝑥 = 0, 𝑥 = 3 and the 𝑥-axis:
A. By using geometry
B. By setting up a definite integral, and applying the result from Part A.
Get all the files at: https://bit.ly/azizhandouts

Part A
The shape that we want the area of is a triangle with:
1 1 9
𝐵𝑎𝑠𝑒 = 𝐻𝑒𝑖𝑔ℎ𝑡 = 3 ⇒ 𝐴𝑟𝑒𝑎 = 𝑏ℎ = (3)(3) =
2 2 2
Part B
𝐹𝑢𝑛𝑐𝑡𝑖𝑜𝑛 = 𝑓(𝑥) = 𝑥
𝐿𝑒𝑓𝑡 𝑒𝑛𝑑𝑝𝑜𝑖𝑛𝑡 = 𝐿𝑜𝑤𝑒𝑟 𝐿𝑖𝑚𝑖𝑡 𝑜𝑓 𝐼𝑛𝑡𝑒𝑔𝑟𝑎𝑡𝑖𝑜𝑛 = 𝑎 = 0
𝑅𝑖𝑔ℎ𝑡 𝑒𝑛𝑑𝑝𝑜𝑖𝑛𝑡 = 𝑈𝑝𝑝𝑒𝑟 𝐿𝑖𝑚𝑖𝑡 𝑜𝑓 𝐼𝑛𝑡𝑒𝑔𝑟𝑎𝑡𝑖𝑜𝑛 = 𝑏 = 0
Combine the above, and substitute:
𝑏 3
9
∫ 𝑓(𝑥) 𝑑𝑥 = ∫ 𝑥 𝑑𝑥 =
𝑎 0 2

B. Integrability vs. Continuity


The continuity of a function is key in determining whether it is integrable.

1.108: Continuous Functions


If a function is continuous over the interval [𝑎, 𝑏] then it is integrable over that interval.

MCMC 1.109
𝑀𝑎𝑟𝑘 𝑎𝑙𝑙 𝑐𝑜𝑟𝑟𝑒𝑐𝑡 𝑜𝑝𝑡𝑖𝑜𝑛𝑠
A 𝑓(𝑥) has domain ℝ − {3}. Then it must not be integrable over:
A. the open interval (2,3)
B. the closed interval[2,3]
C. the half open interval [2,3)
D. the half open interval (2,3]

𝑂𝑝𝑡𝑖𝑜𝑛 𝐴, 𝐵, 𝐶 𝑎𝑛𝑑 𝐷

Example 1.110
1
A. Is 𝑓(𝑥) = 𝑥 integrable over −3 to 0?
1
B. Is 𝑓(𝑥) = 𝑥 integrable over −3 to −1?

31
The questions asks whether ∫0 𝑥 𝑑𝑥 is defined.
1
For 𝑥 to be integrable over −3 to 0, it must be continuous over [−3,0].

1
𝑥
is continuous over the open interval (−∞, 0), but not over the closed interval [−∞, 0].
1
Hence, the function 𝑥 is not integrable over −3 to 0.

1.111: Infinite Discontinuity


If a function has an infinite discontinuity in an interval, then the integral does not exist over that interval.2
If 𝑓(𝑥) has an infinite discontinuity at 𝑐 ∈ [𝑎, 𝑏] then
𝑏
∫ 𝑓(𝑥) 𝑑𝑥 𝑖𝑠 𝑛𝑜𝑡 𝑑𝑒𝑓𝑖𝑛𝑒𝑑
𝑎

2
While the integral does not exist if c is within the interval, we can consider c to the right of the interval and calculate the limit
as 𝑏 approaches 𝑐 from the left. This technique which gives an expression for the area combines the concept of limits and
integrals. Such integrals are called improper integrals.
Get all the files at: https://bit.ly/azizhandouts

Example 1.112
1
𝑑𝑥
The value of the integral ∫ is (𝑱𝑴𝑬𝑻 𝟐𝟎𝟏𝟏/𝟕𝟗)
𝑥2
−1

When 𝑥 = 0:
1 1
= ⇒ 𝑁𝑜𝑡 𝐷𝑒𝑓𝑖𝑛𝑒𝑑
𝑥2 0
1
Hence, the function 𝑥 2 has an infinite discontinuity at 𝑥 = 0. Hence,
1
𝑑𝑥
∫ 𝑑𝑜𝑒𝑠 𝑛𝑜𝑡 𝑒𝑥𝑖𝑠𝑡
𝑥2
−1

MCQ 1.113
𝑀𝑎𝑟𝑘 𝑡ℎ𝑒 𝑐𝑜𝑟𝑟𝑒𝑐𝑡 𝑜𝑝𝑡𝑖𝑜𝑛
𝜋 𝜋
Statement: 𝑓(𝑥) = sec 𝑥 is not integrable over the interval − 2 to 2 .
𝜋 𝜋
Reason: cos 𝑥 has a zero at 𝑥 = − and 𝑥 = .
2 2

A. Statement is true, and the reason explains why it is true.


B. Statement is true, but the reason does not explain why it is true.
C. Statement is true, but reason is false.
D. Statement is false, reason is a true statement by itself.
E. Statement and reason are both false.

𝑂𝑝𝑡𝑖𝑜𝑛 𝐴

Example 1.114
𝑀𝑎𝑟𝑘 𝑡ℎ𝑒 𝑐𝑜𝑟𝑟𝑒𝑐𝑡 𝑜𝑝𝑡𝑖𝑜𝑛
𝜋 𝜋
Statement: 𝑓(𝑥) = sin 𝑥 is not integrable over − 2 to 2 .
Reason: sin 𝑥 has a zero at 𝑥 = 0.

A. Statement is true, and the reason explains why it is true.


B. Statement is true, but the reason does not explain why it is true.
C. Statement is true, but reason is false.
D. Statement is false, reason is a true statement by itself.
E. Statement and reason are both false.
Get all the files at: https://bit.ly/azizhandouts

𝑂𝑝𝑡𝑖𝑜𝑛 𝐷

1.115: Jump Discontinuity


If a function 𝑓(𝑥) has a finite number of jump discontinuities over an interval [𝑎, 𝑏], then it is integrable over
that interval.

Example 1.116
Is the function 𝑦 = ⌊𝑥⌋, ⌊𝑥⌋ represents the floor function integrable over the [𝑎, 𝑏]?

Over a finite interval, the floor function has a finite number of jump
discontinuities.
Hence, it is integrable.

Example 1.117
1, 𝑥 𝑖𝑠 𝑟𝑎𝑡𝑖𝑜𝑛𝑎𝑙
Is the function 𝑓(𝑥) = { integrable over the interval [0,1].
0, 𝑥 𝑖𝑠 𝑖𝑟𝑟𝑎𝑡𝑖𝑜𝑛𝑎𝑙

The function oscillates between 0 and 1.


Every time it oscillates there is a jump discontinuity.

There are an infinite number of rational numbers in the interval [0,1], leading to an infinite number of jump
discontinuities.

Hence, the function is not integrable.

1.5 Definite Integrals (FTC) and Areas


A. Signed Area

1.118: Signed Area


By convention, areas between a function and the 𝑥-axis
➢ above the 𝑥-axis (from left to right) will be positive
➢ above the 𝑥-axis (from right to left) will be negative
➢ below the 𝑥-axis (from left to right) will be negative
➢ below the 𝑥-axis (from right to left) will be positive

Example 1.119
State whether each expression below is positive, negative or zero for
the function 𝑓(𝑥) graphed alongside.
Get all the files at: https://bit.ly/azizhandouts

2
A. ∫1 𝑓(𝑥) 𝑑𝑥
1
B. ∫2 𝑓(𝑥) 𝑑𝑥
1
C. ∫0 𝑓(𝑥) 𝑑𝑥
−1
D. ∫0 𝑓(𝑥) 𝑑𝑥

2
∫ 𝑓(𝑥) 𝑑𝑥 ⇒ 𝐿𝑒𝑓𝑡 𝑡𝑜 𝑟𝑖𝑔ℎ𝑡, 𝐴𝑏𝑜𝑣𝑒 𝑡ℎ𝑒 𝑥 − 𝑎𝑥𝑖𝑠 ⇒ +𝑣𝑒
1
1
∫ 𝑓(𝑥) 𝑑𝑥 ⇒ 𝑅𝑖𝑔ℎ𝑡 𝑡𝑜 𝑙𝑒𝑓𝑡, 𝐴𝑏𝑜𝑣𝑒 𝑡ℎ𝑒 𝑥 − 𝑎𝑥𝑖𝑠 ⇒ −𝑣𝑒
2
1
∫ 𝑓(𝑥) 𝑑𝑥 ⇒ 𝐿𝑒𝑓𝑡 𝑡𝑜 𝑟𝑖𝑔ℎ𝑡, 𝐵𝑒𝑙𝑜𝑤 𝑡ℎ𝑒 𝑥 − 𝑎𝑥𝑖𝑠 ⇒ −𝑣𝑒
0
−1
∫ 𝑓(𝑥) 𝑑𝑥 ⇒ 𝑅𝑖𝑔ℎ𝑡 𝑡𝑜 𝐿𝑒𝑓𝑡, 𝐵𝑒𝑙𝑜𝑤 𝑡ℎ𝑒 𝑥 − 𝑎𝑥𝑖𝑠 ⇒ +𝑣𝑒
0

1.120: Unsigned Area


Unsigned area is found by taking the absolute value of the area found by dividing the integral into parts such
that the signed area is either completely positive or completely negative.

➢ Unsigned area is the area that we consider in geometry.

1.121: Order of Integration


Interchanging the limits of integration keeps the magnitude same, but changes the sign of the integral:
𝑏 𝑎
∫ 𝑓(𝑥) 𝑑𝑥 = − ∫ 𝑓(𝑥) 𝑑𝑥
𝑎 𝑏

➢ We consider area from left to right as positive, and area from right to left as negative.
𝑏
∫ 𝑓(𝑥) 𝑑𝑥 = 𝐹(𝑏) − 𝐹(𝑎)
𝑎
𝑎 𝑏
∫ 𝑓(𝑥) 𝑑𝑥 = 𝐹(𝑎) − 𝐹(𝑏) = −(𝐹(𝑏) − 𝐹(𝑎)) = − ∫ 𝑓(𝑥) 𝑑𝑥
𝑏 𝑎

Example 1.122
𝑏 𝑎
A. If ∫𝑎 𝑓(𝑥) 𝑑𝑥 = 4, then find ∫𝑏 𝑓(𝑥) 𝑑𝑥.
𝜋 𝜋
B. ∫𝜋2 cos 𝑥 𝑑𝑥 + ∫𝜋4 cos 𝑥 𝑑𝑥
4 2

Part A
𝑎 𝑏
∫ 𝑓(𝑥) 𝑑𝑥 = − ∫ 𝑓(𝑥) 𝑑𝑥 = −4
𝑏 𝑎
Part B
𝑏 𝑎
Using the property ∫𝑎 𝑓(𝑥) 𝑑𝑥 = − ∫𝑏 𝑓(𝑥) 𝑑𝑥:
𝜋 𝜋 𝜋 𝜋
2 4 2 2
∫ cos 𝑥 𝑑𝑥 + ∫ cos 𝑥 𝑑𝑥 = ∫ cos 𝑥 𝑑𝑥 − ∫ cos 𝑥 𝑑𝑥 = 0
𝜋 𝜋 𝜋 𝜋
4 2 4 4
Get all the files at: https://bit.ly/azizhandouts

This can also be solved without the property, with much more work.3

B. Fundamental Theorem of Calculus

1.123: Fundamental Theorem of Calculus (FTC)


𝑏
𝑥=𝑏
∫ 𝑓(𝑥) 𝑑𝑥 = [𝐹(𝑥)]𝑥=𝑎 = 𝐹(𝑏) − 𝐹(𝑎)
𝑎

➢ 𝑎 and 𝑏 are the limits of integration


✓ 𝑎 is the lower limit of integration
✓ 𝑏 is the upper limit of integration
➢ 𝐹(𝑥) is an antiderivative of 𝑓(𝑥)

Example 1.124
3
A. ∫0 𝑥 𝑑𝑥
7
B. ∫3 𝑥 𝑑𝑥

3
∫ 𝑥 𝑑𝑥
0
𝑥2
𝑓(𝑥) = 𝑥 ⇒ 𝐹(𝑥) = +𝐶
2
Part A
𝐿𝑜𝑤𝑒𝑟 𝑙𝑖𝑚𝑖𝑡 𝑜𝑓 𝑖𝑛𝑡𝑒𝑔𝑟𝑎𝑡𝑖𝑜𝑛 = 𝑎 = 0
𝑈𝑝𝑝𝑒𝑟 𝑙𝑖𝑚𝑖𝑡 𝑜𝑓 𝑖𝑛𝑡𝑒𝑔𝑟𝑎𝑡𝑖𝑜𝑛 = 𝑏 = 3
Put this all together to get:
3
32 02 9
∫ 𝑥 𝑑𝑥 = 𝐹(3) − 𝐹(0) = ( + 𝐶) − ( + 𝐶) =
0 2 2 2
Part B
Since the constant of integration cancels, we do not need to write the constant when evaluating the definite
integral:
7
72 32 49 9 40
∫ 𝑥 𝑑𝑥 = 𝐹(7) − 𝐹(3) = − = − = = 20
3 2 2 2 2 2

Example 1.125
Evaluate the integral below, and interpret it as an area.
4
∫ 𝑥 2 𝑑𝑥
−1

Calculate the associated indefinite integral:


𝑥3
∫ 𝑥 2 𝑑𝑥 = +𝐶
3
Calculate the given definite integral:

𝜋 𝜋
3 𝜋 𝜋 √2 2−√2 √2−2
∫𝜋2 cos 𝑥 𝑑𝑥 = −[sin 𝑥]𝜋2 = −
⏟ (sin 2 − sin 4 ) = − (1 − 2
) = −(
2
)=
2
4 4
=𝑋
Get all the files at: https://bit.ly/azizhandouts

4 𝑥=4
𝑥3
2
64 1 65
∫ 𝑥 𝑑𝑥 = [ ] = − (− ) =
−1 3 𝑥=−1 3 3 3

The integral is the area between 𝑦 = 𝑥 2 and the 𝑥 axis for 𝑥 = −1 to 𝑥 = 4.

Example 1.126: Exponential and Logarithmic Integrals


𝑒2
1
∫ 𝑑𝑥
𝑒 𝑥

𝑒2
1 2
∫ 𝑑𝑥 = [ln|𝑥|]𝑒𝑒 = ln|𝑒 2 | − ln|𝑒| = 2 − 1 = 1
𝑒 𝑥
Suppose we use the power rule:
𝑒2 𝑒2 2
−1
𝑥 −1+1 𝑒
𝑥0
∫ 𝑥 𝑑𝑥 = ∫ 𝑑𝑥 = ∫ 𝑑𝑥 ⇒ 𝑈𝑛𝑑𝑒𝑓𝑖𝑛𝑒𝑑
𝑒 𝑒 −1 + 1 𝑒 0

Example 1.127: Trigonometric Integrals


𝜋
A. ∫𝜋2 sin 𝑥 𝑑𝑥
4
𝜋
B. ∫ sec 2 𝑥 𝑑𝑥
𝜋
3

4
𝜋
C. ∫0 sin 𝑥 𝑑𝑥
𝜋
D. ∫04 sec 2 𝑥 𝑑𝑥
1
2
E. ∫ cos π𝑥 𝑑𝑥
1
3

𝜋
𝜋
2 𝑥= 1 1
∫ sin 𝑥 𝑑𝑥 = −[cos 𝑥] 𝜋2 = − (0 − )=
𝜋 𝑥= √2 √2
4 4
𝜋
𝜋
3 𝑥=
∫ sec 2 𝑥 𝑑𝑥 = [tan 𝑥] 3
𝜋 = √3 − 1
𝜋 𝑥=
4 4
𝜋
∫ sin 𝑥 𝑑𝑥 = [− cos 𝑥]𝜋0 = −[cos 𝑥]𝜋0 = −(−1 − 1) = 2
0
𝜋
𝜋
4 𝜋
∫ sec 2 𝑥 𝑑𝑥 = [tan 𝑥]04 = tan ( ) − tan 0 = 1 − 0 = 1
0 4
𝜋 1
2 sin 𝜋𝑥 2 1 𝜋 𝜋 1 √3 2 − √3
∫ cos π𝑥 𝑑𝑥 = [ ]1 = ( ) (sin − sin ) = ( ) (1 − ) =
𝜋 𝜋 𝜋 2 3 𝜋 2 2𝜋
3 3

1.128: Integral of Zero Width


𝑎
∫ 𝑓(𝑥) 𝑑𝑥 = 0
𝑎

➢ If you wish to find an integral where the upper and lower limit of integration are the same, then the
width of each rectangle is zero.
➢ The area of a rectangle of zero width is zero. Hence, the property above applies.
Get all the files at: https://bit.ly/azizhandouts

Example 1.129
3𝜋
∫ sin1947 𝑥 𝑑𝑥
3𝜋

3𝜋
∫ sin1947 𝑥 𝑑𝑥 = 0
3𝜋

C. Sum and Difference Property

1.130: Sum and Difference Property


𝑏 𝑏 𝑏
∫ [𝑓(𝑥) + 𝑔(𝑥)] 𝑑𝑥 = ∫ 𝑓(𝑥) 𝑑𝑥 + ∫ 𝑔(𝑥) 𝑑𝑥
𝑎 𝑎 𝑎
𝑏 𝑏 𝑏
∫ [𝑓(𝑥) − 𝑔(𝑥)] 𝑑𝑥 = ∫ 𝑓(𝑥) 𝑑𝑥 − ∫ 𝑔(𝑥) 𝑑𝑥
𝑎 𝑎 𝑎

Example 1.131
Split the integral:
2
∫ (𝑥 + 1) 𝑑𝑥
1

2 2 2
∫ (𝑥 + 1) 𝑑𝑥 = ∫ 𝑥 𝑑𝑥 + ∫ 1 𝑑𝑥
1 1 1

Example 1.132
2 2
If ∫1 [𝑓(𝑥) + 𝑥] 𝑑𝑥 = 4, then find ∫1 𝑓(𝑥) 𝑑𝑥

2
Split the integral ∫1 𝑓(𝑥) + 𝑥 𝑑𝑥 = 4
2 2
∴ ∫ 𝑓(𝑥) 𝑑𝑥 + ∫ 𝑥 𝑑𝑥 = 4
1 1
2 2
∫ 𝑓(𝑥) 𝑑𝑥 = 4 − ∫ 𝑥 𝑑𝑥
1 1

2
Substitute ∫1 𝑥 𝑑𝑥 = 𝐴𝑟𝑒𝑎 𝑢𝑛𝑑𝑒𝑟 𝑡ℎ𝑒 𝐶𝑢𝑟𝑣𝑒 𝑓𝑜𝑟 𝑦 = 𝑥 𝑜𝑣𝑒𝑟 𝑡ℎ𝑒 𝑖𝑛𝑡𝑒𝑟𝑣𝑎𝑙 [1,2] = 1.5
2
∴ ∫ 𝑓(𝑥) 𝑑𝑥 = 4 − 1.5 = 2.5
1

Example 1.133
16
1
∫ (√𝑥 + ) 𝑑𝑥
4 √𝑥

Rewrite in terms of exponents, and integrate:


3 16
16 1 1 2𝑥 2
∫ (𝑥 2 + 𝑥 −2 ) 𝑑𝑥 =[ + 2√𝑥]
4 3
4
Substitute the limits of integration:
Get all the files at: https://bit.ly/azizhandouts

3 3
2(16)2 2(4)2 128 16 112 124
( + 2√16) − ( + 2√4) = ( + 8) − ( + 4) = ( + 4) =
3 3 3 3 3 3

D. Constant Multiple Property

1.134: Constant Multiple Property


𝑏 𝑏
∫ 𝑐𝑓(𝑥) 𝑑𝑥 = 𝑐 ∫ 𝑓(𝑥) 𝑑𝑥
𝑎 𝑎

➢ If you apply a vertical scale to the function, the area under the curve scales proportional to the scale.
➢ This property is used many times when integrating. Hence, it is important to be able to do this
fluently.

Example 1.135
If ∫ 3𝑓(𝑥) 𝑑𝑥 = 5, find ∫ 𝑓(𝑥) 𝑑𝑥

1 1 5
∫ 𝑓(𝑥) 𝑑𝑥 = ∫ 3𝑓(𝑥) 𝑑𝑥 = × 5 =
3 3 3

1.136: Symmetry
If a function is symmetrical, about the origin, or about a point, the symmetry can be exploited to calculate a
definite integral without getting into calculations.

Example 1.137
3
∫ 𝑥 𝑑𝑥
−3

3
The integral ∫−3 𝑥 𝑑𝑥 represents the area between the 𝑥 −axis and the function
𝑓(𝑥) = 𝑥 from 𝑥 = −3 till 𝑥 = 3.

The function 𝑓(𝑥) = 𝑥 is symmetric about the origin. Hence, the green area is
exactly equal to the blue area.
0 3
∫ 𝑥 𝑑𝑥 = ∫ 𝑥 𝑑𝑥 = 𝑋
−3 0
Hence:
3 0 3
∫ 𝑥 𝑑𝑥 = ∫ 𝑥 𝑑𝑥 + ∫ 𝑥 𝑑𝑥 = 𝑋 − 𝑋 = 0
−3 −3 0

1.138: Odd Function


If 𝑓(𝑥) is an odd function, then
𝑎
∫ 𝑓(𝑥) 𝑑𝑥 = 0
−𝑎

A function which is symmetric about the origin is an odd function. That is:
𝑓(−𝑥) = −𝑓(𝑥)
Get all the files at: https://bit.ly/azizhandouts

Example 1.139
2𝜋
∫ sin2021 𝑥 𝑑𝑥
0

𝜋
The function sin2021 𝑥 is symmetric about 𝑥 = 2

𝜋 3𝜋
𝜋 𝜋
2 2
2021 2021
∫ sin 𝑥 + ∫ sin 𝑥 + ∫ sin2021 𝑥 + ∫ sin2021 𝑥
⏟0 𝜋 ⏟𝜋 3𝜋
⏟2 ⏟2
𝑋 −𝑋 −𝑋 𝑋
=𝑋−𝑋−𝑋+𝑋 =0

1.140: Even Function


If 𝑓(𝑥) is an even function, then
𝑎 𝑎
∫ 𝑓(𝑥) 𝑑𝑥 = 2 ∫ 𝑓(𝑥) 𝑑𝑥
−𝑎 0

A function which is symmetric about the 𝑦 − 𝑎𝑥𝑖𝑠 is an even function. That is:
𝑓(−𝑥) = 𝑓(𝑥)

E. Areas

Example 1.141
Find the area under the curve for the function 𝑓(𝑥) = 1 − 𝑥 2 from 𝑥 = 0 to 𝑥 = 1 using definite integration.

𝑥3
𝑓(𝑥) = 1 − 𝑥 2 ⇒ 𝐹(𝑥) = ∫(1 − 𝑥 2 ) 𝑑𝑥 = 𝑥 − +𝐶
3
Substitute 𝑓(𝑥) = 1 − 𝑥 2 ,𝑎 = 0, 𝑏 = 1 in
𝑏 1
13 03 2 2
∫ 𝑓(𝑥) 𝑑𝑥 = ∫ (1 − 𝑥 2 ) 𝑑𝑥 = 1 − + 𝐶 − 0 − + 𝐶 = + 𝐶 − 𝐶
⏟ =
𝑎 0
⏟ 3 ⏟ 3 ⏟
3 3
𝐹(0)
𝐹(1) 𝐹(0) 𝐹(1)
Note that the constant of integration got cancelled.
Hence, we will generally not write the constant of integration when doing definite integration.

Example 1.142
Find the area of the region:
A. Between the graph of 𝑓(𝑥) = 𝜋𝑥 2 , the 𝑥-axis, the line 𝑥 = 2 and the line 𝑥 = 4.
1
B. Between the graph of 𝑦 = 𝑥, the 𝑥-axis, the line 𝑥 = 𝑒 and the line 𝑥 = 𝑒 2 .

Part A
4 4 4
𝑥3 43 23 64 8 56𝜋
∫ 𝜋𝑥 2 𝑑𝑥 = 𝜋 ∫ 𝑥 2 𝑑𝑥 = 𝜋 [ ] = 𝜋( − ) = 𝜋( − ) =
2 2 3 2 3 3 3 3 3
Part B
𝑒2
1 2
∫ 𝑑𝑥 = [ln 𝑥]𝑒𝑒 = ln 𝑒 2 − ln 𝑒 = 2 − 1 = 1
𝑒 𝑥
Get all the files at: https://bit.ly/azizhandouts

Example 1.143
3
Determine ∫0 2𝑡 + 4 𝑑𝑡 and interpret it as an area.

3
∫ 2𝑡 + 4 𝑑𝑡 = [𝑡 2 + 4𝑡]𝑡=3
𝑡=0 = 21 − 0 = 21
0
This represents the area in the region between the lines:
𝑥 = 0, 𝑥 = 3, 𝑦 = 0, 𝑦 = 2𝑥 + 4
F. Unsigned Area

1.144: Splitting Limits of Integration


If 𝑏 ∈ [𝑎, 𝑏], then
𝑏 𝑐 𝑏
∫ 𝑓(𝑥) 𝑑𝑥 = ∫ 𝑓(𝑥) 𝑑𝑥 + ∫ 𝑓(𝑥) 𝑑𝑥
𝑎 𝑎 𝑐

➢ This property lets you “split” an integral into multiple integrals.


➢ This is useful when part of a function is above the 𝑥 axis, and part of the function is below the 𝑥 axis.

Example 1.145
Let 𝑓(𝑥) be an even function, graphed alongside. Split the integral:
2
∫ 𝑓(𝑥) 𝑑𝑥
−1

Split the limits of integration:


2 1 2
∫ 𝑓(𝑥) 𝑑𝑥 = ∫ 𝑓(𝑥) 𝑑𝑥 + ∫ 𝑓(𝑥) 𝑑𝑥
−1 −1 1

𝑎 𝑎
Use the property ∫−𝑎 𝑓(𝑥) 𝑑𝑥 = 2 ∫0 𝑓(𝑥) 𝑑𝑥 in the first term:
1 2
= 2 ∫ 𝑓(𝑥) 𝑑𝑥 + ∫ 𝑓(𝑥) 𝑑𝑥
0 1

1.146: Unsigned Area


Unsigned area is found by taking the absolute value of the area found by dividing the integral into parts such
that the signed area is either completely positive or completely negative.

Example 1.147
Find an expression for the:
A. signed area for the function 𝑓(𝑥) = 𝑥 2 − 1 over the interval [−2,2].
B. unsigned area for the function 𝑓(𝑥) = 𝑥 2 − 1 over the interval [−2,2].
Get all the files at: https://bit.ly/azizhandouts

Part A
The integral for signed area is:
2
∫ 𝑥 2 − 1 𝑑𝑥
−2
Part B
We need to know when:
𝑥 2 − 1 > 0 ⇒ (𝑥 + 1)(𝑥 − 1) > 0
This is a quadratic with leading coefficient >1, which means the graph
will be an upward parabola. The solution of the inequality is:
𝑥 ∈ (−∞, −1) ∪ (1, ∞) ⇒ 𝑥 > 0
Split the integral into three different parts:
−1 1 2
|∫ 𝑥 2 − 1 𝑑𝑥| + |∫ 𝑥 2 − 1 𝑑𝑥| + |∫ 𝑥 2 − 1 𝑑𝑥|
−2 −1 1
Since the first term and the last term are positive, and the middle term is negative, we get:
−1 1 2
= ∫ (𝑥 2 − 1) 𝑑𝑥 − ∫ (𝑥 2 − 1) 𝑑𝑥 + ∫ (𝑥 2 − 1) 𝑑𝑥
−2 −1 1

Example 1.148
Find an expression for the unsigned area for the function 𝑓(𝑥) = 𝑥 2 + 5𝑥 + 6 over the interval [−5,5].

𝑥 2 + 5𝑥 + 6 > 0 ⇒ (𝑥 + 2)(𝑥 + 3) > 0 ⇒ 𝑥 ∈ (−∞, −3) ∪ (−2, ∞)


The indefinite integral for 𝑓(𝑥) is:
𝑥 3 5𝑥 2
∫(𝑥 2 + 5𝑥 + 6) 𝑑𝑥 = + + 6𝑥 + 𝐶
3 2
The unsigned area is:
−3 −2 5
∫ 𝑓(𝑥) 𝑑𝑥 − ∫ 𝑓(𝑥) 𝑑𝑥 + ∫ 𝑓(𝑥) 𝑑𝑥
−5 −3 −2

Example 1.149
Find the unsigned area for the function 𝑓(𝑥) = 𝑥 2 + 𝑥 − 12 over the interval [−1,5].

𝑥 2 + 𝑥 − 12 > 0
(𝑥 + 4)(𝑥 − 3) > 0
𝑥 ∈ (−∞, −4) ∪ (3, ∞)

𝑥3 𝑥2
𝑓(𝑥) = 𝑥 2 + 𝑥 − 12 ⇒ 𝐹(𝑥) = + − 12𝑥 + 𝐶
3 2
3 5
− ∫ (𝑥 2 + 𝑥 − 12) 𝑑𝑥 + ∫ (𝑥 2 + 𝑥 − 12) 𝑑𝑥
−1 3
27 9 −1 1 125 25 27 9
= − [( + − 36) − ( + + 12)] + [( + − 60) − (( + − 36))]
3 2 3 2 3 2 3 2

Example 1.150
Find the unsigned area between the x-axis and the parabola 𝑓(𝑥) = 𝑥 2 − 1 from 𝑥 = 0 to 𝑥 = 2.

Split the integral at the roots, which are: 𝑥 2 − 1 = 0 ⇒ 𝑥 = ±1:


Get all the files at: https://bit.ly/azizhandouts

1 1
2
𝑥3 13 2
∫ 𝑥 − 1 = [ − 𝑥] = ( − 1) = −
0 3 0
3 3
2 2
2
𝑥3 23 13 2 2 4
∫ 𝑥 − 1 = [ − 𝑥] = ( − 2) − ( − 1) = − (− ) =
1 3 1
3 3 3 3 3
Hence, the area from 0 to 2 (taking the absolute value of the negative area) is:
2 4 6
+ = =2
3 3 3

Example 1.151
Find the unsigned area for the function 𝑓(𝑥) = 𝑥 3 − 𝑥 over the interval [−4,2].

To get unsigned area, we need to split the integral at the places where 𝑓(𝑥) < 0. Hence, we need to determine
the interval(s) over which the function is negative.
𝑥 3 − 𝑥 > 0 ⇒ 𝑥(𝑥 + 1)(𝑥 − 1) > 0
The roots of the expression on the LHS above are
{0, ±1}

(−∞, −1) (−1,0) (0,1) (1, ∞)


−𝑣𝑒 +𝑣𝑒 −𝑣𝑒 +𝑣𝑒

Unsigned area from −4 to 2 is:


−1 0 1 2
− ∫ (𝑥 3 − 𝑥) 𝑑𝑥 + ∫ (𝑥 3 − 𝑥) 𝑑𝑥 − ∫ (𝑥 3 − 𝑥) 𝑑𝑥 + ∫ (𝑥 3 − 𝑥) 𝑑𝑥
−4 −1 0 1

𝑥4 𝑥2
∫(𝑥 3 − 𝑥) 𝑑𝑥 = − +𝐶
4 2

Example 1.152
Find the unsigned area between 𝑓(𝑥) = 𝑥 3 + 7𝑥 2 + 16𝑥 + 12 and the 𝑥 − 𝑎𝑥𝑖𝑠 over the interval [−4,4]

𝑥 3 + 7𝑥 2 + 16𝑥 + 12 > 0
To find the factors of the cubic on the LHS, we use the Remainder Theorem. We want to see if 𝑥 − 𝑎 divides
the expression. The values of 𝑎 come from the factors of 12:
±{1,2,3,4,6,12}

We check with only positive values since 16𝑥 − 12 > 0, 𝑥 ≥ 1.


𝑓(−1) = −1 + 7 − 16 + 12 = 2 ≠ 0
𝑓(−2) = −8 + 28 − 32 + 12 = 0 ⇒ 𝑥 + 2 𝑖𝑠 𝑎 𝑓𝑎𝑐𝑡𝑜𝑟

𝑥 3 + 7𝑥 2 + 16𝑥 + 12
= 𝑥 2 + 5𝑥 + 6 = (𝑥 + 2)(𝑥 + 3)
𝑥+2

Original 𝑥3 𝑥2 𝑥 𝐶𝑜𝑛𝑠𝑡𝑎𝑛𝑡
Polynomial
1 7 16 12
−2 −2 −10 −12
(𝑥 + 2)2 (𝑥 + 3) > 0 1 5 6 0
Answer 𝑥2 𝑥 𝐶𝑜𝑛𝑠𝑡𝑎𝑛𝑡 Remainder
The function will be +𝑣𝑒 at +∞.
Get all the files at: https://bit.ly/azizhandouts

Since the root −2 is of even multiplicity, the function will not change sign over the interval (−3, −2). Since the
root −3 is of odd multiplicity, it will change sign.
(−∞, −3) (−3, −2) (−2, ∞)
−𝑣𝑒 +𝑣𝑒 +𝑣𝑒
Unsigned area from −4 to 4:
−3 4
− ∫ 𝑓(𝑥) 𝑑𝑥 + ∫ 𝑓(𝑥) 𝑑𝑥
−4 −3

Example 1.153
Find the unsigned area between the curve 𝑓(𝑥) = sin 𝑥 and the 𝑥 − 𝑎𝑥𝑖𝑠 over one period.

G. Integration with respect to 𝒚

Example 1.154
Find the area of the region inside the rectangle formed by the points (0,0)(4,0), (0,2), (4,2) and above the
curve 𝑦 = √𝑥.

Integrate with respect to 𝒙:


The area of the rectangle
= (2)(4) = 8
The area between the curve and the 𝑥 − 𝑎𝑥𝑖𝑠 is:
4
2 3 4 2 3 16 1
∫ √𝑥 𝑑𝑥 = [ 𝑥 2 ] = (42 ) = =5
0 3 0 3 3 3
The area that we want is:
1 2

8 − 5 =2
𝐴𝑟𝑒𝑎 𝑜𝑓
⏟3 3
𝑅𝑒𝑐𝑡𝑎𝑛𝑔𝑙𝑒 𝐴𝑟𝑒𝑎 𝑏𝑒𝑙𝑜𝑤 𝑡ℎ𝑒
𝐺𝑟𝑒𝑒𝑛 𝐶𝑢𝑟𝑣𝑒
Integrate with respect to 𝒚:
𝑦 = √𝑥 ⇒ 𝑥 = 𝑦 2
2 2
2
𝑦3 23 8 2
∫ 𝑦 𝑑𝑦 = [ ] = = =2
0 3 0 3 3 3

Example 1.155
Find the area of the region inside the rectangle formed by the points (0,0)(𝜋, 0), (0,1), (𝜋, 1) and above the
curve 𝑦 = sin 𝑥.

Integrate with respect to 𝒙:


The area between the curve and 𝑥 −axis is
𝜋
∫ (sin 𝑥) 𝑑𝑥 = −[cos 𝑥]𝜋0 = −(cos 𝜋 − cos 0) = −(−1 − (−1)) = −(−2) = 2
0

𝜋 − ⏟
2
𝐴𝑟𝑒𝑎 𝑜𝑓 𝐴𝑟𝑒𝑎 𝑏𝑒𝑙𝑜𝑤 𝑡ℎ𝑒
𝑅𝑒𝑐𝑡𝑎𝑛𝑔𝑙𝑒 𝐺𝑟𝑒𝑒𝑛 𝐶𝑢𝑟𝑣𝑒
Integrate with respect to 𝒚:
𝜋 𝜋
𝑦 = sin−1 𝑥 ⇒ 𝐷𝑜𝑚𝑎𝑖𝑛 = [−1,1], 𝑅𝑎𝑛𝑔𝑒 = [− , ]
2 2
1
2 ∫ (sin−1 𝑦) 𝑑𝑦
0
Get all the files at: https://bit.ly/azizhandouts

Example 1.156
1
∫ (sin−1 𝑦) 𝑑𝑦
0

𝑥 = sin−1 𝑦 ⇒ 𝑦 = sin 𝑥

𝜋
∫ (sin 𝑥) 𝑑𝑥 = −[cos 𝑥]𝜋0 = −(cos 𝜋 − cos 0) = −(−1 − (−1)) = −(−2) = 2
0
1
∫ (sin−1 𝑦) 𝑑𝑦 = ⏟
𝜋 − ⏟
2 =𝜋−2
0 𝐴𝑟𝑒𝑎 𝑜𝑓 𝐴𝑟𝑒𝑎 𝑏𝑒𝑙𝑜𝑤 𝑡ℎ𝑒
𝑅𝑒𝑐𝑡𝑎𝑛𝑔𝑙𝑒 𝐺𝑟𝑒𝑒𝑛 𝐶𝑢𝑟𝑣𝑒

Example 1.157
Phillips Exeter Math 4

Area under the curve


𝑎
2𝑎3
= ∫ 𝑥 2 𝑑𝑥 =
−𝑎 3
Area of the rectangle
= (2𝑎)(𝑎2 ) = 2𝑎3
Area above the curve
2𝑎3 4𝑎3
2𝑎3 − =
3 3
Ratio
2𝑎3 4𝑎3
= : = 2: 4 = 1: 2
3 3

H. Rationalization

Example 1.158
JEE Main 2024

1.6 Derivatives of Integrals (FTC)


A. Basics
The Fundamental Theorem of Calculus can be used to find the derivative of an integral.
Get all the files at: https://bit.ly/azizhandouts

1.159: Derivative of an Integral


𝑏
𝑑𝑦 𝑑 𝑑
𝑦 = ∫ 𝑓(𝑥) 𝑑𝑥 ⇒ = [𝐹(𝑏)] − [𝐹(𝑎)]
𝑎 𝑑𝑥 𝑑𝑥 𝑑𝑥

From the fundamental theorem of Calculus, we know that given a function 𝑓(𝑥) with integral 𝐹(𝑥), the
definite integral over the interval [𝑎, 𝑏] is:
𝑏
𝑦 = ∫ 𝑓(𝑥) 𝑑𝑥 = 𝐹(𝑏) − 𝐹(𝑎)
𝑎

Differentiate throughout with respect to 𝑥:


𝑏
𝑑𝑦 𝑑 𝑑 𝑑
= (∫ 𝑓(𝑥) 𝑑𝑥) = [𝐹(𝑏)] − [𝐹(𝑎)]
𝑑𝑥 𝑑𝑥 𝑎 𝑑𝑥 𝑑𝑥

Example 1.160
𝑑𝑦 𝑥
A. Find given that 𝑦 = ∫𝑎 𝑡 2 𝑑𝑡
𝑑𝑥
𝑑𝑦 𝑥
B. Find 𝑑𝑥 given that 𝑦 = ∫𝑎 sin 𝑡 𝑑𝑡

Part A 𝑓(𝑡) = 𝑡 2 ⇒ 𝑓(𝑥) = 𝑥 2


Differentiate both sides of the given equation: Part B
𝑥
𝑑𝑦 𝑑 Differentiate both sides of the given equation:
= (∫ 𝑡 2 𝑑𝑡) 𝑥
𝑑𝑥 𝑑𝑥 𝑎 𝑑𝑦 𝑑
= (∫ sin 𝑡 𝑑𝑡)
Use the fundamental theorem of calculus to 𝑑𝑥 𝑑𝑥 𝑎
calculate the integral: Use the fundamental theorem of calculus to
𝑑 𝑡 3 𝑎3 calculate the integral:
= [ − ] 𝑑
𝑑𝑥 3 3 = [− cos 𝑥 − (−cos 𝑎)]
𝑑 𝑥 3
𝑑 𝑎3 𝑑𝑥
= ( )− ( ) 𝑑 𝑑
𝑑𝑥 3 𝑑𝑥 3 = (− cos 𝑥) + (cos 𝑎)
𝑑𝑥 𝑑𝑥
Since the second term is a constant, its derivative Since the second term is a constant, its derivative
is zero: is zero:
= 𝑥2 = sin 𝑥
𝑥 2
Note that in 𝑦 = ∫𝑎 𝑡 𝑑𝑡:

1.161: Derivative of an Integral with a constant limit


𝑥
𝑑𝑦
𝑦 = ∫ 𝑓(𝑡) 𝑑𝑡 ⇒ = 𝑓(𝑥)
𝑎 𝑑𝑥

Differentiate both sides of the given equation:


𝑥
𝑑𝑦 𝑑
= (∫ 𝑓(𝑡) 𝑑𝑡)
𝑑𝑥 𝑑𝑥 𝑎
Use the fundamental theorem of calculus to calculate the integral (where 𝐹(𝑥) is an antiderivative of 𝑓(𝑥)):
𝑑
= [𝐹(𝑥) − 𝐹(𝑎)]
𝑑𝑥
𝑑 𝑑
= (𝐹(𝑥)) − 𝐹(𝑎)
𝑑𝑥 𝑑𝑥
Since the second term is a constant, its derivative is zero:
Get all the files at: https://bit.ly/azizhandouts

= 𝑓(𝑥)

Example 1.162
𝑑𝑦 𝑥
A. Evaluate if 𝑦 = ∫𝑎 sin(cos 𝑡) 𝑑𝑡
𝑑𝑥
𝑑𝑦 𝑥 2
B. Evaluate 𝑑𝑥 if 𝑦 = ∫𝑎 𝑒 𝑡 𝑑𝑡
𝑑𝑦 𝑥
C. Evaluate 𝑑𝑥 when 𝑥 = 3 given that 𝑦 = ∫𝑎 √𝑡 2 + 5𝑡 − 12 𝑑𝑡

Part A
Let 𝑓(𝑡) = sin(cos 𝑡) , 𝐹(𝑡) = ∫ 𝑓(𝑡) 𝑑𝑡:
𝑥
𝑦 = ∫ 𝑓(𝑡) 𝑑𝑡
𝑎
Using FTC:
= 𝐹(𝑥) − 𝐹(𝑎)
Differentiate the above:
𝑑𝑦 𝑑 𝑑
= [𝐹(𝑥)] − [𝐹(𝑎)] = 𝑓(𝑥) + 0 = sin(cos 𝑥)
𝑑𝑥 𝑑𝑥 𝑑𝑥
Part B
2 𝑥
Let 𝑓(𝑡) = 𝑒 𝑡 ⇒ 𝐹(𝑡) = ∫𝑎 𝑓(𝑡)𝑑𝑡:
𝑥
𝑦 = ∫ 𝑓(𝑡) 𝑑𝑡 = 𝐹(𝑥) − 𝐹(𝑎)
𝑎
Differentiate the above:
𝑑𝑦 𝑑 𝑑 2
= [𝐹(𝑥)] − [𝐹(𝑎)] = 𝑓(𝑥) = 𝑒 𝑥
𝑑𝑥 𝑑𝑥 𝑑𝑥
Part C
𝑑𝑦
| = √𝑥 2 + 5𝑥 − 12 = √9 + 15 − 12 = √12 = 2√3
𝑑𝑥 𝑥=3

B. Using the Chain Rule

Example 1.163
𝑑𝑦
Find 𝑑𝑥 given that
𝑥2
𝑦 = ∫ sin 𝑡 𝑑𝑡
𝑎

Method I: Finding the Integral


Differentiate both sides of the given equation:
𝑥2
𝑑𝑦 𝑑
= (∫ sin 𝑡 𝑑𝑡)
𝑑𝑥 𝑑𝑥 𝑎

Use the fundamental theorem of calculus to calculate the integral:


𝑑
= [− cos 𝑥 2 − (−cos 𝑎)]
𝑑𝑥

Use the sum and difference to split the derivative:


𝑑 𝑑
= (− cos 𝑥 2 ) + (cos 𝑎)
𝑑𝑥 𝑑𝑥
Get all the files at: https://bit.ly/azizhandouts

Since the second term is a constant, its derivative is zero:


𝑑 2
= sin 𝑥 2 × (𝑥 ) = 2𝑥 sin 𝑥 2
𝑑𝑥
Method II: Using FTC
Let 𝑓(𝑡) = sin 𝑡 , ∫ 𝑓(𝑡) 𝑑𝑡 = 𝐹(𝑡) + 𝐶. Then by FTC:
𝑥2
𝑦 = ∫ 𝑓(𝑡) 𝑑𝑡 = 𝐹(𝑥 2 ) − 𝐹(𝑎)
𝑎

Differentiate both sides of the above:


𝑑𝑦 𝑑 𝑑 𝑑 𝑑
= [𝐹(𝑥 2 )] − [𝐹(𝑎)] = [ 𝐹(𝑥 2 )] 𝑥 2 = 𝑓(𝑥 2 ) ∙ 2𝑥 = 2𝑥 sin 𝑥 2
𝑑𝑥 𝑑𝑥 𝑑𝑥 𝑑𝑥 𝑑𝑥

We did not need to find 𝐹(𝑥) at any stage in this calculation.

Example 1.164
𝑑𝑦
Without finding the integral, find 𝑑𝑥 given that:
√𝑥
𝑦 = ∫ 𝑒 sin 𝑡 𝑑𝑡
3
√𝑥

Let 𝑓(𝑡) = 𝑒 sin 𝑡 ⇒ ∫ 𝑒 sin 𝑡 𝑑𝑡 = 𝐹(𝑡) + 𝐶


√𝑥
3
𝑦=∫ 𝑓(𝑡) 𝑑𝑡 = 𝐹(√𝑥) − 𝐹( √𝑥 )
3
√𝑥
Differentiate the above:
3 3
𝑑𝑦 𝑑 3 𝑑 3 𝑓(√𝑥) 𝑓( √𝑥 ) 𝑒 sin √𝑥 𝑒 sin √𝑥
= 𝐹 ′ (√𝑥) (√𝑥) − 𝐹 ′ ( √𝑥 ) ( √𝑥 ) = − 2 = − 2
𝑑𝑥 𝑑𝑥 𝑑𝑥 2√𝑥 2 √ 𝑥
3𝑥 3 3𝑥 3

Example 1.165
𝑑𝑦 cos 𝑥 1
Without finding the integral, find 𝑑𝑥 given that 𝑦 = ∫sin 𝑥 𝑑𝑡
𝑡

1
Let 𝑓(𝑡) = ⇒ ∫ 𝑓(𝑡) = 𝐹(𝑡) + 𝐶
𝑡
cos 𝑥
1
𝑦=∫ 𝑑𝑡 = 𝐹(cos 𝑥) − 𝐹(sin 𝑥)
sin 𝑥 𝑡
Differentiate the above:
𝑑𝑦 sin 𝑥 cos 𝑥
= 𝑓(cos 𝑥)(cos 𝑥)′ − 𝑓(sin 𝑥)(sin 𝑥)′ = − − = − tan 𝑥 − cot 𝑥
𝑑𝑥 cos 𝑥 sin 𝑥

Example 1.166
𝑑𝑦
Without calculating the integral, find 𝑑𝑥 if:
𝑥3
𝑦 = ∫ (𝑡 4 + 1)𝑑𝑡
𝑥2

Let 𝑓(𝑡) = 𝑡 4 + 1 ⇒ ∫ 𝑓(𝑡) = 𝐹(𝑡) + 𝐶:


𝑥3
𝑦 = ∫ (𝑡 4 + 1) 𝑑𝑡
𝑥2
Get all the files at: https://bit.ly/azizhandouts

𝑦 = 𝐹(𝑥 3 ) − 𝐹(𝑥 2 )

Take the derivative of the first and the last term with respect to 𝑥:
𝑑𝑦 𝑑 𝑑
= [𝐹(𝑥 3 )] − [𝐹(𝑥 2 )]
𝑑𝑥 𝑑𝑥 𝑑𝑥

Use the chain rule:


𝑑 3 𝑑
= 𝐹 ′ (𝑥 3 ) 𝑥 − 𝐹 ′ (𝑥 2 ) 𝑥 2 = ⏟
𝑓(𝑥 3 )3𝑥 2 − 𝑓(𝑥 2 )2𝑥
𝑑𝑥 𝑑𝑥 ′
𝑭 (𝒙)=𝒇(𝒙)
Apply the definition of 𝑓(𝑥):
= [(𝑥 3 )4 + 1]3𝑥 2 − [(𝑥 2 )4 + 1]2𝑥 = 3𝑥 14 + 3𝑥 2 − 2𝑥 9 − 2𝑥 = 3𝑥 14 − 2𝑥 9 + 3𝑥 2 − 2𝑥

C. Challenging Questions

Challenge 1.167
This question from JEE Advanced makes use of:
1. Definition of definite integral (Reimann sums) as a limit
2. Logarithms
3. Summation and Product Notation
4. Increasing and Decreasing Functions
5. Inequalities
6. Taking limits by taking the logarithm

1.7 𝒖 −Substitution with Definite Integrals


A. Basics
We have already done 𝑢 substitution with indefinite integrals. We can use 𝑢 substitution with definite
integrals as well.

1.168: Method I: Changing the Limits


When doing definite integration with a change of variable, the limits of integration will also change. This
needs to be done carefully.

Example 1.169
Evaluate
1
A. ∫0 √1 + 8𝑥 𝑑𝑥
2
B. ∫1 2𝑥√𝑥 2 + 2 𝑑𝑥
1
C. ∫0 𝑥 2 √3𝑥 3 + 1 𝑑𝑥

Part A
𝑑𝑢
Let 𝑢 = 1 + 8𝑥 ⇒ 8
= 𝑑𝑥. Then:
𝑥 = 0 ⇒ 𝑢 = 1 + 8𝑥 = 1 + 8(0) = 1
𝑥 = 1 ⇒ 𝑢 = 1 + 8𝑥 = 1 + 8(1) = 9
Making the above substitutions:
1 9
√𝑢 1 2 3 9 1 2 1 2 13
∫ √1 + 8𝑥 𝑑𝑥 = ∫ 𝑑𝑢 = × 𝑢2 | = × (27 − 1) = × (26) =
0 1 8 8 3 1 8 3 8 3 6
Get all the files at: https://bit.ly/azizhandouts

Part B
Let 𝑢 = 𝑥 2 + 2 ⇒ 𝑑𝑢 = 2𝑥 𝑑𝑥. Then:
𝑥 = 1 ⇒ 𝑢 = 3, 𝑥=2⇒𝑢=6
Make the above substitutions:
𝑢=6
2 3 2 3 2
∫ √𝑢 𝑑𝑢 = 62 − 32 = (6√6 − 3√3) = 2(2√6 − √3) = 4√6 − 2√3
𝑢=3 3 3 3
Part C
Let 𝑢 = 3𝑥 3 + 5 ⇒ 𝑑𝑢 = 9𝑥 2 𝑑𝑥. Then:
𝑥 = 0 ⇒ 𝑢 = 3𝑥 3 + 5 = 3(0)3 + 1 = 1
𝑥 = 1 ⇒ 𝑢 = 3(1)3 + 1 = 3 + 1 = 4
Make the substitution:
1
1 4 1 2 3 𝑢=4 2 3 𝑢=4 2 14
2√ 3 (8 − 1) =
∫ 𝑥 3𝑥 + 1 𝑑𝑥 = ∫ √𝑢 𝑑𝑢 = × 𝑢 | 2 = × 𝑢2 | =
0 9 1 9 3 𝑢 = 1 27 𝑢=1 27 27

1.170: Method II: Find the associated indefinite integral


A definite integral can also be found by finding the associated indefinite integral (with or without u-
substitution). In this case, you do not need to change the limits of integration.

Example 1.171
2
1
∫ 𝑑𝑥
1 (𝑥 + 1)6

Method I
Let 𝑢 = 𝑥 + 1 ⇒ 𝑑𝑢 = 𝑑𝑥
𝑥 = 2 ⇒ 𝑢 = 3, 𝑥=1⇒𝑢=2
Make the above substitutions:
𝑥=2 𝑢=3 𝑢=3
1 1 1 1 1 1 1
∫ 6
𝑥 𝑑𝑥 = ∫ 6
𝑑𝑢 = ∫ − 𝑑𝑢 = [ − ] = −
𝑥=1 (𝑥 + 1) 𝑢=2 𝑢 𝑢=2 5𝑢5 5(35 ) 5(25 ) 1215 160
Method II
Let 𝑢 = 𝑥 + 1 ⇒ 𝑑𝑢 = 𝑑𝑥
1 1 −6
𝑢−5 (𝑥 + 1)−5 1
∫ 6
𝑑𝑥 = ∫ 6
𝑑𝑢 ⇒ ∫ 𝑢 𝑑𝑢 = + 𝐶 = +𝐶 =− +𝐶
(𝑥 + 1) 𝑢 −5 −5 (𝑥 + 1)5

2 2
1 1 1 1 1 1
∫ 6
𝑑𝑥 = − [ 5 ] = −[ 5
− 5 ]= −
1 (𝑥 + 1) 5(𝑥 + 1) 1 5(2 + 1) 5(1 + 1) 1215 160

B. Trigonometric Integrals

Example 1.172
𝜋
4
∫ cos 2𝑥 𝑑𝑥
0

Method I
Let 𝑢 = 2𝑥 ⇒ 𝑑𝑢 = 2 𝑑𝑥
𝜋 𝜋 𝜋
𝑊ℎ𝑒𝑛 𝑥 = 0 ⇒ 𝑢 = 0, 𝑊ℎ𝑒𝑛 𝑥 = ⇒𝑢 =2× =
4 4 2
Make the above substitutions:
Get all the files at: https://bit.ly/azizhandouts

𝜋 𝜋
𝑥=
4 1 𝑢= 2 1 𝜋 1 1
∫ cos 2𝑥 𝑑𝑥 = ∫ cos 𝑢 𝑑𝑢 = [sin − sin 0] = (1) =
𝑥=0 2 𝑢=0 2 2 2 2

Method II
Let 𝑢 = 2𝑥 ⇒ 𝑑𝑢 = 2 𝑑𝑥
1 1 1
∫ cos 2𝑥 𝑑𝑥 = ∫ cos 𝑢 𝑑𝑢 = sin 𝑢 + 𝐶 = sin 2𝑥 + 𝐶
2 2 2

𝜋 𝜋
4 1 4
∫ cos 2𝑥 𝑑𝑥 = [ sin 2𝑥] = [ ]
0 2 0

C. Inverse Trigonometric Integrals

Example 1.173
1 −1 𝑥 12+9𝑥 2
If α = ∫0 (𝑒 9𝑥+3 tan )( ) 𝑑𝑥 , where tan−1 𝑥 takes only principal values, then the value of
1+𝑥 2
3𝜋
ln(|1 + 𝛼|) − 4
is: (JEE Advanced 2015)

Make the substitution


−1
3 12 + 3𝑥 2
𝑢 = 9𝑥 + 3 tan 𝑥 ⇒ 𝑑𝑢 = 9 + 𝑑𝑥 = 𝑑𝑥
1 + 𝑥2 1 + 𝑥2
Then the limits of integration change to:
3𝜋
𝑥 = 0 ⇒ 𝑢 = 0, 𝑥 =1⇒𝑢 =9+
4
Substitute the above in the given integral:
3𝜋
9+ 3𝜋 3𝜋
4 9+
α=∫ 𝑒 𝑢 𝑑𝑢 = [𝑒 𝑢 ]0 4
= 𝑒 9+ 4 − 1
0
3𝜋
Add 1 to both sides, and substitute 𝛼 + 1 = 𝑒 9+ 4 to evaluate:
3𝜋 3𝜋 3𝜋 3𝜋 3𝜋
ln(|1 + 𝛼|) − = ln (|𝑒 9+ 4 |) − =9+ − =9
4 4 4 4

1.8 Average Value, Area between Curves


A. Mean of a Function

1.174: Mean (Average) of a function


If a function is integrable on a closed interval [𝑎, 𝑏], then:
𝑏
1
𝐴𝑣𝑒𝑟𝑎𝑔𝑒 𝑉𝑎𝑙𝑢𝑒 𝑜𝑓 𝑓(𝑥) = ∫ 𝑓(𝑥) 𝑑𝑥
𝑏−𝑎 𝑎

Since we want the average value, we distribute the total area so that it forms a rectangle. For a rectangle:
𝐴𝑟𝑒𝑎 = 𝐻𝑒𝑖𝑔ℎ𝑡 ∙ 𝑊𝑖𝑑𝑡ℎ

Solve the above expression for 𝐻𝑒𝑖𝑔ℎ𝑡:


𝑏 𝑏
𝐴𝑟𝑒𝑎 ∫ 𝑓(𝑥) 𝑑𝑥 1
𝐻𝑒𝑖𝑔ℎ𝑡 = = 𝑎 = ∫ 𝑓(𝑥) 𝑑𝑥
𝑊𝑖𝑑𝑡ℎ 𝑏−𝑎 𝑏−𝑎 𝑎

Example 1.175
Get all the files at: https://bit.ly/azizhandouts

Find the average value of:


A. 𝑓(𝑥) = 𝑥 2 over the range [2,4].
𝜋
B. 𝑓(𝑥) = sin 𝑥 over the range [0, 2 ].
C. 𝑓(𝑥) = 𝑒 𝑥 over the range [2,4]

Part A
𝑏 4 4
1 1 2
1 𝑥3 1 64 8 1 56 28
∫ 𝑓(𝑥) 𝑑𝑥 = ∫ 𝑥 𝑑𝑥 = [ ] = ( − ) = ( ) =
𝑏−𝑎 𝑎 4−2 2 2 3 2 2 3 3 2 3 3
Part B
𝜋
𝜋
1 2 2 2
2 2
𝜋 ∫ sin 𝑥 𝑑𝑥 = − [cos 𝑥]0 = (− ) (0 − 1) =
𝜋 𝜋 𝜋
2−0 0
Part C
4
1 1 1
∫ 𝑒 𝑥 𝑑𝑥 = [𝑒 𝑥 ]42 = (𝑒 4 − 𝑒 2 )
4−2 2 2 2

Example 1.176
Find the average value of the function 𝑓(𝑥) = |𝑥| over the interval (−2,3).

Use the definition:


3
1 1 3
∫ |𝑥| 𝑑𝑥 = ∫ |𝑥| 𝑑𝑥
3 − (−2) −2 5 −2

Since it is the absolute value function, we need to consider it


piece-wise and split the integral:
1 0 3
= [∫ |𝑥| 𝑑𝑥 + ∫ |𝑥| 𝑑𝑥 ]
5 −2 0
1 0 3
= [∫ −𝑥 𝑑𝑥 + ∫ 𝑥 𝑑𝑥 ]
5 −2 0
0 3
1 𝑥2 𝑥2 1 4 9 1 13 13
= [[− ] + [ ] ] = [(0 − (− )) + ( )] = ( ) = = 1.3
5 2 −2 2 0 5 2 2 5 2 10

B. Area between Curves


Area between curves is given by the integral of difference of two curves.

Example 1.177
The graph alongside shows:
𝑓(𝑥) = 𝑥 2 + 2𝑥 − 2
𝑔(𝑥) = 𝑥
Find the area between 𝑓(𝑥) and 𝑔(𝑥).

From the graph the points of intersection of 𝑓(𝑥) and 𝑔(𝑥) are −2 and 1.
Hence, we want to find area over the interval:
[−2,1]
Get all the files at: https://bit.ly/azizhandouts

If we divide this area into rectangles, the height of a rectangle at a point 𝑥


will be given by:
𝑔(𝑥) − 𝑓(𝑥)

Combining the above two, and writing the required area as an integral
gives us:
1
∫ [𝑔(𝑥) − 𝑓(𝑥)] 𝑑𝑥
−2
Substituting the definition of 𝑓(𝑥) and 𝑔(𝑥) gives us:
1 1
∫ [𝑥 − (𝑥 2 + 2𝑥 − 2)] 𝑑𝑥 = ∫ (−𝑥 2 − 𝑥 + 2) 𝑑𝑥
−2 −2

Integrate, and evaluate:


1
𝑥3 𝑥2 1 1 8 4 9 3
= [− − + 2𝑥] = (− − + 2) − ( − − 4) = − + + 6 = −3 + 1.5 + 6 = 4.5
3 2 −2
3 2 3 2 3 2

Example 1.178
Find the area between the curves 𝑓(𝑥) = 𝑥 2 + 9𝑥 + 12 and 𝑔(𝑥) = 2𝑥.

Equate the two functions to find the points of intersection:


𝑥 2 + 9𝑥 + 12 = 2𝑥
𝑥 2 + 7𝑥 + 12 = 0
(𝑥 + 3)(𝑥 + 4) = 0
𝑥 ∈ {−3, −4}

These points of intersection are also the bounds of integration. Since we do not
know whether the area is positive or negative, we take the absolute value:
−3
|∫ (𝑥 2 + 7𝑥 + 12)𝑑𝑥 |
−4

Evaluate the integral:


−3
𝑥 3 7𝑥 2
= |[ + + 12𝑥] |
3 2 −4
Substitute the limits of integration:
63 64
= |(−9 + − 36) − (− + 56 − 48)|
2 3
Simplify:
63 64 −318 + 189 + 128 −1 1
= |−45 + + − 8| = | |=| |=
2 3 6 6 6

Example 1.179
Find the area enclosed between the parabola 4𝑦 = 3𝑥 2 and the straight line 3𝑥 − 2𝑦 + 12 = 0. (CBSE 2017)
Get all the files at: https://bit.ly/azizhandouts

Points of intersection
Solve the equation of the line for 𝑦:
3𝑥 + 12
3𝑥 − 2𝑦 + 12 = 0 ⇒ 𝑦 =
2
Substitute the above in 2(2𝑦)𝑦 = 3𝑥 2
2(3𝑥 + 12) = 3𝑥 2
3𝑥 2 − 6𝑥 − 24 = 0
𝑥 2 − 2𝑥 − 8 = 0
𝑥 ∈ {−2,4}

Integrate using the points of intersection as the limits


3𝑥 2 3𝑥+12 𝑏
Substitute 𝑓(𝑥) = 4
, 𝑔(𝑥) = 2
,𝑎 = −2, 𝑏 = 4 in ∫𝑎 [𝑓(𝑥) − 𝑔(𝑥)] 𝑑𝑥
4
3𝑥 + 12 3𝑥 2
∫ ( − ) 𝑑𝑥
−2 2 4

Integrate:
4
3𝑥 2 𝑥3
=[ + 6𝑥 − ]
4 4 −2
Evaluate using the limits of integration:
= (12 + 24 − 16) − (3 − 12 + 2) = 20 + 7 = 27

C. Multiple intersections

Example 1.180
The graph alongside shows:
𝑓(𝑥) = 𝑥 3
𝑔(𝑥) = 2𝑥
Find the area between 𝑓(𝑥) and 𝑔(𝑥).

To find the points of intersection, equate the two functions:


𝑥 3 = 2𝑥 ⇒ 𝑥 3 − 2𝑥 = 0 ⇒ 𝑥(𝑥 2 − 2) = 0 ⇒ 𝑥 ∈ {0, ±√2}

Method I
To find the bounds of integration:
𝑥 3 > 2𝑥 ⇒ 𝑥 3 − 2𝑥 > 0

(−∞, −√2) (−√2, 0) (0, √2) (√2, ∞)


−𝑣𝑒 +𝑣𝑒 −𝑣𝑒 +𝑣𝑒

√2 0
∫ [𝑔(𝑥) − 𝑓(𝑥)] 𝑑𝑥 + ∫ [𝑓(𝑥) − 𝑔(𝑥)] 𝑑𝑥
0 −√2

Method II: Symmetry

ℎ(𝑥) = 𝑓(𝑥) − 𝑔(𝑥) = 𝑥 3 − 2𝑥


ℎ(−𝑥) = (−𝑥)3 − 2(−𝑥) = −𝑥 3 + 2𝑥 = −ℎ(𝑥) ⇒ 𝑂𝑑𝑑 𝐹𝑢𝑛𝑐𝑡𝑖𝑜𝑛
Get all the files at: https://bit.ly/azizhandouts

From symmetry, the second integral is the same as the first integral.
√2 √2 √2
3)
𝑥4
2
= 2 ∫ [𝑔(𝑥) − 𝑓(𝑥)] 𝑑𝑥 = 2 ∫ (2𝑥 − 𝑥 𝑑𝑥 = 2 [𝑥 − ]
0 0 4 0

Substitute the limits of integration and simplify:


4
(√2)
2 4
= 2 [(√2) − ] = 2 [2 − ] = 2[1] = 2
4 4

Example 1.181
Find the area between 𝑓(𝑥) = 𝑥 3 − 𝑥 and 𝑔(𝑥) = 𝑥.

To find the points of intersection, equate the two functions:


𝑥 3 − 𝑥 = 𝑥 ⇒ 𝑥 3 − 2𝑥 = 0 ⇒ 𝑥(𝑥 2 − 2) = 0 ⇒ 𝑥 ∈ {0, ±√2}
To find the bounds of integration, solve the inequality:
ℎ(𝑥) = 𝑓(𝑥) − 𝑔(𝑥) > 0 ⇒ 𝑥 3 − 2𝑥 > 0
We already know the critical points from the equation we solved earlier. Make a sign diagram:

(−∞, −√2) (−√2, 0) (0, √2) (√2, ∞)


−𝑣𝑒 +𝑣𝑒 −𝑣𝑒 +𝑣𝑒
0 √2
∫ ℎ(𝑥) 𝑑𝑥 − ∫ ℎ(𝑥) 𝑑𝑥
−√2 0

0 0 0
𝑥4
∫ ℎ(𝑥) 𝑑𝑥 = ∫ (𝑥 3 − 2𝑥) 𝑑𝑥 = [ − 𝑥 2 ] = −(1 − 2) = 1
−√2 −√2 4 − √2

Note that ℎ(𝑥) = 𝑥 3 − 2𝑥 is symmetric about the origin:


ℎ(−𝑥) = (−𝑥)3 − 2(−𝑥) = −𝑥 3 + 2𝑥 = −ℎ(𝑥) ⇒ 𝑂𝑑𝑑 𝐹𝑢𝑛𝑐𝑡𝑖𝑜𝑛

√2 0
− ∫ ℎ(𝑥) 𝑑𝑥 = ∫ ℎ(𝑥) 𝑑𝑥 = 1
0 −√2
Hence, the final answer is:
1+1=2

Example 1.182
Find the area between 𝑓(𝑥) = sin 𝑥 and 𝑔(𝑥) = sin 2𝑥 over the interval [0,2𝜋].

To find the points of intersection, equate the two functions:


sin 2𝑥 = sin 𝑥
Use the identity sin 2𝑥 = 2 sin 𝑥 cos 𝑥
2 sin 𝑥 cos 𝑥 = sin 𝑥
Case I: If sin 𝑥 = 0, the equation is satisfied:
sin 𝑥 = 0 ⇒ 𝑥 ∈ {0, 𝜋, 2𝜋 }
Get all the files at: https://bit.ly/azizhandouts

Case II: If sin 𝑥 ≠ 0, divide both sides by sin 𝑥:


1 𝜋 5𝜋
2 cos 𝑥 = 1 ⇒ cos 𝑥 = ⇒ 𝑥 ∈ { , }
2 3 3

Example 1.183
The blue region is the area between 𝑦 = 1, and 𝑦 = 1 + cos 𝑥. The
brown region is the area between 𝑦 = 2, 𝑦 = 1 and 𝑦 = 1 + cos 𝑥.
Find the ratio of the blue area to that of the brown area.

Shift the curves down by 1 unit each to get the diagram alongside.
Blue Region
The blue region is the absolute value of:
3𝜋
3𝜋
2 3𝜋 𝜋
∫ cos 𝑥 𝑑𝑥 = [sin 𝑥]𝜋2 = sin ( ) − sin ( ) = −1 − 1 = −2
𝜋
2 2 2
2
And hence
𝐴𝑟𝑒𝑎 𝑜𝑓 𝐵𝑙𝑢𝑒 𝑅𝑒𝑔𝑖𝑜𝑛 = |−2| = 2
Brown Region
For the brown region, calculate:
2𝜋
∫ (1 − cos 𝑥)𝑑𝑥 = [𝑥 − sin 𝑥]2𝜋
0 = [2𝜋 − sin(2𝜋)] − [0 − sin(0)] = 2𝜋
0
And t
𝐴𝑟𝑒𝑎 𝑜𝑓 𝐵𝑟𝑜𝑤𝑛 𝑅𝑒𝑔𝑖𝑜𝑛 = 2𝜋 − 2
Finally, the required ratio is:
2: 2𝜋 − 2 = 1: 𝜋 − 1

D. Area of a Triangle

Example 1.184
Using integration, find the area of Δ𝐴𝐵𝐶, the coordinates of whose vertices are 𝐴(2,5), 𝐵(4,7) and 𝐶(6,2).
(CBSE 2010, 2011, 2019)

Substitute 𝑚 = 1, (𝑥1 , 𝑦1 ) = (2,5) in 𝑦 − 𝑦1 = 𝑚(𝑥 − 𝑥1 ) to


find the equation of the line 𝐴𝐵:
𝑦 − 5 = 1(𝑥 − 2) ⇒ 𝑦 = 𝑥 + 3

5
𝑚 = 2 , (𝑥1 , 𝑦1 ) = (4,7) in 𝑦 − 𝑦1 = 𝑚(𝑥 − 𝑥1 ) to find the
equation of the line 𝐵𝐶:
5 5
𝑦 − 7 = − (𝑥 − 4) ⇒ 𝑦 = − 𝑥 + 17
2 2
3
𝑚 = − 4 , (𝑥1 , 𝑦1 ) = (2,5) in 𝑦 − 𝑦1 = 𝑚(𝑥 − 𝑥1 ) to find the
equation of the line 𝐴𝐶:
3 3 13
𝑦 − 5 = − (𝑥 − 2) ⇒ 𝑦 = − 𝑥 +
4 4 2
4 6 6
∫ 𝐿𝑖𝑛𝑒 𝐴𝐵 + ∫ 𝐿𝑖𝑛𝑒 𝐵𝐶 − ∫ 𝐿𝑖𝑛𝑒 𝐴𝐶
2 4 2
Get all the files at: https://bit.ly/azizhandouts

4 6 6
5 3 13
∫ (𝑥 + 3) 𝑑𝑥 + ∫ (− 𝑥 + 17) 𝑑𝑥 − ∫ (− 𝑥 + ) 𝑑𝑥
2 4 2 2 4 2
The final answer is:
𝐴𝑟𝑒𝑎 = 7

4 6
∫ (𝐿𝑖𝑛𝑒 𝐴𝐵 − 𝐿𝑖𝑛𝑒 𝐴𝐶) + ∫ (𝐿𝑖𝑛𝑒 𝐵𝐶 − 𝐿𝑖𝑛𝑒 𝐴𝐶)
2 4

Example 1.185
𝑦 = 𝑥2
𝑦 = −𝑥 2 + 7
𝑦 = sin 𝑥

E. Integrating with respect to y


So far, we have been integrating with respect to 𝑥, drawing vertical rectangles (in terms of the Reimann sum),
and finding the area. Instead of drawing vertical rectangles, we can draw horizontal rectangles, create a
Reimann sum, write the Reimann sum as an integral, and evaluate. Most importantly, note that we will now be
integrating with respect to 𝑦.

Example 1.186
3
Find the area between the curves 𝑦 = √𝑥 and 𝑦 = √𝑥 by
A. Integrating with respect to 𝑥
B. Integrating with respect to 𝑦

Integrate with respect to 𝒙:


Find the points of intersection:
3
√𝑥 = √𝑥 ⇒ 𝑥 ∈ {0,1}
Find the integral:
1
3 1
∫ ( √𝑥 − √𝑥) 𝑑𝑥 =
0 12
Integrate with respect to 𝒚:
Get all the files at: https://bit.ly/azizhandouts

3
𝑦 = √𝑥 ⇒ 𝑥 = 𝑦 3 , 𝑦 = √𝑥 ⇒ 𝑥 = 𝑦 2
𝑦 3 = 𝑦 2 ⇒ 𝑦 ∈ {0,1}
1
1 1 1
∫ (𝑦 3 − 𝑦 2 ) 𝑑𝑥 = − ⇒ 𝐴𝑟𝑒𝑎 = |− | =
0 12 12 12

F. Bounded Regions

Example 1.187
Using integration, prove that curves 𝑦 2 = 4𝑥 and 𝑥 2 = 4𝑦 divide the area of the
square bounded by 𝑥 = 0, 𝑥 = 4, 𝑦 = 4 and 𝑦 = 0 into three equal parts. (CBSE
2009, 2015, 2019)

Points of Intersection

G. Back Calculations

1.9 Volumes with Cross Sections


A. Volumes with Cross Sections

1.188: Volume using Cross Sections


A solid with cross sectional area 𝐴(𝑥) from 𝑥 = 𝑎 to 𝑥 = 𝑏 has volume given by the definite integral of 𝐴(𝑥)
from 𝑎 to 𝑏:
𝑏
𝑉𝑜𝑙𝑢𝑚𝑒 = ∫ 𝐴(𝑥) 𝑑𝑥
𝑎

Example 1.189: Warmup


Show that:
A. A cuboid with length 𝑙, width 𝑤, and height ℎ has volume 𝑙𝑤ℎ.
B. A cylinder with base radius 𝑟, and height ℎ has volume 𝜋𝑟 2 ℎ.
C. (𝑁𝑜𝑛 𝐶𝑜𝑛𝑠𝑡𝑎𝑛𝑡 𝐶𝑟𝑜𝑠𝑠 𝑆𝑒𝑐𝑡𝑖𝑜𝑛𝑎𝑙 𝐴𝑟𝑒𝑎) A rectangular pyramid with length 𝑙, width 𝑤, and height ℎ has
1
volume 3 𝑙𝑤ℎ.
1
D. The volume of a circular cone with radius 𝑟 is 3 𝜋𝑟 2 ℎ.
1
E. A pyramid with area of base 𝐵 has volume 𝐵ℎ.
3

Part A
Introduce a coordinate system with origin at bottom left of the cuboid. The bottom
left of the cuboid has 𝑦 = 0 and the top left of the cuboid has 𝑦 = ℎ, where ℎ is the
height of the cuboid.

The limits of integration are:


𝑦 = 0, 𝑦 = ℎ

Any horizontal cross section of the cuboid at height 𝑦 has area:


𝐴(𝑦) = 𝑙𝑤 = 𝐶𝑜𝑛𝑠𝑡𝑎𝑛𝑡
Get all the files at: https://bit.ly/azizhandouts

Hence, the volume is:


ℎ ℎ
∫ 𝑙𝑤 𝑑𝑦 = 𝑙𝑤 ∫ 1 𝑑𝑦 = 𝑙𝑤[𝑦]ℎ0 = 𝑙𝑤ℎ
0 0

Part B
Any horizontal cross section of the cylinder at height 𝑦 has area:
𝐴(𝑦) = 𝜋𝑟 2 = 𝐶𝑜𝑛𝑠𝑡𝑎𝑛𝑡
Hence, the volume is:
ℎ ℎ
∫ 𝜋𝑟 2 𝑑𝑦 = 𝜋𝑟 2 ∫ 1 𝑑𝑦 = 𝜋𝑟 2 [𝑦]ℎ0 = 𝜋𝑟 2 ℎ
0 0

Part C
Note that for a general rectangular pyramid, it is not necessary that the apex
of the pyramid is above the center of the base.

Introduce a coordinate system with origin at the apex of the pyramid, and
positive in the downward direction.

Note that any horizontal cross section of the pyramid will be similar to the
base. The scaling factor due to the similarity will be:
𝑦

In fact, if 𝑦 is the distance from the top of the pyramid, then the area of a horizontal cross section of the
pyramid is:
𝑦 𝑦 𝑙𝑤
𝐴(𝑦) = (𝑙 ∙ ) (𝑤 ∙ ) = 2 ∙ 𝑦 2
⏟ ℎ ⏟ ℎ ℎ
𝐿𝑒𝑛𝑔𝑡ℎ 𝑊𝑖𝑑𝑡ℎ
The volume is:
ℎ ℎ
𝑙𝑤 𝑙𝑤 ℎ 𝑙𝑤 𝑦 3 𝑙𝑤 ℎ3 1
∫ 2 ∙ 𝑦 2 𝑑𝑦 = 2 ∫ 𝑦 2 𝑑𝑦 = 2 [ ] = 2 ∙ = 𝑙𝑤ℎ
0 ℎ ℎ 0 ℎ 3 0 ℎ 3 3
Part D
Using similarity, if 𝑦 is the distance from the top of the pyramid, then the
area of a horizontal cross section of the pyramid is:
𝑦2
𝐴(𝑦) = 𝜋𝑟 2 ∙ 2

The volume is:
ℎ ℎ
𝑦2 𝜋𝑟 2 ℎ 𝜋𝑟 2 𝑦 3 𝜋𝑟 2 ℎ3 1 2
∫ 𝜋𝑟 ∙ 2 𝑑𝑦 = 2 ∫ 𝑦 2 𝑑𝑦 = 2 [ ] = 2 ∙
2
= 𝜋𝑟 ℎ
0 ℎ ℎ 0 ℎ 3 0 ℎ 3 3
Part E
Using similarity, if 𝑦 is the distance from the top of the pyramid, then the area of a horizontal cross section of
the pyramid is:
𝑦2
𝐴(𝑦) = 𝐵 ∙ 2

The volume is:
ℎ ℎ
𝑦2 𝐵 ℎ 2 𝐵 𝑦3 𝐵 ℎ3 1
∫ 𝐵∙ 𝑑𝑦 = ∫ 𝑦 𝑑𝑦 = [ ] = ∙ = 𝐵ℎ
0 ℎ2 ℎ2 0 ℎ2 3 0 ℎ2 3 3

Example 1.190: Semi-Circular Cross Sections


Get all the files at: https://bit.ly/azizhandouts

A solid region has a circular base with center at the origin and a radius of 4. Cross sections perpendicular to
the 𝑥-axis are semi-circles. Find the volume of the solid using integration.

Recognize that the shape is a hemisphere with volume:


1 4 1 4 128
= ∙ 𝜋𝑟 3 = ∙ 𝜋(4)3 = 𝜋
2 3 2 3 3

As shown in the diagram, the radius of the semi-circle is 𝑦. Area of the semicircle
1 1 𝜋
= 𝐴(𝑦) = ∙ 𝜋𝑟 2 = ∙ 𝜋𝑦 2 = 𝑦 2
2 2 2

Substitute 𝑦 2 + 𝑥 2 = 𝑟 2 ⇒ 𝑦 2 + 𝑥 2 = 42 ⇒ 𝑦 2 = 16 − 𝑥 2
𝜋
𝐴(𝑥) = (16 − 𝑥 2 )
2

The limits of integration are:


𝑎 = −4, 𝑏 = 4
𝑏 4
𝜋 𝜋 4
𝑉𝑜𝑙𝑢𝑚𝑒 = ∫ 𝐴(𝑥) 𝑑𝑥 = ∫ [ (16 − 𝑥 2 )] 𝑑𝑥 = ∫ [16 − 𝑥 2 ]𝑑𝑥
𝑎 −4 2 2 −4

Since 𝐴(𝑥) is an even function, we can find use the property of a symmetric function:
4
= 𝜋 ∫ [16 − 𝑥 2 ] 𝑑𝑥
0

Example 1.191
The region 𝑅 is the circle with radius 3 centered at the origin. Cross sections perpendicular to the 𝑥 axis that
lie in the region 𝑅 are squares. Find the volume.

Solve the equation of a circle for 𝑦 2 :


𝑥2 + 𝑦2 = 9 ⇒ 𝑦2 = 9 − 𝑥2

Let the side of the square be:


𝑠(𝑥) = 2𝑦

The area function is then:


𝐴(𝑥) = [𝑠(𝑥)]2 = (2𝑦)2 = 4𝑦 2 = 4(9 − 𝑥 2 )

The integral that gives us the volume is:


𝑏 3 3
∫ 𝐴(𝑥) 𝑑𝑥 = 4 ∫ (9 − 𝑥 2 ) 𝑑𝑥 = 8 ∫ (9 − 𝑥 2 ) 𝑑𝑥 = 8(18) = 144
𝑎 −3 0

B. Disc Method

1.192: Disc Method: Revolving around the 𝒙 − 𝒂𝒙𝒊𝒔


A solid generated by revolving the curve 𝑦 = 𝑓(𝑥) around the 𝑥 − 𝑎𝑥𝑖𝑠 from 𝑥 = 𝑎 to 𝑥 = 𝑏 has volume given
by:
𝑏
𝑉𝑜𝑙𝑢𝑚𝑒 = 𝜋 ∫ [𝑓(𝑥)]2 𝑑𝑥
𝑎
Get all the files at: https://bit.ly/azizhandouts

➢ The cross section of the solid is a circle with 𝑟𝑎𝑑𝑖𝑢𝑠 = 𝑅(𝑥) = 𝑓(𝑥)
➢ Hence, the area is found by using the formula for the area of a circle:
= 𝐴(𝑥) = 𝜋[𝑅(𝑥)]2 = 𝜋[𝑓(𝑥)]2

Example 1.193
3
Find, in terms of 𝜋, the volume of the solid generated by revolving the curve 𝑦 = √𝑥 , 1 ≤ 𝑥 ≤ 8 around the
𝑥 −axis.

3
Substitute 𝑎 = 1, 𝑏 = 8, 𝑓(𝑥) = √𝑥 :
𝑏 8 8 2
3 2
𝑉𝑜𝑙𝑢𝑚𝑒 = 𝜋 ∫ [𝑓(𝑥)]2 𝑑𝑥 = 𝜋 ∫ [ √𝑥 ] 𝑑𝑥 = 𝜋 ∫ 𝑥 3 𝑑𝑥
𝑎 1 1
Integrate and simplify:
3 5 8 3𝜋 5 5 3𝜋 93𝜋
= 𝜋 ∙ [𝑥 3 ] = (83 − 13 ) = (32 − 1) =
5 1 5 5 5

Example 1.194
Find, in terms of 𝜋, the volume of the solid generated by revolving the curve 𝑦 = sin 𝑥 , 0 ≤ 𝑥 ≤ 𝜋 around the
𝑥 −axis.

Substitute 𝑎 = 0, 𝑏 = 𝜋, 𝑓(𝑥) = sin 𝑥:


𝑏 𝜋 𝜋
1 − cos 2𝑥
𝑉𝑜𝑙𝑢𝑚𝑒 = 𝜋 ∫ [𝑓(𝑥)]2 𝑑𝑥 = 𝜋 ∫ [sin 𝑥]2 𝑑𝑥 = 𝜋 ∫ ( ) 𝑑𝑥
𝑎 0 0 2
Integrate and simplify:
𝑥 sin 2𝑥 𝜋 𝜋 sin 2𝜋 𝜋 0 𝜋2
= 𝜋[ − ] = 𝜋( − ) = 𝜋( − ) =
2 4 0 2 4 2 4 2
Get all the files at: https://bit.ly/azizhandouts

Example 1.195
Find the volume of a sphere with radius 𝑟.

The equation of a circle:


𝑦 2 + 𝑥 2 = 𝑟 2 ⇒ 𝑦 = ±√𝑟 2 − 𝑥 2
Because of the ±, this is not a function. However, we take only the positive value,
we get a semicircle with radius 𝑟 centered at the origin:
𝑅(𝑥) = 𝑦 = √𝑟 2 − 𝑥 2 ⇒ [𝑅(𝑥)]2 = 𝑦 2 = 𝑟 2 − 𝑥 2
Evaluate the integral below:
𝑟
2
∫ 𝜋(𝑅(𝑥)) 𝑑𝑥
−𝑟
Since this is an even function, the integral from −𝑟 to 𝑟 is twice the integral from 0 to 𝑟:
𝑟 𝑟
𝑥3 𝑟3 4
2𝜋 ∫ (𝑟 − 𝑥 ) 𝑑𝑥 = 2𝜋 [𝑟 𝑥 − ] = 2𝜋 (𝑟 3 − ) = 𝜋𝑟 3
2 2 2
0 3 0 3 3

Example 1.196
A chef has cheese in a spherical shape with a radius of 2 inches. He asks his assistant to get him one-fourth of
the cheese. The assistant places the cheese on the countertop, slices it into two parts using a cut parallel to the
countertop and 3 inches above the countertop. He brings the smaller part to the chef. What is the ratio of the
cheese that the chef got to the cheese that he expected.

Consider a vertical cross section of the cheese ball (spherical ball), and introduce a coordinate system with
origin at the center of the sphere.

Then:
𝑅𝑎𝑑𝑖𝑢𝑠 𝑜𝑓 𝑆𝑝ℎ𝑒𝑟𝑒 = 2
𝐻𝑒𝑖𝑔ℎ𝑡 = ℎ

Using Pythagoras,
𝑅𝑎𝑑𝑖𝑢𝑠 𝑜𝑓 𝐻𝑜𝑟𝑖𝑧𝑜𝑛𝑡𝑎𝑙 𝐶𝑟𝑜𝑠𝑠𝑆𝑒𝑐𝑡𝑖𝑜𝑛 = √4 − ℎ2

The formula for volume using a disc is:


Get all the files at: https://bit.ly/azizhandouts

𝑏
𝑉𝑜𝑙𝑢𝑚𝑒 = 𝜋 ∫ [𝑓(𝑥)]2 𝑑𝑥
𝑎

We will integrate with respect to ℎ:


2 2 2
2 ℎ3
𝑉𝑜𝑙𝑢𝑚𝑒 = 𝜋 ∫ (√4 − ℎ2 ) 𝑑ℎ = 𝜋 ∫ (4 − ℎ2 ) 𝑑ℎ = 𝜋 [4ℎ − ]
1 1 3 1
8 1 7 5
= 𝜋 [(8 − ) − (4 − )] = 𝜋 (4 − ) = 𝜋
3 3 3 3

5 5
3𝜋 = 3 =5
1 4 3) 8 8

4 3 𝜋(2 3

Example 1.197
A hemispherical bowl is filled with water to one half of its depth. Find the ratio of water to air in the bowl.

Since all bowls are similar, consider a bowl with radius 10. Consider a vertical cross section of the bowl, and
introduce a coordinate system with origin at the center of the sphere.
𝑅𝑎𝑑𝑖𝑢𝑠 𝑜𝑓 𝑆𝑝ℎ𝑒𝑟𝑒 = 10, 𝐻𝑒𝑖𝑔ℎ𝑡 = ℎ
Using Pythagoras,
𝑅𝑎𝑑𝑖𝑢𝑠 𝑜𝑓 𝐻𝑜𝑟𝑖𝑧𝑜𝑛𝑡𝑎𝑙 𝐶𝑟𝑜𝑠𝑠𝑆𝑒𝑐𝑡𝑖𝑜𝑛 = √100 − ℎ2
Substitute √100 − ℎ2 in
𝑏
𝑉𝑜𝑙𝑢𝑚𝑒 = 𝜋 ∫ [𝑅(𝑥)]2 𝑑𝑥
𝑎
To get:
−5 2 −5
𝑉𝑜𝑙𝑢𝑚𝑒 = 𝜋 ∫ (√100 − ℎ2 ) 𝑑ℎ = 𝜋 ∫ (100 − ℎ2 ) 𝑑ℎ
−10 −10
Integrate:
−5
ℎ3
= 𝜋 [100ℎ − ]
3 −10
Simplify:
−125 −1000
= 𝜋 [(−500 − ) − (−1000 − )]
3 3
125 1000 875 1500 875 625
= 𝜋 [−500 + + 1000 − ] = 𝜋 [500 − ] = 𝜋[ − ] = 𝜋[ ]
3 3 3 3 3 3
The ratio of the volume of the filled bowl to that of the entire bowl is:
625 625 625
𝜋[ 3 ] 625 3 5
= 3 = 3 = × =
1 4 3 2 2000 3 2000 16
× 𝜋𝑟 (10)3
2 3 3 3

625
𝜋[ 3 ] 5
=
1 4 3 16
2 × 3 𝜋𝑟
625 5 1 4 3
𝜋[ ]= × × 𝜋𝑟
3 16 2 3
Get all the files at: https://bit.ly/azizhandouts

625 5 2000
𝜋[ ]= × 𝜋
3 16 3

Example 1.198
𝑥2 𝑦2
Find the volume of the solid generated by rotating the ellipse 𝑎2 + 𝑏2 = 1 around the 𝑥 −axis.

The limits of integration are from −𝑎 to 𝑎. Since the function is symmetric, we


instead integrate from 0 to 𝑎, and double the answer.

𝑥2
Substitute [𝑓(𝑥)]2 = 𝑦 2 = 𝑏 2 (1 − ) in the formula for
𝑎2
𝑏 𝑎
𝑥2
𝑉𝑜𝑙𝑢𝑚𝑒 = 𝜋 ∫ [𝑓(𝑥)]2 𝑑𝑥 = 2𝜋 ∫ 𝑏 2 (1 − ) 𝑑𝑥
𝑎 0 𝑎2

Factor out 𝑏 2 and carry out the integration:


𝑎
𝑥3
= 2𝜋𝑏 2 [𝑥 − ]
3𝑎2 0

Substitute the limits of integration and simplify:


𝑎3 𝑎 2 4
= 2𝜋𝑏 (𝑎 − 2 ) = 2𝜋𝑏 2 (𝑎 − ) = 2𝜋𝑏 2 ( 𝑎) = 𝜋𝑎𝑏 2
2
3𝑎 3 3 3

C. Washer Method

1.199: Washer Method


The volume of the solid between 𝑦 = 𝑓(𝑥) and 𝑦 = 𝑔(𝑥) from 𝑥 = 𝑎 to 𝑥 = 𝑏 is:
𝑏
𝑉𝑜𝑙𝑢𝑚𝑒 = 𝜋 ∫ [𝑅(𝑥)]2 − [𝑟(𝑥)]2 𝑑𝑥
𝑎

Example 1.200
3
Find, but do not evaluate, an integral expression for the volume of the solid between 𝑦 = 1 and 𝑦 = √𝑥 for
1 ≤ 𝑥 ≤ 8.

The limits of integration are:


𝑥 = 1, 𝑥 = 8

8 8 8
3 2 3 2
𝑉𝑜𝑙𝑢𝑚𝑒 = 𝜋 ∫ [ √𝑥 ] − [1]2 𝑑𝑥 = 𝜋 (∫ [ √𝑥 ] 𝑑𝑥 − ∫ 1 𝑑𝑥 )
1 1 1

Example 1.201
Find, in terms of 𝜋, the volume of the solid obtained by rotating, around the 𝑥-axis, the area between 𝑓(𝑥) =
sin(𝑥) + 2 and 𝑔(𝑥) = 1 for 0 ≤ 𝑥 ≤ 𝜋.

𝑏
𝑉𝑜𝑙𝑢𝑚𝑒 = 𝜋 ∫ [𝑅(𝑥)]2 − [𝑟(𝑥)]2 𝑑𝑥
𝑎
Substitute 𝑅(𝑥) = sin(𝑥) + 2, 𝑟(𝑥) = 1:
Get all the files at: https://bit.ly/azizhandouts

𝑏
𝜋 ∫ [sin(𝑥) + 2]2 − [1]2 𝑑𝑥
𝑎

1.202: Revolving around a general function


A solid generated by revolving the curve 𝑦 = 𝑓(𝑥) around the function 𝑦 = 𝑔(𝑥) from 𝑥 = 𝑎 to 𝑥 = 𝑏 has
volume given by:
𝑏 𝑏
𝑉𝑜𝑙𝑢𝑚𝑒 = ∫ 𝜋[𝑅(𝑥)]2 𝑑𝑥 = 𝜋 ∫ [𝑓(𝑥) − 𝑔(𝑥)]2 𝑑𝑥
𝑎 𝑎

Example 1.203
3
Find, in terms of 𝜋, the volume of the solid generated by revolving the curve 𝑦 = √𝑥 , 1 ≤ 𝑥 ≤ 8 around the
line 𝑦 = 1.

The limits of integration are:


𝑥 = 1, 𝑥 = 8

𝑏 8
3 2
𝑉𝑜𝑙𝑢𝑚𝑒 = 𝜋 ∫ [𝑓(𝑥) − 𝑔(𝑥)]2 𝑑𝑥 = 𝜋 ∫ [ √𝑥 − 1] 𝑑𝑥
𝑎 1

Example 1.204
1
The curve 𝑓(𝑥) = sin 𝑥 is rotated around 𝑔(𝑥) = 2. Find, in terms of 𝜋, the volume of the solid between 𝑓(𝑥)
and 𝑔(𝑥) for 0 ≤ 𝑥 ≤ 𝜋.

𝑏 𝜋
12
𝜋 ∫ [𝑓(𝑥) − 𝑔(𝑥)]2 𝑑𝑥 = 𝜋 ∫ [sin(𝑥) − ] 𝑑𝑥
𝑎 0 2

D. Rotations around the 𝒚-axis

1.205: Disc Method: Revolving around the 𝒚 − 𝒂𝒙𝒊𝒔


A solid generated by revolving the curve 𝑥 = 𝑓(𝑦) around the 𝑦 − 𝑎𝑥𝑖𝑠 from 𝑦 = 𝑎 to 𝑦 = 𝑏 has volume given
by:
𝑏
𝑉𝑜𝑙𝑢𝑚𝑒 = 𝜋 ∫ [𝑓(𝑦)]2 𝑑𝑦
𝑎

Example 1.206
Find, in terms of 𝜋, the volume of the solid generated by revolving the curve 𝑦 = 𝑥 2 , 1 ≤ 𝑦 ≤ 4 around the
𝑦 −axis.

Take circular cross sections perpendicular to the 𝑦-axis.


Substitute 𝑎 = 1, 𝑏 = 4, 𝑥 2 = [𝑓(𝑦)]2 = 𝑦 in the formula for the volume of a solid rotated about the 𝑦-axis:
Get all the files at: https://bit.ly/azizhandouts

𝑏 4
𝑉𝑜𝑙𝑢𝑚𝑒 = 𝜋 ∫ [𝑓(𝑦)]2 𝑑𝑦 = 𝜋 ∫ 𝑦 2 𝑑𝑦 =
𝑎 1

Decisions

Rotate Integrate
Intersection Limits
about Along

𝑥-axis Disc 𝑥 Upper Limit

𝑦-axis Washer 𝑦 Lower Limit

Other

1.10 Volumes with Cylindrical Shells


A. Cylindrical Shells

Example 1.207
Setup an integral to find the volume of the solid of revolution formed by revolving the region bounded by 𝑥 =
3𝑦 + 5 and 𝑥 = 𝑦 2 + 1 about 𝑦 = 6

Use a change of variable. Let:


𝑋 = 𝑦, 𝑌 = 𝑥 ⇒ 𝑅: 𝑌 = 3𝑋 + 5, 𝑌 = 𝑋 2 + 1
The limits of integration are:
3𝑋 + 5 = 𝑋 2 + 1 ⇒ 𝑋 ∈ {−1,4}

Hence, the integral is:


4
[(3𝑋 + 5) − (𝑋 2 + 1)] 𝑑𝑋
(6 − 𝑋) ⏟
∫ ⏟
−1 𝑆ℎ𝑒𝑙𝑙 𝑆ℎ𝑒𝑙𝑙
𝑅𝑎𝑑𝑖𝑢𝑠 𝐻𝑒𝑖𝑔ℎ𝑡

Change back to the original variable:


4
⏟ + 5) − (𝑦 2 + 1)] 𝑑𝑦
⏟ − 𝑦) [(3𝑦
∫ (6
−1 𝑆ℎ𝑒𝑙𝑙 𝑆ℎ𝑒𝑙𝑙
𝑅𝑎𝑑𝑖𝑢𝑠 𝐻𝑒𝑖𝑔ℎ𝑡

Example 1.208
Get all the files at: https://bit.ly/azizhandouts

Determine the volume of the solid generated by revolving the triangle with vertices at (0,0), (2,2) and (3,0)
around the 𝑥-axis using the:
A. Disc/washer method
B. Shell Method

Part A: Disc/Washer Method


The volume is:
2 3
𝜋 ∫ 𝑅12 𝑑𝑥 + 𝜋 ∫ 𝑅22 𝑑𝑥
0 2
Substitute 𝑅1 = 𝑦 = 𝑥, 𝑅2 = 𝑦 = 6 − 2𝑥:
2 3
𝜋 ∫ 𝑥 2 𝑑𝑥 + 𝜋 ∫ (6 − 2𝑥)2 𝑑𝑥
0 2

Part B: Shell Method


𝑦
𝑦 = 6 − 2𝑥1 ⇒ 𝑥1 = 3 −
2
𝑥2 = 𝑦
𝑦 3𝑦
𝑥1 − 𝑥2 = 3 − − 𝑦 = 3 −
2 2

The volume is then:


2 2
3𝑦
2𝜋 ∫ 𝑦
⏟ 3− 𝑑𝑦 = 2𝜋 ∫ 6𝑦 − 3𝑦 2 𝑑𝑦
0
⏟ 2 0
𝑆ℎ𝑒𝑙𝑙
𝑅𝑎𝑑𝑖𝑢𝑠 𝑆ℎ𝑒𝑙𝑙
𝐻𝑒𝑖𝑔ℎ𝑡

1.11 Kinematics, Centroids, & Biology


A. Kinematics

1.209: Acceleration, Velocity and Displacement


Displacement is the integral of velocity with respect to time
𝑑𝑠
𝑠=∫ 𝑑𝑡 = ∫ 𝑣 𝑑𝑡 + 𝐶
𝑑𝑡

Velocity is the integral of acceleration with respect to time


𝑑𝑠 𝑑𝑣
= 𝑣=∫ 𝑑𝑡 = ∫ 𝑎 𝑑𝑡 + 𝐶
𝑑𝑡 𝑑𝑡

Example 1.210
A. A car starting from rest goes in the positive direction in a straight line with a constant acceleration of
𝑚2
3 𝑠
. Find the distance travelled by the car in 30 seconds.
𝑚2
B. An object with an initial velocity 𝑣0 goes in a straight line with a constant acceleration of 𝑎 𝑠
. If the
initial displacement is 𝑠0 , find the displacement function of the car.

Part A
𝑣(𝑡) = ∫ 3 𝑑𝑡 = 3𝑡 + 𝑣0 = 3𝑡
Since the car is starting from rest:
𝐼𝑛𝑖𝑡𝑖𝑎𝑙 𝑉𝑒𝑙𝑜𝑐𝑖𝑡𝑦 = 𝑣0 = 0 Let the position of the car at time 𝑡 = 0 be 𝑥 = 0.
Get all the files at: https://bit.ly/azizhandouts

𝐼𝑛𝑖𝑡𝑖𝑎𝑙 𝑑𝑖𝑠𝑝𝑙𝑎𝑐𝑒𝑚𝑒𝑛𝑡 = 𝑠0 = 0 𝐼𝑛𝑖𝑡𝑖𝑎𝑙 𝑉𝑒𝑙𝑜𝑐𝑖𝑡𝑦 = 𝑣0


3𝑡 2 3𝑡 2
𝑠(𝑡) = ∫ 3𝑡 𝑑𝑡 = + 𝑠0 = 𝑣(𝑡) = ∫ 𝑎 𝑑𝑡 = 𝑎𝑡 + 𝑣0
2 2
To find the distance travelled by the car in 30
seconds, evaluate: 𝐼𝑛𝑖𝑡𝑖𝑎𝑙 𝑑𝑖𝑠𝑝𝑙𝑎𝑐𝑒𝑚𝑒𝑛𝑡 = 𝑠0
3(30)2 𝑎𝑡 2
𝑠(30) = = 1350 𝑚 𝑠(𝑡) = ∫(𝑎𝑡 + 𝑣0 ) 𝑑𝑡 = + 𝑣0 𝑡 + 𝑠0
2 2
Part B

Example 1.211
A particle moves on the 𝑥-axis with velocity at time 𝑡 ≥ 0 given by 𝑣 = 𝑡 2 − 9𝑡 + 20. Find the maximum
possible difference between the displacement and the distance travelled by the particle.

We first check when the velocity is negative:


𝑡 2 − 9𝑡 + 20 < 0 ⇒ (𝑡 − 4)(𝑡 − 5) < 0 ⇒ 𝑡 ∈ (4,5)

If velocity does not change sign, distance and displacement will increase at the same rate.

The distance is the length of path travelled. Hence, we will need to take the absolute value of the integral.
Divide the integral into intervals based on 𝑡 ∈ (4,5):
4 5 𝑛
𝑠 = |∫ 𝑣(𝑡) 𝑑𝑡| + |∫ 𝑣(𝑡) 𝑑𝑡| + |∫ 𝑣(𝑡) 𝑑𝑡|
0 4 5
Velocity is positive in the first and the third intervals, and negative in the second interval. Hence, the above
expression becomes:
4 5 𝑛
= ∫ 𝑣(𝑡) 𝑑𝑡 − ∫ 𝑣(𝑡) 𝑑𝑡 + ∫ 𝑣(𝑡) 𝑑𝑡
0 4 5
The displacement is:
4 5 𝑛
𝑑 = ∫ 𝑣(𝑡) 𝑑𝑡 + ∫ 𝑣(𝑡) 𝑑𝑡 + ∫ 𝑣(𝑡) 𝑑𝑡
0 4 5
The difference between the two is maximum for 𝑡 ≥ 5:
5
𝑠 − 𝑑 = −2 ∫ 𝑣(𝑡)
4

Example 1.212
1
A particle moves on the x-axis with displacement function 𝑠(𝑥) = . Show that the velocity of the particle
√1+3𝑥
is never constant.

B. Centroid

1.213: Centroid
1 1
𝑥̅ = ∫ 𝑥̃ 𝑑𝐴, 𝑦̅ = ∫ 𝑦̃ 𝑑𝐴
𝐴 𝐴

Example 1.214
Determine the centroid of a quarter circle with center at the origin and radius 𝑅.
Get all the files at: https://bit.ly/azizhandouts

1
𝑦̅ = ∫ 𝑦̃ 𝑑𝐴
𝐴

𝑦 𝜋𝑟 2
Substitute 𝑦̃ = , 𝑑𝐴 = 𝑦 𝑑𝑥, 𝐴 = :
2 4
1 𝑦 2
= 2 ∫ ( ) 𝑦 𝑑𝑥 = ∫ 𝑦 2 𝑑𝑥
𝜋𝑅 2 𝜋𝑅 2
4

Substitute 𝑥 2 + 𝑦 2 = 𝑅 2 ⇒ 𝑦 2 = 𝑅 2 − 𝑥 2 , and integrate:


𝑅
2 2 2)
2 2
𝑥3
= ∫(𝑅 − 𝑥 𝑑𝑥 = [𝑅 𝑥 − ]
𝜋𝑅 2 𝜋𝑅 2 3 0

Substitute the limits of integration, and simplify:


2 3
𝑅3 2 2𝑅3 4𝑅
= (𝑅 − ) = ( ) =
𝜋𝑅 2 3 𝜋𝑅 2 3 3𝜋

C. Biology

Example 1.215
Suppose that the population of fish in a particular lake 𝑡 years after 1 Jan 2000 is given by
𝑃(𝑡) = 𝑡 3 + 3𝑡 2 + 25, 0 ≤ 𝑡 ≤ 6
𝑑𝑃
A. What is the meaning of 𝑃′(𝑡) = ?
𝑑𝑡
B. What is the population of the lake at the end of 2003?
C. At the beginning of the year 2005, what is the rate of change of the population of fish?

Part A
𝑑𝑃
𝑟𝑒𝑝𝑟𝑒𝑠𝑒𝑛𝑡𝑠 𝑟𝑎𝑡𝑒 𝑜𝑓 𝑐ℎ𝑎𝑛𝑔𝑒 𝑜𝑓 𝑃 𝑤𝑖𝑡ℎ 𝑟𝑒𝑠𝑝𝑒𝑐𝑡 𝑡𝑜 𝑐ℎ𝑎𝑛𝑔𝑒 𝑖𝑛 𝑡.
𝑑𝑡

Part B
𝑃(4) = 43 + 3(42 ) + 25 = 64 + 48 + 25 = 137

Part C
𝑓𝑖𝑠ℎ
𝑃′(𝑡) = 3𝑡 2 + 6𝑡 ⇒ 𝑃(5) = 3(52 ) + 6(5) = 105
𝑦𝑒𝑎𝑟

Example 1.216
In Orange County, the population at time 𝑡 = 0 is 𝑝.
𝐵𝑖𝑟𝑡ℎ 𝑟𝑎𝑡𝑒 = 𝑏(𝑡)
𝐷𝑒𝑎𝑡ℎ 𝑟𝑎𝑡𝑒 = 𝑑(𝑡)
where 𝑡 represents time in years.
A. What is the interpretation of 𝑐(𝑡) = 𝑏(𝑡) − 𝑑(𝑡)
𝑡
B. What is the interpretation of 𝑝 + ∫0 𝑐(𝑡) 𝑑𝑡?

Part A
Part B
Get all the files at: https://bit.ly/azizhandouts

𝑆𝑡𝑎𝑟𝑡𝑖𝑛𝑔 𝑝𝑜𝑝𝑢𝑙𝑎𝑡𝑖𝑜𝑛 = 𝑝
𝑡
𝐶ℎ𝑎𝑛𝑔𝑒 𝑖𝑛 𝑝𝑜𝑝𝑢𝑙𝑎𝑡𝑖𝑜𝑛 𝑓𝑟𝑜𝑚 𝑡 = 0 𝑡𝑜 𝑡 = 𝑡: ∫ 𝑐(𝑡)
0
𝑡
𝑝 + ∫ 𝑐(𝑡) = 𝑃𝑜𝑝𝑢𝑙𝑎𝑡𝑖𝑜𝑛 𝑎𝑡 𝑡𝑖𝑚𝑒 𝑡
0

Example 1.217
In Orange County
𝑃𝑜𝑝𝑢𝑙𝑎𝑡𝑖𝑜𝑛 𝑎𝑡 𝑡𝑖𝑚𝑒 𝑡 = 0 𝑖𝑠 4
𝐶ℎ𝑎𝑛𝑔𝑒 𝑖𝑛 𝑝𝑜𝑝𝑢𝑙𝑎𝑡𝑖𝑜𝑛 𝑎𝑡 𝑡𝑖𝑚𝑒 𝑡 = 𝑐(𝑡) = 2𝑡 + 4
𝑃𝑜𝑝𝑢𝑙𝑎𝑡𝑖𝑜𝑛 𝑎𝑡 𝑡𝑖𝑚𝑒 𝑡 = 𝑃(𝑡)
A. Find ∫ 𝑐(𝑡) 𝑑𝑡 and interpret it.
B. Find 𝑃(𝑡)
C. Determine the increase in population from 𝑡 = 0 to 𝑡 = 3 by:
a. Using 𝑃(𝑡)
𝑏
b. Solving a definite integral of the form ∫𝑎 𝑐(𝑡) 𝑑𝑡

Part A
∫ 𝑐(𝑡) 𝑑𝑡 = ∫ 2𝑡 + 4 𝑑𝑡 = 𝑡 2 + 4𝑡 + 𝐶

Part B
𝑃(0) = 4 ⇒ 𝐶 = 4
𝑃(𝑡) = 𝑡 2 + 4𝑡 + 4 = (𝑡 + 2)2
Part B
𝐸𝑛𝑑𝑖𝑛𝑔 𝑃𝑜𝑝𝑢𝑙𝑎𝑡𝑖𝑜𝑛 − 𝑆𝑡𝑎𝑟𝑡𝑖𝑛𝑔 𝑃𝑜𝑝𝑢𝑙𝑎𝑡𝑖𝑜𝑛 = 𝑃(3) − 𝑃(0) = (3 + 2)2 − (0 + 2)2 = 25 − 4 = 21

Part C
𝑏 3
∫ 𝑐(𝑡) 𝑑𝑡 = ∫ 2𝑡 + 4 𝑑𝑡 = [𝑡 2 + 4𝑡]𝑡=3
𝑡=0 = 21 − 0 = 21
𝑎 0
Get all the files at: https://bit.ly/azizhandouts

2. MORE INTEGRATION
2.1 Integration by Parts
A. Background
Running the chain rule in reverse gives 𝑢 substitution. Correspondingly, integration by parts is based on the
product rule for differentiation.

2.1: Integration by Parts


∫ 𝑢 ∙ 𝑑𝑣 = 𝑢𝑣 − ∫ 𝑣 ∙ 𝑑𝑢

Start with the formula for the derivative of a product:


(𝑢𝑣)′ = 𝑢𝑣 ′ + 𝑣𝑢′

Integrate both sides:


∫(𝑢𝑣)′ = ∫ 𝑢𝑣 ′ + 𝑣𝑢′
On the LHS, integration and differentiation cancel.
On the RHS, separate out the two integrals:
𝑢𝑣 = ∫ 𝑢𝑣 ′ + ∫ 𝑣𝑢′
Rearrange:
∫ 𝑢𝑣 ′ = 𝑢𝑣 − ∫ 𝑣𝑢′
Rewrite in differentials:
∫ 𝑢 𝑑𝑣 = 𝑢𝑣 − ∫ 𝑣 𝑑𝑢

2.2: Choice of 𝒖 and 𝒗


We want to choose
𝑢 𝑠𝑜 𝑡ℎ𝑎𝑡 𝑖𝑡𝑠 𝑑𝑒𝑟𝑖𝑣𝑎𝑡𝑖𝑣𝑒 𝑖𝑠 𝑒𝑎𝑠𝑖𝑒𝑟
𝑑𝑣 𝑠𝑜 𝑡ℎ𝑎𝑡 𝑖𝑡𝑠 𝑖𝑛𝑡𝑒𝑔𝑟𝑎𝑙 𝑖𝑠 𝑒𝑎𝑠𝑖𝑒𝑟

➢ We are going to differentiate 𝑢 to find 𝑑𝑢 so we would like its derivative to be easier to work with.
➢ We are going to integrate 𝑑𝑣 so we would to choose 𝑑𝑣 so that its integral is easier.

2.3: ILATE
Order of preference for 𝑢.

𝐼 = 𝐼𝑛𝑣𝑒𝑟𝑠𝑒
𝐿 = 𝐿𝑜𝑔𝑎𝑟𝑖𝑡ℎ𝑚𝑖𝑐
𝐴 = 𝐴𝑙𝑔𝑒𝑏𝑟𝑎𝑖𝑐
𝑇 = 𝑇𝑟𝑖𝑔𝑜𝑛𝑜𝑚𝑒𝑡𝑟𝑖𝑐
𝐸 = 𝐸𝑥𝑝𝑜𝑛𝑒𝑛𝑡𝑖𝑎𝑙
B. Logarithmic Integrals

Example 2.4
Evaluate
Get all the files at: https://bit.ly/azizhandouts

∫ ln 𝑥 𝑑𝑥

We want to choose 𝑢 so that the derivative is easier. Taking 𝑢 = ln 𝑥 converts a logarithmic function to an
algebraic function.
𝑑 1
(ln 𝑥) =

𝑑𝑥 𝑳𝒐𝒈𝒂𝒓𝒊𝒕𝒉𝒎𝒊𝒄 ⏟𝑥
𝑨𝒍𝒈𝒆𝒃𝒓𝒂𝒊𝒄

There is no 𝑣 that we can seemingly take. However, we can always multiply by 1. Take 𝑑𝑣 = 1 ∙ 𝑑𝑥:
𝑢 = ln 𝑥 , 𝑑𝑣 = 1 ∙ 𝑑𝑥
1
𝑑𝑢 = 𝑑𝑥, 𝑣=𝑥
𝑥

Substitute in ∫ 𝑢 𝑑𝑣 = 𝑢𝑣 − ∫ 𝑣 𝑑𝑢:
1
∫ ln 𝑥 𝑑𝑥 = 𝑥 ∙ ln 𝑥 − ∫ 𝑥 𝑑𝑥
𝑥
= 𝑥 ∙ ln 𝑥 − ∫ 1 ∙ 𝑑𝑥
= 𝑥 ∙ ln 𝑥 − 𝑥 + 𝐶

Example 2.5
Evaluate
∫ 𝑥 3 ln 𝑥 𝑑𝑥

𝑢 = ln 𝑥 , 𝑑𝑣 = 𝑥 3 𝑑𝑥
1 𝑥4
𝑑𝑢 = 𝑑𝑥, 𝑣=
𝑥 4

Substitute ∫ 𝑢 𝑑𝑣 = 𝑢𝑣 − ∫ 𝑣 𝑑𝑢:
𝑥4 𝑥 4 𝑑𝑥
ln 𝑥 ( ) − ∫( )( )
4 4 𝑥
Simplify:
𝑥 4 ln 𝑥 𝑥3
= − ∫ ( ) 𝑑𝑥
4 4
Integrate:
𝑥 4 ln 𝑥 𝑥 4
− +𝐶
4 16

Example 2.6
∫ 𝑥 𝑛 ln 𝑥 𝑑𝑥
Evaluate the above. Ensure you take all special cases into consideration.4

𝑢 = ln 𝑥 , 𝑑𝑣 = 𝑥 𝑛 𝑑𝑥

4
You can be asked to integrate in terms of unknown constants. This is equivalent to proving a formula. Use this example to
improve your manipulation skills. (Don’t Memorize).
Get all the files at: https://bit.ly/azizhandouts

1 𝑥 𝑛+1
𝑑𝑢 = 𝑑𝑥, 𝑣=
𝑥 𝑛+1

Apply ∫ 𝑢 𝑑𝑣 = 𝑢𝑣 − ∫ 𝑣 𝑑𝑢:
𝑥 𝑛+1 𝑥 𝑛+1 1
∫ 𝑥 𝑛 ln 𝑥 𝑑𝑥 = ln 𝑥 ( )−∫( ) ( 𝑑𝑥)
𝑛+1 𝑛+1 𝑥
Simplify:
𝑥 𝑛+1 ln 𝑥 𝑥𝑛
= −∫ 𝑑𝑥
𝑛+1 𝑛+1
Integrate
𝑥 𝑛+1 ln 𝑥 𝑥 𝑛+1
= − +𝐶
𝑛+1 (𝑛 + 1)2

Special Case: 𝒏 = −𝟏
But wait. The above is not applicable when 𝑛 = −1:
1 𝑥 𝑛+1
∫ 𝑥 −1 𝑑𝑥 = ∫ 𝑑𝑥 = ln 𝑥 ≠
𝑥 𝑛

Hence, we need to find this separately:


ln 𝑥 𝑢2 (ln 𝑥)2
∫ 𝑥 −1 ln 𝑥 𝑑𝑥 = ∫ 𝑑𝑥 = ∫ 𝑢 𝑑𝑢 = +𝐶 = +𝐶
𝑥 2 2
Where we made a 𝑢-substitution using
1
𝑢 = ln 𝑥 ⇒ 𝑑𝑢 = 𝑑𝑥
𝑥

Example 2.7
Evaluate
∫ 𝑥𝑒 𝑥 𝑑𝑥

𝑢 = 𝑥, 𝑑𝑣 = 𝑒 𝑥 𝑑𝑥
𝑑𝑢 = 𝑑𝑥, 𝑣 = 𝑒𝑥

Apply ∫ 𝑢 𝑑𝑣 = 𝑢𝑣 − ∫ 𝑣 𝑑𝑢:
∫ 𝑥𝑒 𝑥 𝑑𝑥 = 𝑥 ∙ 𝑒 𝑥 − ∫ 𝑒 𝑥 𝑑𝑥 = 𝑥 ∙ 𝑒 𝑥 − 𝑒 𝑥 + 𝐶

Example 2.8
Evaluate
∫ 𝑥𝑒 −𝑥 𝑑𝑥

𝑢 = 𝑥, 𝑑𝑣 = 𝑒 −𝑥 𝑑𝑥
𝑑𝑢 = 𝑑𝑥, 𝑣 = −𝑒 −𝑥 𝑑𝑥

Apply ∫ 𝑢 𝑑𝑣 = 𝑢𝑣 − ∫ 𝑣 𝑑𝑢:
∫ 𝑥𝑒 −𝑥 𝑑𝑥 = −𝑥𝑒 −𝑥 + ∫ 𝑒 −𝑥 𝑑𝑢 = −𝑥𝑒 −𝑥 − 𝑒 −𝑥 + 𝐶
Get all the files at: https://bit.ly/azizhandouts

C. Successive Integration
It is sometimes necessary to apply integration by parts multiple times. The key here is that the integral that
you are left with should be simpler than the one that you started with. If that is not the case, you should re-
consider your strategy.

Example 2.9
Evaluate
∫ 𝑥 2 𝑒 𝑥 𝑑𝑥

Integration by Parts
𝑢 = 𝑥2, 𝑑𝑣 = 𝑒 𝑥 𝑑𝑥
𝑑𝑢 = 2𝑥 𝑑𝑥, 𝑣 = 𝑒𝑥
Substitute ∫ 𝑢 𝑑𝑣 = 𝑢𝑣 − ∫ 𝑣 𝑑𝑢:
= 𝑥 2 𝑒 𝑥 − ∫ 2𝑥𝑒 𝑥 𝑑𝑥

= 𝑥 2 𝑒 𝑥 − 2 ∫ 𝑥𝑒 𝑥 𝑑𝑥
Integration by Parts One More Time
We now need to find ∫ 𝑥𝑒 𝑥 𝑑𝑥:
𝑢 = 𝑥, 𝑑𝑣 = 𝑒 𝑥 𝑑𝑥
𝑑𝑢 = 𝑑𝑥, 𝑣 = 𝑒𝑥
Substitute ∫ 𝑢 𝑑𝑣 = 𝑢𝑣 − ∫ 𝑣 𝑑𝑢:
∫ 𝑥𝑒 𝑥 𝑑𝑥 = 𝑥𝑒 𝑥 − ∫ 𝑒 𝑥 𝑑𝑥 = 𝑥𝑒 𝑥 − 𝑒 𝑥 + 𝐶

And, hence the final answer is:


= 𝑥 2 𝑒 𝑥 − 2(𝑥𝑒 𝑥 − 𝑒 𝑥 ) + 𝐶
= 𝑥 2 𝑒 𝑥 − 2𝑥𝑒 𝑥 + 2𝑒 𝑥 + 𝐶

D. Tabular Integration
Successive integration by parts becomes much easier if you put it in a table.

Example 2.10
Evaluate
∫ 𝑥 2 𝑒 𝑥 𝑑𝑥

𝑢 𝑑𝑣
𝑒𝑥
+ 𝑥2 𝑒𝑥
− 2𝑥 𝑒𝑥
+ 2 𝑒𝑥

𝑥 2 𝑒 𝑥 − 2𝑥𝑒 𝑥 + 2𝑒 𝑥 + 𝐶

Example 2.11
Evaluate
Get all the files at: https://bit.ly/azizhandouts

∫ 𝑥 2 𝑒 −𝑥 𝑑𝑥

𝑢 𝑑𝑣
𝑒 −𝑥
+ 𝑥2 −𝑒 −𝑥
− 2𝑥 𝑒 −𝑥
+ 2 −𝑒 −𝑥

−𝑥 2 𝑒 −𝑥 − 2𝑥𝑒 −𝑥 − 2𝑒 −𝑥 + 𝐶

Example 2.12
Formula for integration by parts equivalent to tabular integration
∫ 𝑢𝑣 = 𝑢𝑣1 − 𝑢′ 𝑣2 + 𝑢′′ 𝑣3 − ⋯

Where
𝑢 𝑔𝑒𝑡𝑠 𝑑𝑖𝑓𝑓𝑒𝑟𝑒𝑛𝑡𝑖𝑎𝑡𝑒𝑑 𝑒𝑎𝑐ℎ 𝑡𝑖𝑚𝑒
𝑣 𝑔𝑒𝑡𝑠 𝑖𝑛𝑡𝑒𝑔𝑟𝑎𝑡𝑒𝑑 𝑒𝑎𝑐ℎ 𝑡𝑖𝑚𝑒

Stop when one of the terms becomes zero.

E. Reduction Formulas
Reduction formulas are formulas that let you “reduce” the power of an integral. In other words, they let you
replace
∫ 𝑓(𝑥) → ∫ 𝑔(𝑥)
Where
𝑓(𝑥) ℎ𝑎𝑠 𝑝𝑜𝑤𝑒𝑟 𝑛
𝑔(𝑥) ℎ𝑎𝑠 𝑝𝑜𝑤𝑒𝑟 𝑚 < 𝑛

In the earlier example, we evaluated


➢ ∫ 𝑥𝑒 𝑥 𝑑𝑥 using integration by parts
➢ ∫ 𝑥 2 𝑒 𝑥 𝑑𝑥 using integration by parts twice

This makes us conjecture that if we needed to evaluate ∫ 𝑥 3 𝑒 𝑥 𝑑𝑥, we could do so by using integration by
parts three times.
But rather than doing this, we prove a reduction formula that lets us evaluate ∫ 𝑥 𝑛 𝑒 𝑥 𝑑𝑥 in terms of the
simpler integral given by ∫ 𝑛𝑥 𝑛−1 𝑒 𝑥 𝑑𝑥

2.13: Reduction Formula


∫ 𝑥 𝑛 𝑒 𝑥 𝑑𝑥 = 𝑥 𝑛 𝑒 𝑥 − ∫ 𝑛𝑥 𝑛−1 𝑒 𝑥 𝑑𝑥, 𝑛 > 0, 𝑛 ∈ ℕ

𝑢 = 𝑥𝑛, 𝑑𝑣 = 𝑒 𝑥 𝑑𝑥
𝑑𝑢 = 𝑛𝑥 𝑛−1 𝑑𝑥, 𝑣 = 𝑒𝑥
Substitute ∫ 𝑢 𝑑𝑣 = 𝑢𝑣 − ∫ 𝑣 𝑑𝑢:
∫ 𝑥 𝑛 𝑒 𝑥 𝑑𝑥 = 𝑥 𝑛 𝑒 𝑥 − ∫ 𝑛𝑥 𝑛−1 𝑒 𝑥 𝑑𝑥
Get all the files at: https://bit.ly/azizhandouts

Example 2.14
∫ 𝑥 5 𝑒 𝑥 𝑑𝑥

Apply the reduction formula:


∫ 𝑥 5 𝑒 𝑥 𝑑𝑥 = 𝑥 5 𝑒 𝑥 − ∫ 5𝑥 4 𝑒 𝑥 𝑑𝑥
Apply the reduction formula a second time:
= 𝑥 5 𝑒 𝑥 − 5 (𝑥 4 𝑒 𝑥 − ∫ 4𝑥 3 𝑒 𝑥 𝑑𝑥) = 𝑥 5 𝑒 𝑥 − 5𝑥 4 𝑒 𝑥 + 20 ∫ 𝑥 3 𝑒 𝑥 𝑑𝑥
Apply the reduction formula a third time:
= 𝑥 5 𝑒 𝑥 − 5𝑥 4 𝑒 𝑥 + 20 (𝑥 3 𝑒 𝑥 − ∫ 3𝑥 2 𝑒 𝑥 𝑑𝑥)

= 𝑥 5 𝑒 𝑥 − 5𝑥 4 𝑒 𝑥 + 20𝑥 3 𝑒 𝑥 − 60 ∫ 𝑥 2 𝑒 𝑥 𝑑𝑥
Apply the reduction formula a fourth time:
= 𝑥 5 𝑒 𝑥 − 5𝑥 4 𝑒 𝑥 + 20𝑥 3 𝑒 𝑥 − 60 (𝑥 2 𝑒 𝑥 − ∫ 2𝑥𝑒 𝑥 𝑑𝑥)

= 𝑥 5 𝑒 𝑥 − 5𝑥 4 𝑒 𝑥 + 20𝑥 3 𝑒 𝑥 − 60𝑥 2 𝑒 𝑥 + 120 ∫ 𝑥𝑒 𝑥 𝑑𝑥


Apply the reduction formula a last time:
= 𝑥 5 𝑒 𝑥 − 5𝑥 4 𝑒 𝑥 + 20𝑥 3 𝑒 𝑥 − 60𝑥 2 𝑒 𝑥 + 120 (𝑥𝑒 𝑥 − ∫ 𝑒 𝑥 𝑑𝑥)

= 𝑥 5 𝑒 𝑥 − 5𝑥 4 𝑒 𝑥 + 20𝑥 3 𝑒 𝑥 − 60𝑥 2 𝑒 𝑥 + 120𝑥𝑒 𝑥 − 120 ∫ 𝑒 𝑥 𝑑𝑥

Now we don’t need to apply the reduction formula, since we directly integrate (and also it is not applicable):
= 𝑥 5 𝑒 𝑥 − 5𝑥 4 𝑒 𝑥 + 20𝑥 3 𝑒 𝑥 − 60𝑥 2 𝑒 𝑥 + 120𝑥𝑒 𝑥 − 120𝑒 𝑥 + 𝐶

𝑢 𝑑𝑣
𝑒𝑥
+ 𝑥5 𝑒𝑥
− 5𝑥 4 𝑒𝑥
+ 20𝑥 3 𝑒𝑥
− 60𝑥 2 𝑒𝑥
+ 120𝑥 𝑒𝑥
− 120 𝑒𝑥

𝑥 5 𝑒 𝑥 − 5𝑥 4 𝑒 𝑥 + 20𝑥 3 𝑒 𝑥 − 60𝑥 2 𝑒 𝑥 + 120𝑥𝑒 𝑥 − 120𝑒 𝑥 + 𝐶


𝑒 𝑥 (𝑥 5 − 5𝑥 4 + 20𝑥 3 − 60𝑥 2 + 120𝑥 − 120) + 𝐶

F. Trigonometric Integrals

Example 2.15
Evaluate
∫ 𝑥 sin 𝑥 𝑑𝑥
Get all the files at: https://bit.ly/azizhandouts

𝑢 = 𝑥, 𝑑𝑣 = sin 𝑥 𝑑𝑥
𝑑𝑢 = 𝑑𝑥, 𝑣 = − cos 𝑥

Apply ∫ 𝑢 𝑑𝑣 = 𝑢𝑣 − ∫ 𝑣 𝑑𝑢:
= 𝑥(− cos 𝑥) − ∫ − cos 𝑥 𝑑𝑥

= 𝑥(− cos 𝑥) + ∫ cos 𝑥 𝑑𝑥


= −𝑥(cos 𝑥) + sin 𝑥 + 𝐶

Example 2.16
Evaluate
∫ 𝑥 2 sin 𝑥 𝑑𝑥

Integration by Parts
Choose the algebraic function for 𝑢:
𝑢 = 𝑥2, 𝑑𝑣 = sin 𝑥 𝑑𝑥
𝑑𝑢 = 2𝑥 𝑑𝑥, 𝑣 = − cos 𝑥

Apply ∫ 𝑢 𝑑𝑣 = 𝑢𝑣 − ∫ 𝑣 𝑑𝑢:
∫ 𝑥 2 sin 𝑥 𝑑𝑥 = 𝑥 2 (− cos 𝑥) − ∫ − cos 𝑥 2𝑥 𝑑𝑥 = −𝑥 2 (cos 𝑥) + ∫ cos 𝑥 2𝑥 𝑑𝑥

Apply Integration by Parts Again


∫ cos 𝑥 2𝑥 𝑑𝑥
𝑢 = 2𝑥, 𝑑𝑣 = cos 𝑥 𝑑𝑥
𝑑𝑢 = 2 𝑑𝑥, 𝑣 = sin 𝑥

Substitute ∫ 𝑢 𝑑𝑣 = 𝑢𝑣 − ∫ 𝑣 𝑑𝑢:
∫ cos 𝑥 2𝑥 𝑑𝑥 = 2𝑥 sin 𝑥 − ∫(sin 𝑥) 2 𝑑𝑥 = 2𝑥 sin 𝑥 + 2 cos 𝑥 + 𝐶

Bring everything together:


∫ 𝑥 2 sin 𝑥 𝑑𝑥 = −𝑥 2 (cos 𝑥) + 2𝑥 sin 𝑥 + 2 cos 𝑥 + 𝐶

Example 2.17
Evaluate
∫ 𝑥 2 cos 3𝑥 𝑑𝑥

Integration by Parts
Choose the algebraic function for 𝑢:
𝑢 = 𝑥2, 𝑑𝑣 = cos 3𝑥 𝑑𝑥
1
𝑑𝑢 = 2𝑥 𝑑𝑥, 𝑣 = sin 3𝑥
3
Get all the files at: https://bit.ly/azizhandouts

Apply ∫ 𝑢 𝑑𝑣 = 𝑢𝑣 − ∫ 𝑣 𝑑𝑢:
1 1
∫ 𝑥 2 cos 3𝑥 𝑑𝑥 = 𝑥 2 sin 3𝑥 − ∫ sin 3𝑥 2𝑥 𝑑𝑥
3 3

Apply Integration by Parts Again


∫ 2𝑥 sin 3𝑥 𝑑𝑥
𝑢 = 2𝑥, 𝑑𝑣 = sin 3𝑥 𝑑𝑥
1
𝑑𝑢 = 2 𝑑𝑥, 𝑣 = − cos 3𝑥
3

Substitute ∫ 𝑢 𝑑𝑣 = 𝑢𝑣 − ∫ 𝑣 𝑑𝑢:
1 2 2 2
∫ sin 3𝑥 2𝑥 𝑑𝑥 = − (2𝑥)(cos 3𝑥) + ∫ cos 3𝑥 𝑑𝑥 = − 𝑥 cos 3𝑥 + sin 3𝑥
3 3 3 9

1 2 1 2 2
𝑥 sin 3𝑥 − (− 𝑥 cos 3𝑥 + sin 3𝑥)
3 3 3 9

1 2 2
= 𝑥 2 sin 3𝑥 − 𝑥 cos 3𝑥 − sin 3𝑥
3 9 27
G. Solving for the Integral
When using integration by parts, the remaining integral can cycle and give you the original integral. This may
feel like a dead end, but you can then solve for the integral.

Example 2.18
Evaluate
∫ 𝑒 𝑥 sin 𝑥 𝑑𝑥

Choose the algebraic function for 𝑢:


𝑢 = sin 𝑥 , 𝑑𝑣 = 𝑒 𝑥 𝑑𝑥
𝑑𝑢 = cos 𝑥 𝑑𝑥, 𝑣 = 𝑒𝑥

Apply ∫ 𝑢 𝑑𝑣 = 𝑢𝑣 − ∫ 𝑣 𝑑𝑢:
𝑒 𝑥 sin 𝑥 − ∫ 𝑒 𝑥 cos 𝑥 𝑑𝑥

Use integration by parts one more time


𝑢 = cos 𝑥 , 𝑑𝑣 = 𝑒 𝑥 𝑑𝑥
𝑑𝑢 = − sin 𝑥 𝑑𝑥, 𝑣 = 𝑒𝑥
− ∫ 𝑒 𝑥 cos 𝑥 𝑑𝑥 = − [𝑒 𝑥 cos 𝑥 − ∫ 𝑒 𝑥 (− sin 𝑥)𝑑𝑥] = −𝑒 𝑥 cos 𝑥 − ∫ 𝑒 𝑥 sin 𝑥 𝑑𝑥

Substitute back into the original expression:


∫ 𝑒 𝑥 sin 𝑥 𝑑𝑥 = 𝑒 𝑥 sin 𝑥 − 𝑒 𝑥 cos 𝑥 − ∫ 𝑒 𝑥 sin 𝑥 𝑑𝑥

2 ∫ 𝑒 𝑥 sin 𝑥 𝑑𝑥 = 𝑒 𝑥 sin 𝑥 − 𝑒 𝑥 cos 𝑥


Get all the files at: https://bit.ly/azizhandouts

1
∫ 𝑒 𝑥 sin 𝑥 𝑑𝑥 = 𝑒 𝑥 (sin 𝑥 − cos 𝑥)
2

Example 2.19
Evaluate
∫ sec 3 𝜃 𝑑𝜃

𝑢 = sec 𝜃 , 𝑑𝑣 = sec 2 𝜃 𝑑𝜃
𝑑𝑢 = sec 𝜃 tan 𝜃 𝑑𝜃, 𝑣 = tan 𝜃

Apply ∫ 𝑢 𝑑𝑣 = 𝑢𝑣 − ∫ 𝑣 𝑑𝑢:
= sec 𝜃 tan 𝜃 − ∫ ⏟ sec 𝜃 tan 𝜃 𝑑𝜃 = sec 𝜃 tan 𝜃 − ∫ tan2 𝜃 sec 𝜃 𝑑𝜃
tan 𝜃 ⏟
𝑣 𝑑𝑢

Substitute tan2 𝜃 = sec 2 𝜃 − 1:


= sec 𝜃 tan 𝜃 − ∫(sec 2 𝜃 − 1) sec 𝜃 𝑑𝜃

∫ sec 3 𝜃 𝑑𝜃 = sec 𝜃 tan 𝜃 − ∫ sec 3 𝜃 𝑑𝜃 + ∫ sec 𝜃 𝑑𝜃

Substitute ∫ sec 𝜃 𝑑𝜃 = ln|sec 𝜃 + 𝑡𝑎𝑛 𝜃|, and move ∫ sec 3 𝜃 𝑑𝜃 from RHS to LHS:
2 ∫ sec 3 𝜃 𝑑𝜃 = sec 𝜃 tan 𝜃 + ln|sec 𝜃 + 𝑡𝑎𝑛 𝜃|

Solve for ∫ sec 3 𝜃 𝑑𝜃:


1
∫ sec 3 𝜃 𝑑𝜃 = (sec 𝜃 tan 𝜃 + ln|sec 𝜃 + 𝑡𝑎𝑛 𝜃|)
2

Example 2.20
𝐼 = ∫ csc 3 𝑥 𝑑𝑥

𝑢 = csc 𝑥 , 𝑑𝑣 = csc 2 𝑥 𝑑𝑥
𝑑𝑢 = − csc 𝑥 cot 𝑥 , 𝑣 = − cot 𝑥

Apply ∫ 𝑢 𝑑𝑣 = 𝑢𝑣 − ∫ 𝑣 𝑑𝑢:
𝐼 = − cot 𝑥 csc 𝑥 − ∫ csc 𝑥 cot 2 𝑥 𝑑𝑥
Substitute cot 2 𝑥 = csc 2 𝑥 − 1:
𝐼 = − cot 𝑥 csc 𝑥 − ∫ csc 𝑥 (csc 2 𝑥 − 1) 𝑑𝑥

𝐼 = − cot 𝑥 csc 𝑥 − ∫ csc 3 𝑥 𝑑𝑥 + ∫ csc 𝑥 𝑑𝑥

2𝐼 = − cot 𝑥 csc 𝑥 + ∫ csc 𝑥 𝑑𝑥


1 1
𝐼 = − cot 𝑥 csc 𝑥 − ln|csc 𝑥 + cot 𝑥|
2 2
Get all the files at: https://bit.ly/azizhandouts

Example 2.21
𝐼 = ∫ sin 6𝑥 cos 2𝑥 𝑑𝑥

𝑢 = sin 6𝑥 , 𝑑𝑣 = cos 2𝑥 𝑑𝑥
sin 2𝑥
𝑑𝑢 = 6 cos 6𝑥 𝑑𝑥, 𝑣=
2

Apply ∫ 𝑢 𝑑𝑣 = 𝑢𝑣 − ∫ 𝑣 𝑑𝑢:
sin 2𝑥 sin 2𝑥
𝐼 = (sin 6𝑥) ( ) − ∫( ) (6 cos 6𝑥)𝑑𝑥
2 2

sin 2𝑥
𝑢 = 6 cos 6𝑥 , 𝑑𝑣 = 𝑑𝑥
2
cos 2𝑥
𝑑𝑢 = −36 sin 6𝑥 𝑑𝑥, 𝑣=−
4

Apply ∫ 𝑢 𝑑𝑣 = 𝑢𝑣 − ∫ 𝑣 𝑑𝑢:
sin 2𝑥 cos 2𝑥 cos 2𝑥
𝐼 = (sin 6𝑥) ( ) − [(6 cos 6𝑥) (− ) − ∫ (− ) (−36 sin 6𝑥 𝑑𝑥)]
2 4 4

Substitute 𝐼 = ∫ sin 6𝑥 cos 2𝑥 𝑑𝑥 in the last term, and open the parenthesis:
sin 2𝑥 3
𝐼 = (sin 6𝑥) ( ) + cos 6𝑥 cos 2𝑥 + 9𝐼
2 2
1 3
𝐼 = − (sin 6𝑥 sin 2𝑥) − (cos 6𝑥 cos 2𝑥)
16 16
H. 𝒖 substitution

2.22: 𝒖 substitution with Integration by parts


Some integrals may need 𝑢 substitution prior to using integration by parts.

Example 2.23
∫ 𝑒 √𝑥 𝑑𝑥

Substitute √𝑥 = 𝑡 ⇒ 𝑥 = 𝑡 2 ⇒ 𝑑𝑥 = 2𝑡 𝑑𝑡:
∫ 𝑒 𝑡 2𝑡 𝑑𝑡 = 2 ∫ 𝑡𝑒 𝑡 𝑑𝑡

Use integration by parts:


𝑢 = 𝑡, 𝑑𝑣 = 𝑒 𝑡 𝑑𝑡
𝑑𝑢 = 𝑑𝑡, 𝑣 = 𝑒𝑡

Substitute in ∫ 𝑢 𝑑𝑣 = 𝑢𝑣 − ∫ 𝑣 𝑑𝑢:
2 [𝑡𝑒 𝑡 − ∫ 𝑒 𝑡 𝑑𝑡] = 2[𝑡𝑒 𝑡 − 𝑒 𝑡 ] + 𝐶

Change back to the original variable:


= 2(√𝑥𝑒 √𝑥 − 𝑒 √𝑥 ) + 𝐶
Get all the files at: https://bit.ly/azizhandouts

Example 2.24
𝜋2
∫ cos √𝑥 𝑑𝑥
0

Indefinite Integral
Substitute √𝑥 = 𝑡 ⇒ 𝑥 = 𝑡 2 ⇒ 𝑑𝑥 = 2𝑡 𝑑𝑡:
∫(cos 𝑡)(2𝑡)𝑑𝑡
𝑢 = 2𝑡, 𝑑𝑣 = cos 𝑡 𝑑𝑡
𝑑𝑢 = 2 𝑑𝑡, 𝑣 = sin 𝑡

Substitute in ∫ 𝑢 𝑑𝑣 = 𝑢𝑣 − ∫ 𝑣 𝑑𝑢:
2𝑡 sin 𝑡 − ∫(sin 𝑡)(2 𝑑𝑡)
= 2𝑡 sin 𝑡 + 2 cos 𝑡 + 𝐶

Change back to the original variable:


= 2 √𝑥 sin √𝑥 + 2 cos √𝑥 + 𝐶

Definite Integral
𝜋2
𝜋2
∫ cos √𝑥 𝑑𝑥 = [2 √𝑥 sin √𝑥 + 2 cos √𝑥]0
0
= (2 √𝜋 2 sin √𝜋 2 + 2 cos √𝜋 2 ) − (2 √0 sin 0 + 2 cos 0)
= (0 − 2) − (2 + 0) = −4

Example 2.25
∫ cos(ln 𝑥) 𝑑𝑥

Substitute ln 𝑥 = 𝑡 ⇒ 𝑥 = 𝑒 𝑡 ⇒ 𝑑𝑥 = 𝑒 𝑡 𝑑𝑡
𝐼 = ∫ 𝑒 𝑡 cos 𝑡 𝑑𝑡

𝑬𝒒𝒖𝒂𝒕𝒊𝒐𝒏 𝑰
Use integration by parts:
𝑢 = 𝑒𝑡, 𝑑𝑣 = cos 𝑡 𝑑𝑡
𝑡
𝑑𝑢 = 𝑒 𝑑𝑡, 𝑣 = sin 𝑡

Substitute in ∫ 𝑢 𝑑𝑣 = 𝑢𝑣 − ∫ 𝑣 𝑑𝑢:
𝐼 = 𝑒 𝑡 sin 𝑡 𝑑𝑡 − ∫ 𝑒 𝑡 sin 𝑡 𝑑𝑡

𝑬𝒒𝒖𝒂𝒕𝒊𝒐𝒏 𝑰𝑰

Use integration by parts one more time:


𝑢 = 𝑒𝑡, 𝑑𝑣 = sin 𝑡 𝑑𝑡
𝑑𝑢 = 𝑒 𝑡 𝑑𝑡, 𝑣 = − cos 𝑡

Substitute in ∫ 𝑢 𝑑𝑣 = 𝑢𝑣 − ∫ 𝑣 𝑑𝑢:
Get all the files at: https://bit.ly/azizhandouts

∫ 𝑒 𝑡 sin 𝑡 𝑑𝑡 = −𝑒 𝑡 cos 𝑡 − ∫ −𝑒 𝑡 cos 𝑡 𝑑𝑡



𝑬𝒒𝒖𝒂𝒕𝒊𝒐𝒏 𝑰𝑰𝑰

Substitute Equation II in Equation III:


𝐼 = 𝑒 𝑡 sin 𝑡 − [−𝑒 𝑡 cos 𝑡 − ∫ −𝑒 𝑡 cos 𝑡 𝑑𝑡]

𝐼 = 𝑒 𝑡 sin 𝑡 + 𝑒 𝑡 cos 𝑡 − ∫ 𝑒 𝑡 cos 𝑡 𝑑𝑡

Substitute 𝐼 = ∫ cos(𝑡) 𝑒 𝑡 𝑑𝑡 for the last term:


𝐼 = 𝑒 𝑡 sin 𝑡 𝑑𝑡 + 𝑒 𝑡 cos 𝑡 − 𝐼

Isolate 𝐼:
2𝐼 = 𝑒 𝑡 sin 𝑡 + 𝑒 𝑡 cos 𝑡

Divide both sides by 2:


1
𝐼 = 𝑒 𝑡 (sin 𝑡 + cos 𝑡) + 𝐶
2

Change back to the original variable (𝑡 = ln 𝑥):


1
𝐼 = 𝑒 ln 𝑥 (sin(ln 𝑥) + cos(ln 𝑥)) + 𝐶
2
1
𝐼 = 𝑥(sin(ln 𝑥) + cos(ln 𝑥)) + 𝐶
2
I. Cancellation

Example 2.26
1 − sin 2𝑥
∫ 𝑒 2𝑥 ( ) 𝑑𝑥 (𝑪𝑩𝑺𝑬 𝟐𝟎𝟏𝟑)
1 − cos 2𝑥

Substitute 1 − sin 2𝑥 = 1 − 2 sin 𝑥 cos 𝑥 , 1 − cos 2𝑥 = 2 sin2 𝑥:


1 − 2 sin 𝑥 cos 𝑥
∫ 𝑒 2𝑥 ( ) 𝑑𝑥
2 sin2 𝑥

Split the fraction:


1 sin 𝑥 cos 𝑥
∫ 𝑒 2𝑥 ( 2
− ) 𝑑𝑥
2sin 𝑥 sin2 𝑥

Simplify and split the integral:


1
= ∫ 𝑒 2𝑥 (csc 2 𝑥)𝑑𝑥 − ∫ 𝑒 2𝑥 (cot 𝑥)𝑑𝑥
2
𝑢 = 𝑒 2𝑥 , 𝑑𝑣 = csc 2 𝑥 𝑑𝑥
𝑑𝑢 = 2𝑒 2𝑥 𝑑𝑥, 𝑣 = − cot 𝑥

Substitute in ∫ 𝑢 𝑑𝑣 = 𝑢𝑣 − ∫ 𝑣 𝑑𝑢:
1 1
= − 𝑒 2𝑥 cot 𝑥 − ∫(− cot 𝑥)(2𝑒 2𝑥 ) − ∫ 𝑒 2𝑥 (cot 𝑥)𝑑𝑥
2 2
1
= − 𝑒 2𝑥 cot 𝑥 + 𝐶
2
Get all the files at: https://bit.ly/azizhandouts

Example 2.27
1
∫ [ln(ln 𝑥) + ] 𝑑𝑥 (𝑪𝑩𝑺𝑬 𝟐𝟎𝟏0)
(ln 𝑥)2

1
∫ ln(ln 𝑥) 𝑑𝑥 + ∫ 𝑑𝑥
(ln 𝑥)2
𝑢 = ln(ln 𝑥) , 𝑑𝑣 = 𝑑𝑥
1
𝑑𝑢 = ,𝑣 = 𝑥
𝑥 ln 𝑥

J. Inverse Trigonometric Integrals

Example 2.28
Evaluate
∫ sin−1 (cos 𝑥) 𝑑𝑥

𝜋 𝜋 𝜋 𝑥2
∫ sin−1 (sin ( − 𝑥)) 𝑑𝑥 = ∫ − 𝑥 𝑑𝑥 = 𝑥 − + 𝐶
2 2 2 2

Example 2.29
𝑥 sin−1 𝑥
∫ 𝑑𝑥 (𝑪𝑩𝑺𝑬 𝟐𝟎𝟏𝟐, 𝟐𝟎𝟏𝟔)
√1 − 𝑥 2

Let
1
sin−1 𝑥 = 𝑡 ⇒ 𝑑𝑥 = 𝑑𝑡, 𝑥 = sin 𝑡
√1 − 𝑥 2
Substitute the above:
𝑥 sin−1 𝑥
∫ 𝑑𝑥 = ∫ 𝑡 sin 𝑡 𝑑𝑡
√1 − 𝑥 2

Using ∫ 𝑥 sin 𝑥 𝑑𝑥 = −𝑥(cos 𝑥) + sin 𝑥 + 𝐶:


∫ 𝑡 sin 𝑡 𝑑𝑡 = −𝑡(cos 𝑡) + sin 𝑡 + 𝐶

Change back to the original variable (sin−1 𝑥 = 𝑡):


= −(sin−1 𝑥)(cos(sin−1 𝑥)) + sin(sin−1 𝑥) + 𝐶
= −(sin−1 𝑥)(cos(sin−1 𝑥)) + 𝑥 + 𝐶

Draw a reference triangle.


cos(sin−1 𝑥) = cos 𝑡 = √1 − 𝑥 2

The integration becomes:


= −(sin−1 𝑥)√1 − 𝑥 2 + 𝑥 + 𝐶

Example 2.30
Evaluate ∫ sin−1 𝑥 𝑑𝑥 by
Get all the files at: https://bit.ly/azizhandouts

A. Integration by parts, followed by 𝑢 substitution.


B. 𝑢 substitution, followed by integration by parts

Part A
𝑢 = sin−1 𝑥 , 𝑑𝑣 = 𝑑𝑥
1
𝑑𝑢 = 𝑑𝑥, 𝑣=𝑥
√1 − 𝑥 2

Apply ∫ 𝑢 𝑑𝑣 = 𝑢𝑣 − ∫ 𝑣 𝑑𝑢:
𝑥
∫ sin−1 𝑥 = 𝑥 sin−1 𝑥 − ∫ 𝑑𝑥
√1 − 𝑥 2
𝑥
To evaluate − ∫ 𝑑𝑥 let 𝑢 = √1 − 𝑥 2 ⇒ 𝑑𝑢 = −2𝑥 𝑑𝑥:
√1−𝑥 2
−2𝑥 1
∫ 𝑑𝑥 = ∫ 𝑑𝑢 = √𝑢 = √1 − 𝑥 2
2√1 − 𝑥 2 2√𝑢

Hence, the final answer is:


𝑥 sin−1 𝑥 + √1 − 𝑥 2 + 𝐶

Part B
To “cancel” the sin−1 𝑥 substitute 𝑡 = sin−1 𝑥 ⇒ 𝑥 = sin 𝑡 ⇒ 𝑑𝑥 = cos 𝑡 𝑑𝑡
∫ sin−1 𝑥 𝑑𝑥 = ∫ 𝑡 cos 𝑡 𝑑𝑡

𝑢 = 𝑡, 𝑑𝑣 = cos 𝑡 𝑑𝑡
𝑑𝑢 = 𝑑𝑡, 𝑣 = sin 𝑡

Apply ∫ 𝑢 𝑑𝑣 = 𝑢𝑣 − ∫ 𝑣 𝑑𝑢:
∫ 𝑡 cos 𝑡 𝑑𝑡 = 𝑡 sin 𝑡 𝑑𝑡 − ∫ sin 𝑡 𝑑𝑡 = 𝑡 sin 𝑡 𝑑𝑡 + cos 𝑡 + 𝐶
Change back to the original variable:
𝑡 = sin−1 𝑥 , sin 𝑡 = 𝑥

To find cos 𝑡, draw a reference triangle using 1 as the hypotenuse to


simplify calculations:
𝑜𝑝𝑝
sin 𝑡 = 𝑥 ⇒ = 𝑥 ⇒ 𝑜𝑝𝑝 = 𝑥 ∙ ℎ𝑦𝑝 = 𝑥
ℎ𝑦𝑝

Find 𝑦 which satisfies the relation:


𝑥 2 + 𝑦 2 = 1 ⇒ 𝑦 2 = 1 − 𝑥 2 ⇒ 𝑦 = √1 − 𝑥 2

Find cos 𝑡 from our reference triangle:


𝑎𝑑𝑗 √1 − 𝑥 2
cos 𝑡 = = = √1 − 𝑥 2
ℎ𝑦𝑝 1

Substitute everything back to get our answer in terms of 𝑥:


𝑡 sin 𝑡 𝑑𝑡 + cos 𝑡 + 𝐶 = 𝑥 sin−1 𝑥 + √1 − 𝑥 2 + 𝐶
Get all the files at: https://bit.ly/azizhandouts

Example 2.31
Show that
∫ cos −1 𝑥 = 𝑥 cos −1 𝑥 − √1 − 𝑥 2 + 𝐶

Example 2.32
Evaluate ∫ tan−1 𝑥 𝑑𝑥 by
A. Integration by parts, followed by 𝑢 substitution.
B. 𝑢 substitution, followed by integration by parts

Part A
𝑢 = tan−1 𝑥 , 𝑑𝑣 = 𝑑𝑥
1
𝑑𝑢 = 𝑑𝑥 , 𝑣=𝑥
1 + 𝑥2

Apply ∫ 𝑢 𝑑𝑣 = 𝑢𝑣 − ∫ 𝑣 𝑑𝑢:
𝑥
∫ tan−1 𝑥 𝑑𝑥 = 𝑥 ∙ tan−1 𝑥 − ∫ 𝑑𝑥
1 + 𝑥2

Substitute 𝑢 = 1 + 𝑥 2 ⇒ 𝑑𝑢 = 2𝑥 𝑑𝑥 in
𝑥 1 2𝑥 1 1 1 1
−∫ 2
𝑑𝑥 = − ∫ 2
𝑑𝑥 = − ∫ 𝑑𝑢 = − ln|𝑢| = − ln(1 + 𝑥 2 )
1+𝑥 2 1+𝑥 2 𝑢 2 2

The final answer is:


1
∫ tan−1 𝑥 𝑑𝑥 = 𝑥 ∙ tan−1 𝑥 − ln(1 + 𝑥 2 ) + 𝐶
2
Part B
Let
𝑡 = tan−1 𝑥 ⇒ 𝑥 = tan 𝑡 ⇒ 𝑑𝑥 = sec 2 𝑡

∫ tan−1 𝑥 𝑑𝑥 = ∫ 𝑡 sec 2 𝑑𝑡

Use integration by parts:


𝑢 = 𝑡, 𝑑𝑣 = sec 2 𝑑𝑡
𝑑𝑢 = 𝑑𝑡, 𝑣 = tan 𝑡
Apply ∫ 𝑢 𝑑𝑣 = 𝑢𝑣 − ∫ 𝑣 𝑑𝑢:
∫ 𝑡 sec 2 𝑑𝑡 = 𝑡 ∙ tan 𝑡 − ∫ tan 𝑡 𝑑𝑡 = 𝑡 ∙ tan 𝑡 − ln|sec 𝑡| + 𝐶

Now convert back:

Example 2.33
tan−1 √𝑥
∫ 𝑑𝑥
√𝑥
1
Substitute 𝑢 = √𝑥 ⇒ 𝑑𝑢 = 2 𝑑𝑥
√𝑥
tan−1 √𝑥
∫ 𝑑𝑥 = 2 ∫ tan−1 𝑢 𝑑𝑢
√𝑥
Get all the files at: https://bit.ly/azizhandouts

1
= 2 (𝑢 ∙ tan−1 𝑢 − ln(1 + 𝑢2 )) + 𝐶
2

Substitute 𝑢 = √𝑥:
1 2
= 2 (√𝑥 ∙ tan−1 √𝑥 − ln (1 + √𝑥 )) + 𝐶
2
= 2√𝑥 ∙ tan−1 √𝑥 − ln(1 + |𝑥|) + 𝐶

2.2 Trigonometric Integrals


A. Odd Powers of sine and cosine
➢ This section using trigonometric identities to work with integrals.
➢ Knowing the trig identities is essential for this section.
➢ Recognizing which method to apply requires significant practice.

2.34: Pythagorean Identity


sin2 𝑥 + cos 2 𝑥 = 1

We being with the most common trigonometric identity, which is the Pythagorean Identity
And see how it can be applied to 𝑢 −substitution.

Example 2.35
∫ cos5 𝑥 𝑑𝑥

Separate out a single power:


∫ cos 𝑥 cos 4 𝑥 𝑑𝑥
Rewrite cos4 𝑥 = (cos2 𝑥)2
∫ cos 𝑥 (cos2 𝑥)2 𝑑𝑥
Substitute cos2 𝑥 = 1 − sin2 𝑥:
= ∫ cos 𝑥 (1 − sin2 𝑥)2 𝑑𝑥

Now we can make a 𝑢 −substitution since


𝑢 = sin 𝑥 ⇒ 𝑑𝑢 = cos 𝑥 𝑑𝑥
Make the above substitution:
(1 − sin2 𝑥)2 ⏟
∫⏟ cos 𝑥 𝑑𝑥 = ∫(1 − 𝑢2 )2 𝑑𝑢
(𝟏−𝒖𝟐 )𝟐 𝒅𝒖

Expand using (𝑎 + 𝑏)2 = 𝑎2 + 2𝑎𝑏 + 𝑏 2 :


= ∫ 1 − 2𝑢2 + 𝑢4 𝑑𝑢

𝑢𝑛+1
This is now a polynomial in 𝑢 which can be integrated using the power rule ∫ 𝑢𝑛 = 𝑛+1
,𝑛 ≠ 1:
Get all the files at: https://bit.ly/azizhandouts

2𝑢3 𝑢5
=𝑢− + +𝐶
3 5

Change back to the original variable and substitute 𝑢 = sin 𝑥:


2 sin3 𝑥 sin5 𝑥
= sin 𝑥 − + +𝐶
3 5

2.36: Integrating ∫ 𝐜𝐨𝐬𝟐𝒏+𝟏 𝒙 , ∫ 𝐬𝐢𝐧𝟐𝒏+𝟏 𝒙, 𝒏 ∈ ℕ


To integrate an expression which contains an odd power of sin or cos:
➢ Separate out a single power.
➢ Convert the remaining powers from cos to sin or sin to cos using the identity sin2 𝑥 + cos 2 𝑥 = 1
➢ Do a 𝑢-substitution with 𝑢 = sin 𝑥 ⇒ 𝑑𝑢 = cos 𝑥 𝑑𝑥

This strategy works since


𝑑
sin 𝑥 = cos 𝑥
𝑑𝑥

Example 2.37
∫ cos3 𝑥 𝑑𝑥

Separate out a single power:


∫ cos 𝑥 cos 2 𝑥 𝑑𝑥
Substitute cos2 𝑥 = 1 − sin2 𝑥:
= ∫ cos 𝑥 (1 − sin2 𝑥) 𝑑𝑥

Substitute 𝑢 = sin 𝑥 ⇒ 𝑑𝑢 = cos 𝑥 𝑑𝑥


𝑢3
= ∫(1 − 𝑢2 )𝑑𝑢 = 𝑢 − +𝐶
3

Change back to the original variable and substitute 𝑢 = sin 𝑥:


sin3 𝑥
= sin 𝑥 − +𝐶
3

Example 2.38
𝜋
∫ sin3 𝑥
0

Substitute sin2 𝑥 = 1 − cos 2 𝑥, and separate out a power of sin 𝑥:


𝜋
∫ (1 − cos2 𝑥 ) sin 𝑥 𝑑𝑥
0
Substitute
𝑢 = cos 𝑥 ⇒ 𝑑𝑢 = − sin 𝑥 𝑑𝑥
𝑥 = 0 ⇒ 𝑢 = cos 𝑥 = cos 0 = 1
𝑥 = 𝜋 ⇒ 𝑢 = cos 𝑥 = cos 𝜋 = −1
Get all the files at: https://bit.ly/azizhandouts

−1 −1 −1
2) 2
𝑢3
∫ −(1 − 𝑢 𝑑𝑢 = ∫ 𝑢 − 1 𝑑𝑢 = [ − 𝑢]
1 1 3 1

Substitute the values:


−1 1 2 2 4
=( − (−1)) − ( − 1) = ( ) − (− ) =
3 3 3 3 3

Example 2.39
Consider the rectangle formed by the four points 𝐴 = (0,0), 𝐵 = (0,1), 𝐶 = (𝜋, 0), 𝐷 = (𝜋, 1).
A. What proportion of the area of this rectangle lies between the curves 𝑦 = sin 𝑥 and 𝑦 = sin3 𝑥.
B. Is this proportion rational or irrational?

∫ sin 𝑥 − sin3 𝑥 𝑑𝑥

∫ sin 𝑥 (1 − sin2 𝑥) 𝑑𝑥 = ∫ sin 𝑥 cos 2 𝑥 𝑑𝑥


Use the substitution 𝑢 = cos 𝑥 ⇒ 𝑑𝑢 = − sin 𝑥 𝑑𝑥:
1 1
∫ −𝑢2 𝑑𝑢 = − 𝑢3 + 𝐶 = − cos3 𝑥 + 𝐶
3 3
𝜋
1 1 1 1 1 2
[− cos 3 𝑥] = (− (−1)3 ) − (− (1)) = + =
3 0 3 3 3 3 3

Area of the Rectangle


=1×𝜋 =𝜋
Proportion
2
3𝜋

2.40: Integrating with 𝒔𝒊𝒏 and 𝒄𝒐𝒔


The strategy that we learnt can be used if both powers of sin and cos are present in the expression.
As before, separate out the function that has an odd power.

Example 2.41
∫ sin5 𝑥 cos 4 𝑥 𝑑𝑥

Separate out a single power of sin 𝑥:


∫ sin 𝑥 sin4 𝑥 cos 4 𝑥 𝑑𝑥
Substitute sin4 𝑥 = (1 − cos 2 𝑥)2
= ∫ sin 𝑥 (1 − cos2 𝑥)2 cos4 𝑥 𝑑𝑥

Substitute 𝑢 = cos 𝑥 ⇒ 𝑑𝑢 = − sin 𝑥 𝑑𝑥:


= ∫ −(1 − 𝑢2 )2 𝑢4 𝑑𝑢
Expand:
− ∫(1 − 2𝑢2 + 𝑢4 )𝑢4 𝑑𝑢
Get all the files at: https://bit.ly/azizhandouts

Multiply:
= − ∫ 𝑢4 − 2𝑢6 + 𝑢8 𝑑𝑢
Integrate:
𝑢5 2𝑢7 𝑢9
= −( − + )+𝐶
5 7 9

Change back to the original variable. Substitute 𝑢 = cos 𝑥:


cos 9 𝑥 2 cos 7 𝑥 cos5 𝑥
= −( − + )+𝐶
9 7 5

2.42: Generalizing odd power of cos (Optional)5


For natural number 𝑛, we have:
𝑛
2𝑛+1
𝑛 sin2𝑘+1 𝑥
∫ cos 𝑥 𝑑𝑥 = ∑(−1)𝑘 ( ) +𝐶
𝑘 2𝑘 + 1
𝑘=0

Separate out a single power of cos 𝑥:


∫(cos2 𝑥)𝑛 cos 𝑥 𝑑𝑥
Substitute cos2 𝑥 = 1 − sin2 𝑥:
∫(1 − sin2 𝑥)𝑛 cos 𝑥 𝑑𝑥
Substitute 𝑢 = sin 𝑥 ⇒ 𝑑𝑢 = cos 𝑥 𝑑𝑥:
∫(1 − 𝑢2 )𝑛 𝑑𝑢
Expand using the binomial theorem:
𝑛
𝑛
∫ ∑ ( ) (−1)𝑘 𝑢2𝑘 𝑑𝑢
𝑘
𝑘=0
Get the constants out of the integral sign:
𝑛
𝑛
∑(−1)𝑘 ( ) ∫ 𝑢2𝑘 𝑑𝑢
𝑘
𝑘=0
Integrate using the reverse power rule:
𝑛
𝑛 𝑢2𝑘+1
∑(−1)𝑘 ( ) +𝐶
𝑘 2𝑘 + 1
𝑘=0

B. Products of Even Powers

2.43: Strategy: Even powers


Reduce the power of the trigonometric function by using the double angle identities:
1
sin2 𝑥 = (1 − cos 2𝑥)
2
1
cos2 𝑥 = (1 + cos 2𝑥)
2

5 This needs the binomial theorem. You can refer the Note on Binomial Theorem if you need a review, or skip it without
loss of continuity.
Get all the files at: https://bit.ly/azizhandouts

Example 2.44
A. ∫ sin2 𝑥 𝑑𝑥
B. ∫ cos2 𝑥 𝑑𝑥

Part A
Use the double angle trigonometric identity to rewrite the integral:
1
∫ (1 − cos 2𝑥)𝑑𝑥
2

Split the integral:


1 cos 2𝑥
∫ 𝑑𝑥 − ∫ 𝑑𝑥
2 2
Let 𝑢 = 2𝑥 ⇒ 𝑑𝑢 = 2 𝑑𝑥:
1 cos 2𝑥 1
∫ 𝑑𝑥 − ∫ ∙ ∙⏟
2 ∙ 𝑑𝑥
2 2 2 𝑑𝑢
Make the substitution:
1 cos 𝑢
∫ 𝑑𝑥 − ∫ 𝑑𝑢
2 4
Integrate:
𝑥 sin 𝑢
− +𝐶
2 4
Change back to the original variable:
𝑥 sin 2𝑥
= − +𝐶
2 4
Part B
Similarly:
1 1 sin 2𝑥
∫ (1 + cos 2𝑥)𝑑𝑥 = (𝑥 + )+𝐶
2 2 2

Example 2.45
A. ∫ sin4 𝑥 𝑑𝑥
B. ∫ cos4 𝑥 𝑑𝑥

Since we have a fourth power, we will need to use the identity twice:

Part A
∫ sin4 𝑥 𝑑𝑥 = ∫(sin2 𝑥)2 𝑑𝑥
1
Substitute sin2 𝑥 = (1 − cos 2𝑥):
2
1 − cos 2𝑥 2
= ∫( ) 𝑑𝑥
2
Expand:
1
= ∫ 1 − 2 cos 2𝑥 + cos 2 2𝑥 𝑑𝑥
4
1
Substitute cos2 𝑥 = (1 + cos 2𝑥):
2
1 1
= ∫ 1 − 2 cos 2𝑥 + (1 + cos 4𝑥)𝑑𝑥
4 2
Simplify:
Get all the files at: https://bit.ly/azizhandouts

1 3 cos 4𝑥
= ∫ − 2 cos 2𝑥 + 𝑑𝑥
4 2 2
Integrate:
1 3 sin 4𝑥
= ( 𝑥 − sin 2𝑥 + )+𝐶
4 2 8
Simplify:
3 sin 2𝑥 sin 4𝑥
= 𝑥− + +𝐶
8 4 32

C. Reduction Formulas

D. Double Angle Identity

Example 2.46
∫ √1 + cos 2𝑥 𝑑𝑥

Substitute 1 + cos 2𝑥 = cos 2 𝑥


∫ cos 𝑥 𝑑𝑥 = − sin 𝑥 + 𝐶

E. Products of sines and cosines

2.47: Product to Sum Formulas


1
sin 𝐴 cos 𝐵 = [sin(𝐴 − 𝐵) + sin(𝐴 + 𝐵)]
2
1
sin 𝐴 sin 𝐵 = [cos(𝐴 − 𝐵) − cos(𝐴 + 𝐵)]
2
1
cos 𝐴 cos 𝐵 = [cos(𝐴 − 𝐵) + cos(𝐴 + 𝐵)]
2

Example 2.48
∫ sin 6𝑥 cos 2𝑥 𝑑𝑥

1
Use sin 𝐴 cos 𝐵 = 2 [sin(𝐴 − 𝐵) + sin(𝐴 + 𝐵)] with 𝐴 = 6𝑥 and 𝐵 = 2𝑥6:
1
= ∫ (sin 4𝑥 + sin 8𝑥) 𝑑𝑥
2
1 cos 4𝑥 cos 8𝑥
=− ( + )+𝐶
2 4 8
cos 4𝑥 cos 8𝑥
= −( + )+𝐶
8 16

Example 2.49
∫ cos(5𝑡) cos(10𝑡) 𝑑𝑡

6
The focus of this question is more on trigonometry than on integration. In fact, that is the focus of the entire section: using
trigonometric identities to make integration easier.
Get all the files at: https://bit.ly/azizhandouts

1
Using cos 𝐴 cos 𝐵 = 2 [cos(𝐴 − 𝐵) + cos(𝐴 + 𝐵)]

∫ cos(5𝑡) + cos(15𝑡) 𝑑𝑡

F. Integrals with 𝒔𝒆𝒄𝟐 𝒙

2.50: Pythagorean Identity: Alternate Version


tan2 𝑥 = sec 2 𝑥 − 1

Example 2.51: Observation


You should be able to get the examples below by using derivatives of basic functions, or simple identities.
Evaluate
A. ∫ sec 2 𝑥 𝑑𝑥
B. ∫ sec 𝑥 tan 𝑥 𝑑𝑥
C. ∫ tan2 𝑥 𝑑𝑥

∫ sec 2 𝑥 𝑑𝑥 = tan 𝑥 + 𝐶

∫ sec 𝑥 tan 𝑥 𝑑𝑥 = sec 𝑥 + 𝐶


Use the Pythagorean Identity to substitute tan2 𝑥 = sec 2 𝑥 − 1
∫(sec 2 𝑥 − 1) 𝑑𝑥 = tan 𝑥 − 𝑥 + 𝐶

2.52: Integrating 𝒔𝒆𝒄𝒏 𝒙


➢ Separate out sec 2 𝑥
➢ Convert remaining powers of sec 𝑥 to tan 𝑥 using sec 2 𝑥 = tan2 𝑥 + 1
➢ Substitute 𝑢 = tan 𝑥 ⇒ 𝑑𝑢 = sec 2 𝑥 𝑑𝑥

Example 2.53
Evaluate
∫ sec 4 𝑥 𝑑𝑥

Separate out sec 2 𝑥:


∫ sec 2 𝑥 sec 2 𝑥 𝑑𝑥

Convert remaining powers of sec 𝑥 to tan 𝑥 using sec 2 𝑥 = tan2 𝑥 + 1


= ∫ sec 2 𝑥 (tan2 𝑥 + 1)𝑑𝑥

Substitute 𝑢 = tan 𝑥 ⇒ 𝑑𝑢 = sec 2 𝑥 𝑑𝑥:


= ∫(𝑢2 + 1) 𝑑𝑢
Integrate:
𝑢3
= +𝑢+𝐶
3
Change back to the original variable:
Get all the files at: https://bit.ly/azizhandouts

tan3 𝑥
= + tan 𝑥 + 𝐶
3

Example 2.54
Evaluate
𝑥 𝑥
∫ tan ( ) sec 2 ( ) 𝑑𝑥
2 2

𝑥 1
Substitute 𝑢 = 2 ⇒ 𝑑𝑢 = 2 𝑑𝑥

2 ∫ tan 𝑢 sec 2 𝑢 𝑑𝑥
Substitute 𝑣 = tan 𝑢 ⇒ 𝑑𝑣 = sec 2 𝑢
𝑥 2
= ∫ 2𝑣 𝑑𝑣 = 𝑣 2 + 𝐶 = tan2 𝑣 + 𝐶 = 2 (tan ) + 𝐶
2

Example 2.55
Evaluate
∫ tan8 2𝑧 sec 6 2𝑧 𝑑𝑧

Use 𝑥 = 2𝑧 ⇒ 𝑑𝑥 = 2 𝑑𝑧
1
∫ tan8 𝑥 sec 6 𝑥 𝑑𝑥
2
Separate out sec 2 𝑥:
1
∫ tan8 𝑥 sec 4 𝑥 sec 2 𝑥 𝑑𝑥
2

Rewrite using the identity sec 2 𝑥 = tan2 𝑥 + 1:


1
∫ tan8 𝑥 (tan2 𝑥 + 1)2 sec 2 𝑥 𝑑𝑥
2

Let 𝑢 = tan 𝑥 ⇒ 𝑑𝑢 = sec 2 𝑥 𝑑𝑥:


1
∫ 𝑢8 (𝑢2 + 1)2 𝑑𝑢
2

Expand:
1
∫ 𝑢8 (𝑢4 + 2𝑢2 + 1) 𝑑𝑢
2

Multiply:
1
= ∫ 𝑢12 + 2𝑢10 + 𝑢8 𝑑𝑢
2

Integrate:
1 𝑢13 2𝑢11 𝑢9
( + + )+𝐶
2 13 11 9

Substitute 𝑢 = tan 𝑥:
Get all the files at: https://bit.ly/azizhandouts

1 tan13 𝑥 2 tan11 𝑥 tan9 𝑥


( + + )+𝐶
2 13 11 9
Substitute 𝑥 = 2𝑧:
1 tan13 2𝑧 2 tan11 2𝑧 tan9 2𝑧
( + + )+𝐶
2 13 11 9

2.56: Creating 𝒔𝒆𝒄𝟐 𝒙


If sec 2 𝑥 is not available, then it can be created using the identity
tan2 𝑥 = sec 2 𝑥 − 1

Example 2.57
Evaluate
∫ tan4 𝑥 𝑑𝑥

Expand:
∫ tan2 𝑥 tan2 𝑥 𝑑𝑥

Substitute tan2 𝑥 = sec 2 𝑥 − 1:


∫ tan2 𝑥 (sec 2 𝑥 − 1) 𝑑𝑥

Multiply and separate into two integrals:


∫ tan2 𝑥 sec 2 𝑥 𝑑𝑥 − ∫ tan2 𝑥 𝑑𝑥

In the first integral let 𝑢 = tan 𝑥 ⇒ 𝑑𝑢 = sec 2 𝑥 𝑑𝑥:


𝑢3 tan3 𝑥
∫ tan2 𝑥 sec 2 𝑥 𝑑𝑥 = ∫ 𝑢2 𝑑𝑢 = +𝐶 = + 𝐶1
3 3

The second integral is one we solved in the earlier examples:


− ∫ tan2 𝑥 𝑑𝑥 = − ∫(sec 2 𝑥 − 1) 𝑑𝑥 = − tan 𝑥 + 𝑥 + 𝐶2

And we combine the two to get:


tan3 𝑥
∫ tan4 𝑥 𝑑𝑥 = − tan 𝑥 + 𝑥 + 𝐶
3
G. Separating out 𝐬𝐞𝐜 𝒙 𝐭𝐚𝐧 𝒙

2.58: Creating 𝐬𝐞𝐜 𝒙 𝐭𝐚𝐧 𝒙


Separate out sec 𝑥 tan 𝑥, then we can make the substitution
𝑢 = sec 𝑥 ⇒ 𝑑𝑢 = sec 𝑥 tan 𝑥 𝑑𝑥

Convert remaining powers of tan 𝑥 to sec 𝑥 using


tan2 𝑥 = sec 2 𝑥 − 1
H. Integrals with 𝒄𝒔𝒄𝟐 𝒙
Get all the files at: https://bit.ly/azizhandouts

2.59: Integrating 𝒄𝒔𝒄𝒏 𝒙


➢ Separate out csc 2 𝑥
➢ Convert remaining powers of sec 𝑥 to tan 𝑥 using 𝑐𝑠𝑐 2 𝑥 = cot 2 𝑥 + 1
➢ Substitute 𝑢 = cot 𝑥 ⇒ 𝑑𝑢 = −csc 2 𝑥 𝑑𝑥

Example 2.60
∫ csc 4 𝑥 𝑑𝑥

Split the product:


∫(csc 2 𝑥)(csc 2 𝑥) 𝑑𝑥

Rewrite using 𝑐𝑠𝑐 2 𝑥 = cot 2 𝑥 + 1:


∫(csc 2 𝑥)(cot 2 𝑥 + 1) 𝑑𝑥

Substitute 𝑢 = cot 𝑥 ⇒ 𝑑𝑢 = −csc 2 𝑥 𝑑𝑥


= − ∫(𝑢2 + 1) 𝑑𝑢
Integrate:
𝑢3
=− −𝑢+𝐶
3
Change back to the original variable:
cot 3 𝑥
=− − cot 𝑥 + 𝐶
3

2.61: Creating 𝒄𝒔𝒄𝟐 𝒙


𝒄𝑠𝑐 𝟐 𝒙 can be created using the identity
tan2 𝑥 = sec 2 𝑥 − 1

Example 2.62
∫ cot 4 𝑥 𝑑𝑥

Split the powers inside the integral:


∫(cot 2 𝑥)(cot 2 𝑥) 𝑑𝑥

Substitute cot 2 𝑥 = csc 2 𝑥 − 1


∫(cot 2 𝑥)(csc 2 𝑥 − 1) 𝑑𝑥

Multiply and separate into two integrals:


∫ cot 2 𝑥 csc 2 𝑥 𝑑𝑥 − ∫ cot 2 𝑥 𝑑𝑥

For the first integral 𝑙𝑒𝑡 𝑢 = cot 𝑥 ⇒ 𝑑𝑢 = −csc 2 𝑥 𝑑𝑥


Get all the files at: https://bit.ly/azizhandouts

𝑢3 cot 3 𝑥
− ∫ 𝑢2 𝑑𝑢 = − =−
3 3
For the second integral:
− ∫ cot 2 𝑥 𝑑𝑥 = − ∫ csc 2 𝑥 − 1 𝑑𝑥 = ∫ 1 − csc 2 𝑥 𝑑𝑥 = 𝑥 + cot 𝑥

Combining the first and the second integrals:


cot 3 𝑥
− + cot 𝑥 + 𝑥 + 𝐶
3

Example 2.63
∫ csc 3 𝑥 𝑑𝑥

Substitute csc 2 𝑥 = cot 2 𝑥 + 1


∫ csc 𝑥 (1 + cot 2 𝑥) 𝑑𝑥

= ∫ csc 𝑥 𝑑𝑥 + ∫ cot 𝑥 (csc 𝑥 cot 𝑥) 𝑑𝑥

The first integral is:


∫ csc 𝑥 𝑑𝑥 = − ln|csc 𝑥 + cot 𝑥|

Use integration by parts for ∫ csc 𝑥 cot 2 𝑥 𝑑𝑥


𝑢 = cot 𝑥 , 𝑑𝑣 = csc 𝑥 cot 𝑥 𝑑𝑥
2
𝑑𝑢 = − csc 𝑥 𝑑𝑥, 𝑣 = − csc 𝑥 𝑑𝑥

Apply ∫ 𝑢 𝑑𝑣 = 𝑢𝑣 − ∫ 𝑣 𝑑𝑢:
− cot 𝑥 csc 𝑥 − ∫(− csc 𝑥) (− csc 2 𝑥 𝑑𝑥)

= − cot 𝑥 csc 𝑥 − ∫ csc 3 𝑥 𝑑𝑥

∫ csc 3 𝑥 𝑑𝑥 = − ln|csc 𝑥 + cot 𝑥| − cot 𝑥 csc 𝑥 − ∫ csc 3 𝑥 𝑑𝑥

2 ∫ csc 3 𝑥 𝑑𝑥 = − ln|csc 𝑥 + cot 𝑥| − cot 𝑥 csc 𝑥


1 1
∫ csc 3 𝑥 𝑑𝑥 = − ln|csc 𝑥 + cot 𝑥| − cot 𝑥 csc 𝑥
2 2

I. Integration by Parts

Example 2.64
∫ 𝑥 sin3 𝑥

Use integration by parts


𝑢 = 𝑥, 𝑑𝑣 = sin3 𝑥
Get all the files at: https://bit.ly/azizhandouts

cos 3 𝑥
𝑑𝑢 = 𝑑𝑥, 𝑣= − cos 𝑥
3

Apply ∫ 𝑢 𝑑𝑣 = 𝑢𝑣 − ∫ 𝑣 𝑑𝑢:
cos3 𝑥 cos3 𝑥
𝑥( − cos 𝑥) − ∫ − cos 𝑥 𝑑𝑥
3 3

sin3 𝑥
Using ∫ cos3 𝑥 𝑑𝑥 = sin 𝑥 − 3
+𝐶
cos3 𝑥 1 sin3 𝑥
= 𝑥( − cos 𝑥) − (sin 𝑥 − ) − sin 𝑥 + 𝐶
3 3 3

2.3 Trigonometric Substitutions


A. Summary of Substitutions

2.65: Substitutions
𝑥 = 𝑎 sin 𝜃 ⇒ √𝑎2 − 𝑥 2 = 𝑎 cos 𝜃
𝑥 = 𝑎 tan 𝜃 ⇒ √𝑎2 + 𝑥 2 = 𝑎 sec 𝜃
𝑥 = 𝑎 sec 𝜃 ⇒ √𝑥 2 − 𝑎2 = 𝑎 tan 𝜃

Part A
Substitute 𝑥 = 𝑎 sin 𝜃 in √𝑎2 − 𝑥 2 :
√𝑎2 − 𝑎2 sin2 𝜃 = √𝑎2 (1 − sin2 𝜃) = 𝑎√1 − sin2 𝜃 = 𝑎√cos 2 𝜃 = 𝑎 cos 𝜃 7
Part B
Substitute 𝑥 = 𝑎 tan 𝜃 in √𝑎2 + 𝑥 2 :
√𝑎2 + 𝑎2 tan2 𝜃 = √𝑎2 (1 + tan2 𝜃) = 𝑎√(1 + tan2 𝜃) = 𝑎√sec 2 𝜃 = 𝑎 sec 𝜃
Part C
Substitute 𝑥 = 𝑎 sec 𝜃 in √𝑥 2 − 𝑎2 :
√𝑎2 sec 2 𝜃 − 𝑎2 = √𝑎2 (sec 2 𝜃 − 1) = 𝑎√sec 2 𝜃 − 1 = 𝑎√tan2 𝜃 = 𝑎 tan 𝜃

B. 𝐜𝐨𝐬𝟐 𝒕 = 𝟏 − 𝐬𝐢𝐧𝟐 𝒕
The idea behind trigonometric substitutions is to use identities (such as the Pythagorean Identity) to convert
expressions that have square roots into ones where the square root is no longer there.
We do this using a change of variable. Hence, this is a specific case of 𝑢 substitution.

2.66: Change of Variable using 𝐜𝐨𝐬 𝟐 𝒕 = 𝟏 − 𝐬𝐢𝐧𝟐 𝒕


The square root expression √𝑟 2 − 𝑥 2 can be converted into one that does not have the square root using the
change of variable 𝑥 = 𝑟 sin 𝑡. That is:

𝑥 = 𝑟 sin 𝑡 ⇒ √𝑟 2 − 𝑥 2 = 𝑟 cos 𝑡

We begin with the expression:

7
Specifically √𝑥 2 = |𝑥|. For the time being, let us assume that cos 𝜃 is positive.
Get all the files at: https://bit.ly/azizhandouts

√𝑟 2 − 𝑥 2

Use a change of variable. Substitute 𝑥 = 𝑟 sin 𝑡:


√𝑟 2 − (𝑟 sin 𝑡)2 = √𝑟 2 − 𝑟 2 sin2 𝑡 = 𝑟√1 − sin2 𝑡

Substituting 1 − sin2 𝑡 = cos 2 𝑡:


= 𝑟√cos 2 𝑡 = 𝑟 cos 𝑡

2.67: Equation of a Circle


The equation of a circle with center at the origin and radius 𝑟 is
𝑦2 + 𝑥2 = 𝑟2

Using the distance formula, distance of any point on the circle will be
√(𝑦 − 0)2 + (𝑥 − 0)2 = √𝑦 2 + 𝑥 2 = 𝑟

Example 2.68
Show that the area of a circle with center at the origin and radius 𝑟 is
𝑟
4 ∫ √𝑟 2 − 𝑥 2 𝑑𝑥
0
Evaluate the above integral.

Solve the equation of a circle 𝑦 2 + 𝑥 2 = 𝑟 2 for 𝑦:


𝑦 = ±√𝑟 2 − 𝑥 2

This is not a function, but we consider the positive square root and double the
area (by symmetry).
In fact, since 𝑓(𝑥) = √𝑟 2 − 𝑥 2 is even, we can find four times the area in the first
quadrant:
𝑟
4 ∫ √𝑟 2 − 𝑥 2 𝑑𝑥
0

Use a change of variable. Substitute 𝑥 = 𝑟 sin 𝜃 and change the limits of integration:
𝑥 = 𝑟 sin 𝜃 ⇒ 𝑑𝑥 = 𝑟 cos 𝜃 𝑑𝜃
𝐿𝑜𝑤𝑒𝑟 𝐿𝑖𝑚𝑖𝑡: 𝑥 = 𝑟 sin 𝜃 = 0 ⇒ sin 𝜃 = 0 ⇒ 𝜃 = 0
𝜋
𝑈𝑝𝑝𝑒𝑟 𝐿𝑖𝑚𝑖𝑡: 𝑥 = 𝑟 sin 𝜃 = 𝑟 ⇒ sin 𝜃 = 1 ⇒ 𝜃 =
2
Hence, we get:
𝜋
2
4 ∫ √𝑟 2 − (𝑟 sin 𝜃)2 𝑟 cos 𝜃 𝑑𝜃
0
Factor out 𝑟, combine with the other 𝑟, and move both outside the integral:
𝜋
2
2
= 4𝑟 ∫ √12 − sin2 𝜃 cos 𝜃 𝑑𝜃
0
Substitute 1 − sin2 𝑡 = cos 2 𝑡:
𝜋
2
2
= 4𝑟 ∫ √cos2 𝜃 cos 𝜃 𝑑𝜃
0
Get all the files at: https://bit.ly/azizhandouts

Substitute √cos2 𝜃 = |cos 𝜃| = cos 𝜃, since 𝜃 is in the first quadrant or zero, and hence never negative:
𝜋 𝜋
2 2
= 4𝑟 ∫ cos 𝜃 cos 𝜃 𝑑𝜃 = 4𝑟 ∫ cos2 𝜃 𝑑𝜃
2 2
0 0

1
Reduce the power of cosine by using the half-angle formula cos2 𝜃 = 2 (1 + cos 2𝜃):
𝜋
21
2 (1 + cos 2𝜃)𝑑𝜃
= 4𝑟 ∫
0 2

1
Move 2 outside the integral, and integrate term by term:
𝜋
2
1 2
= 2𝑟 [𝜃 + sin 2𝜃]
2 0

Substitute the limits of integration:


𝜋 1 1
= 2𝑟 2 [( + sin 𝜋) − (0 + sin 0)]
2 2 2

Evaluate and simplify:


𝜋 𝜋
= 2𝑟 2 [( + 0) − (0 + 0)] = 2𝑟 2 [ ] = 𝜋𝑟 2
2 2

2.69: Equation of an Ellipse


The equation of an ellipse located at the origin with 𝑎, 𝑏 as the lengths of the axes is
𝑥2 𝑦2
+ =1
𝑎2 𝑏 2

➢ A circle is a special case of an ellipse with 𝑎 = 𝑏 = 𝑟.

Example 2.70: Area of an Ellipse


Show that the area of the ellipse below is 𝜋𝑎𝑏.
𝑥2 𝑦2
+ =1
𝑎2 𝑏 2

Solve the equation for 𝑦:


𝑦2 𝑥 2 𝑎2 − 𝑥 2
= 1 − =
𝑏2 𝑎2 𝑎2
2
𝑏
𝑦 2 = ( 2 ) (𝑎2 − 𝑥 2 )
𝑎
Take the square root both sides:
𝑏
𝑦 = ± √𝑎2 − 𝑥 2
𝑎

Since this is symmetrical with respect to both the axes, we can find four times the area in the first quadrant,
and hence we want:
𝑎
𝑏
4 ∫ √𝑎2 − 𝑥 2 𝑑𝑥
0 𝑎

Let 𝑥 = 𝑎 sin 𝜃:
Get all the files at: https://bit.ly/azizhandouts

𝑥 = 0 ⇒ 0 = 𝑎 sin 𝜃 ⇒ 0 = sin 𝜃 ⇒ 𝜃 = 0
𝜋
𝑥 = 𝑎 ⇒ 𝑎 = 𝑎 sin 𝜃 ⇒ 1 = sin 𝜃 ⇒ 𝜃 =
2
√𝑎2 − 𝑥 2 = √𝑎2 − (𝑎 sin 𝜃)2 = 𝑎√1 − sin2 𝜃 = 𝑎√cos2 𝜃 = 𝑎 cos 𝜃
𝑑𝑥 = 𝑎 cos 𝜃 𝑑𝜃

Make the substitutions above:


𝜋 𝜋
2𝑏 2
4∫ 𝑎 cos 𝜃 𝑎 cos 𝜃 𝑑𝜃 = 4𝑎𝑏 ∫ cos2 𝜃 𝑑𝜃
0 𝑎 0

By comparing this with the integral solved previously and using the constant multiple rule, we see that is it:
𝜋𝑎𝑏

Example 2.71
Evaluate
∫ √𝑎2 − 𝑥 2 𝑑𝑥

Use 𝒖-substitution
Substitute 𝑥 = 𝑎 sin 𝜃 to get:
√𝑎2 − 𝑥 2 = √𝑎2 − (𝑎 sin 𝜃)2 = 𝑎 cos 𝜃
𝑥 = 𝑎 sin 𝜃 ⇒ 𝑑𝑥 = 𝑎 cos 𝜃 𝑑𝜃

Making the above substitutions yields:


∫ √𝑎2 − 𝑥 2 𝑑𝑥 = ∫ 𝑎 cos 𝜃 𝑎 cos 𝜃 𝑑𝜃 = 𝑎2 ∫ cos2 𝜃 𝑑𝜃

Substitute using the half-angle identity:


1
= 𝑎2 ∫ (1 + cos 2𝜃) 𝑑𝜃
2
Integrate term by term:
𝑎2 sin 2𝜃
= [(𝜃 + )] + 𝐶
2 2

Substitute using the double-angle identity:


𝑎2 2 sin 𝜃 cos 𝜃
= [(𝜃 + )] + 𝐶
2 2

Convert back to original variable


Get our answer back in terms of the original variable. Draw a reference
triangle.
Note that:
𝑥
sin 𝜃 =
𝑎
−1
𝑥
𝜃 = sin ( )
𝑎
√𝑎 − 𝑥 2
2
cos 𝜃 =
𝑎

And now we can make the substitutions:


Get all the files at: https://bit.ly/azizhandouts

𝑎2 𝑥 𝑥 √𝑎2 − 𝑥 2
= [(sin−1 ( ) + ( ) )] + 𝐶
2 𝑎 𝑎 𝑎
𝑎2 𝑥 𝑥
= sin−1 ( ) + √𝑎2 − 𝑥 2 + 𝐶
2 𝑎 2

Example 2.72
𝑥2
∫ 3 𝑑𝑥
(9 − 𝑥 2 )2

Use the substitution 𝑥 = 3 sin 𝜃 ⇒ 𝑑𝑥 = 3 cos 𝜃 𝑑𝜃:


3 3 3 3
(9 − 𝑥 2 )2 = [√9 − 𝑥 2 ] = [√9 − 9 sin2 𝜃] = [3√cos 2 𝜃] = [3 cos 𝜃]3 = 27 cos3 𝜃

The integral becomes:


9 sin2 𝜃
∫ ∙ 3 cos 𝜃 𝑑𝜃 = ∫ tan2 𝜃 𝑑𝜃
27 cos3 𝜃

Substitute tan2 𝜃 = 1 − sec 2 𝜃


= ∫(1 − sec 2 𝜃) 𝑑𝜃 = 𝜃 − tan 𝜃 + 𝐶

𝑥 𝑥
𝑥 = 3 sin 𝜃 ⇒ = sin 𝜃 ⇒ 𝜃 = sin−1 ( )
3 3
𝑥
tan 𝜃 =
√9 − 𝑥 2

Changing back to the original variable:


𝑥 𝑥
= sin−1 ( ) + +𝐶
3 √9 − 𝑥 2

C. 𝐭𝐚𝐧𝟐 𝒕 = 𝐬𝐞𝐜 𝟐 𝒕 − 𝟏

2.73: Integral of ∫ 𝐬𝐞𝐜 𝒙 𝒅𝒙


∫ sec 𝑥 𝑑𝑥 = ln|sec 𝑥 + tan 𝑥| + 𝐶

We found ∫ sec 𝑥 in the chapter on 𝑢-substitution. Let’s redo it. We will use this formula later.

𝑓′(𝑥)
Bring the integral into the form ∫ 𝑓(𝑥) 𝑑𝑥 = ln|𝑓(𝑥)|, but we need to employ a clever trick by multiplying by a
𝑓𝑜𝑟𝑚 − 𝑜𝑓 − 𝑢𝑛𝑖𝑡𝑦:
sec 𝑥 + tan 𝑥 sec 2 𝑥 + sec 𝑥 tan 𝑥
∫ sec 𝑥 𝑑𝑥 = ∫ 𝑑𝑥
sec 𝑥 + tan 𝑥 sec 𝑥 + tan 𝑥

Let 𝑢 = sec 𝑥 + tan 𝑥 ⇒ 𝑑𝑢 = sec 𝑥 tan 𝑥 + sec 2 𝑥 𝑑𝑥 :


1
= ∫ 𝑑𝑢 = ln|𝑢| + 𝐶 = ln|sec 𝑥 + tan 𝑥| + 𝐶
𝑢
Get all the files at: https://bit.ly/azizhandouts

2.74: Change of Variable using 𝐭𝐚𝐧𝟐 𝒕 = 𝐬𝐞𝐜 𝟐 𝒕 − 𝟏


𝑥 = 𝑟 sec 𝑡 ⇒ √𝑥 2 − 𝑟 2 = 𝑟 tan 𝑡

The identity tan2 𝑡 = sec 2 𝑡 − 1 lets us eliminate square roots in expressions of the form √𝑥 2 − 𝑟 2 .

Use a change of variable. Substitute 𝑥 = 𝑟 sec 𝑡:


√(𝑟 sec 𝑡)2 − 𝑟 2 = √𝑟 2 sec 2 𝑡 − 𝑟 2 = 𝑟√sec 2 𝑡 − 1

Substituting sec 2 𝑡 − 1 = tan2 𝑡:


= 𝑟√tan2 𝑡 = 𝑟 tan 𝑡

Example 2.75
1
∫ 𝑑𝑥
√𝑥 2 − 1

Substitute 𝑥 = sec 𝑡 ⇒ 𝑑𝑥 = sec 𝑡 tan 𝑡 𝑑𝑡


1
∫ sec 𝑡 tan 𝑡 𝑑𝑡
√sec 2 𝑡 − 1
Substitute tan2 𝑡 = sec 2 𝑡 − 1:
1 1
∫ sec 𝑡 tan 𝑡 𝑑𝑡 = ∫ sec 𝑡 tan 𝑡 𝑑𝑡 = ∫ sec 𝑡 𝑑𝑡
√tan2 𝑡 tan 𝑡

Use the formula for ∫ sec 𝑡 𝑑𝑡 to integrate (see above):


∫ sec 𝑡 𝑑𝑡 = ln|sec 𝑡 + tan 𝑡| + 𝐶

We now need to convert back in terms of 𝑥.


ℎ𝑦𝑝 𝑥
sec 𝑡 = 𝑥 ⇒ =
𝑎𝑑𝑗 1
Then:
𝑜𝑝𝑝 √𝑥 2 − 1
tan 𝑡 = = = √𝑥 2 − 1
𝑎𝑑𝑗 1

Hence, the final answer is:


1
∫ 𝑑𝑥 = ln |𝑥 + √𝑥 2 − 1| + 𝐶
√𝑥 2 − 1

Example 2.76
Evaluate
1
∫ sin−1 ( ) 𝑑𝑥
𝑥

Integration by parts
1
𝑢 = sin−1 ( ) , 𝑑𝑣 = 𝑑𝑥
𝑥
1 1 1
𝑑𝑢 = − 𝑑𝑥 = − 𝑑𝑥 = − 𝑑𝑥, 𝑣=𝑥
1 √𝑥 2−1 √𝑥 2−1∙𝑥
√1 − 2 ∙ 𝑥 2
𝑥 ∙ 𝑥2
𝑥
Get all the files at: https://bit.ly/azizhandouts

Apply ∫ 𝑢 𝑑𝑣 = 𝑢𝑣 − ∫ 𝑣 𝑑𝑢:
1 1 1 1 1
∫ sin−1 ( ) 𝑑𝑥 = 𝑥 ∙ sin−1 − ∫ 𝑥 × − 𝑑𝑥 = 𝑥 ∙ sin−1 + ∫ 𝑑𝑥
𝑥 𝑥 2
√𝑥 − 1 ∙ 𝑥 𝑥 2
√𝑥 − 1
1
From the examples above, we know that ∫ 2 𝑑𝑥 = ln|𝑥 + √𝑥 2 − 1| + 𝐶. Hence, the final answer is:
√𝑥 −1
1 1
∫ sin−1 ( ) 𝑑𝑥 = 𝑥 ∙ sin−1 + ln |𝑥 + √𝑥 2 − 1| + 𝐶
𝑥 𝑥

D. 𝐬𝐞𝐜 𝟐 𝒕 = 𝟏 + 𝐭𝐚𝐧𝟐 𝒕

2.77: A trigonometric Integral


1
∫ sec 3 𝜃 𝑑𝜃 = (sec 𝜃 tan 𝜃 + ln|sec 𝜃 + 𝑡𝑎𝑛 𝜃|) + 𝐶
2

This was solved in the chapter on integration by parts.

Example 2.78
Evaluate
∫ √𝑎2 + 𝑥 2 𝑑𝑥

Let 𝑥 = 𝑎 tan 𝜃 ⇒ 𝑑𝑥 = 𝑎 sec 2 𝜃 𝑑𝜃:


√𝑎2 + 𝑥 2 = √𝑎2 + 𝑎2 tan2 𝜃 = 𝑎√1 + tan2 𝜃 = 𝑎 sec 𝜃

Making the substitutions above:


∫ 𝑎 sec 𝜃 𝑎 sec 2 𝜃 𝑑𝜃 = 𝑎2 ∫ sec 3 𝜃 𝑑𝜃

Which as per the property mentioned above is:


𝑎2
= (sec 𝜃 tan 𝜃 + ln|sec 𝜃 + 𝑡𝑎𝑛 𝜃|) + 𝐶1
2

𝑥 hyp √𝑎 2 +𝑥 2
Substitute tan 𝜃 = 𝑎 ⇒ sec 𝜃 = 𝑎𝑑𝑗 = 𝑎
𝑎2 𝑥 √𝑎2 + 𝑥 2 √𝑎2 + 𝑥2 𝑥
= ( ∙ + ln | + |) + 𝐶1
2 𝑎 𝑎 𝑎 𝑎

𝑥 𝑎2 𝑎2
= √𝑎2 + 𝑥 2 + (ln |𝑥 + √𝑎2 + 𝑥 2 |) − ln|𝑎| + 𝐶1
2 2 2

𝑎2
Substitute − 2
ln|𝑎| + 𝐶1 = 𝐶:
𝑥 𝑎2
= √𝑎2 + 𝑥 2 + (ln |𝑥 + √𝑎2 + 𝑥 2 |) + 𝐶
2 2
Get all the files at: https://bit.ly/azizhandouts

2.4 Partial Fractions8


A. Linear Factors
➢ It is difficult to integrate expressions that have a quadratic in the denominator.
➢ It is much easier to integrate expressions which have a linear factor in the denominator.
➢ Hence, the technique of partial fractions is useful.
➢ Partial fractions splits fractions which have a quadratic or higher polynomial into fractions which only
have linear factors in their denominator.
➢ Partial fractions itself is an Algebra technique and does not require any Calculus.

Example 2.79
1
∫ 𝑑𝑥
𝑥2 − 𝑎2

Since 𝑥 2 − 𝑎2 = (𝑥 + 𝑎)(𝑥 − 𝑎), we write:


1 𝐴 𝐵
= +
𝑥2
⏟ −𝑎 2 𝑥+𝑎 𝑥−𝑎
𝑰𝒅𝒆𝒏𝒕𝒊𝒕𝒚 𝑰

We wish to determine constants 𝐴 and 𝐵 such that the above is an identity. Which means that it is true for all
values of 𝑥 and 𝑎.

Eliminate fractions by multiplying both sides by 𝑥 2 − 𝑎2 :


1 = 𝐴(𝑥 − 𝑎) + 𝐵(𝑥 + 𝑎)

Since Identity I is true for all values of 𝑥, it must be true for 𝑥 = 𝑎:


1
1 = 𝐴(𝑎 − 𝑎) + 𝐵(𝑥 + 𝑎) ⇒ 1 = 2𝐵𝑎 ⇒ 𝐵 =
2𝑎

Since Identity I is true for all values of 𝑥, it must be true for 𝑥 = −𝑎:
1
1 = 𝐴(−𝑎 − 𝑎) + 𝐵(−𝑎 + 𝑎) ⇒ 1 = −2𝑎𝐴 ⇒ 𝐴 =
−2𝑎
1 1
Substitute 𝐴 = −2𝑎 and 𝐵 = 2𝑎 in the integral. Using the technique of partial fractions, we no longer have a
quadratic in the denominator.
1 1 1 1 1
∫ 𝑑𝑥 = − ∫ 𝑑𝑥 + ∫ 𝑑𝑥
𝑥 2 − 𝑎2 2𝑎 𝑥 + 𝑎 2𝑎 𝑥 − 𝑎

1
The above integrals can be evaluated using ∫ 𝑡 = ln|𝑡| + 𝐶
1 1
= − ln|𝑥 + 𝑎| + ln|𝑥 − 𝑎| + 𝐶
2𝑎 2𝑎

Example 2.80
𝑥+2
∫ 𝑑𝑥
(𝑥 − 3)(𝑥 + 5)

8
You should be able to write an algebraic fraction in parts using partial fractions. This is covered in the Note on Rational
Functions, and you should review it before you continue.
Get all the files at: https://bit.ly/azizhandouts

𝑥+2 𝐴 𝐵
= +
(𝑥 − 3)(𝑥 + 5) 𝑥 − 3 𝑥 + 5
Eliminate fractions:
𝑥 + 2 = 𝐴(𝑥 + 5) + 𝐵(𝑥 − 3)

Substitute:
3
𝑥 = −5 ⇒ −5 + 2 = 𝐴(−5 + 5) + 𝐵(−5 − 3) ⇒ −3 = −8𝐵 ⇒ 𝐵 =
8
5
𝑥 = 3 ⇒ 3 + 2 = 𝐴(3 + 5) + 𝐵(3 − 3) ⇒ 5 = 8𝐴 ⇒ 𝐴 =
8

Hence, the integration is:


5 1 3 1
∫ 𝑑𝑥 + ∫ 𝑑𝑥
8 𝑥−3 8 𝑥+5

Which we can integrate as:


5 3
ln|𝑥 − 3| + ln|𝑥 + 5| + 𝐶
8 8

Example 2.81
1
∫ 𝑑𝑥
2𝑥 2 − 3𝑥 − 5

2𝑥 2 − 3𝑥 − 5
𝑃𝑟𝑜𝑑𝑢𝑐𝑡 = (−5)(2) = −10 =
𝑆𝑢𝑚 = −3 = −5 + 2

2𝑥 2 − 5𝑥 + 2𝑥 − 5
𝑥(2𝑥 − 5) + 1(2𝑥 − 5)
(2𝑥 − 5)(𝑥 + 1)

Step I: Split the Fraction


1 𝐴 𝐵
= +
(2𝑥 − 5)(𝑥 + 1) 2𝑥 − 5 𝑥 + 1
Eliminate fractions:
1 = 𝐴(𝑥 + 1) + 𝐵(2𝑥 − 5)
0𝑥 + 1 = (𝐴 + 2𝐵)𝑥 + 𝐴 − 5𝐵

Use the method of undetermined coefficients. Equate coefficients on the left with coefficients on the right:
1 2
𝐴 + 2𝐵 = 0, 𝐴 − 5𝐵 = 1 ⇒ 𝐵 = − , 𝐴 =
7 7

Hence, the above integral can be written:


2 1
1 7 −7
∫ 2 𝑑𝑥 = ∫ + 𝑑𝑥
2𝑥 − 3𝑥 − 5 2𝑥 − 5 𝑥 + 1

Step II: Split the Fraction


Split the integral:
Get all the files at: https://bit.ly/azizhandouts

2 1
7 −7
∫ 𝑑𝑥 + ∫ 𝑑𝑥
2𝑥 − 5 𝑥+1

Move the constant outside:


1 2 1 1
∫ 𝑑𝑥 − ∫ 𝑑𝑥
7 2𝑥 − 5 7 𝑥+1

By observation or by using 𝑢 substitution:


1 1
ln|2𝑥 − 5| − ln|𝑥 + 1| + 𝐶
7 7

2.82: Linear Factors


An expression with a denominator that can be written as a product of linear factors can always be expressed
as the sum of fractions that only have linear factors in their denominator9:
𝐶 𝐴1 𝐴2 𝐴𝑛
= + + ⋯+
(𝑎1 𝑥 + 𝑏1 )(𝑎2 𝑥 + 𝑏2 ) … (𝑎𝑛 𝑥 + 𝑏𝑛 ) 𝑎1 𝑥 + 𝑏1 𝑎2 𝑥 + 𝑏2 𝑎𝑛 𝑥 + 𝑏𝑛

Note: Do not memorize the formula. It is meant to illustrate the concept.

2.83:Integration of Linear Factors


𝐴 𝐴
∫ = ln|𝑎𝑥 + 𝑏|
𝑎𝑥 + 𝑏 𝑎

Example 2.84
Write the form of the partial form decomposition (do not find the constants) of the following:

(Calculator Allowed) Example 2.85


Use long division to write the fraction as the sum of a polynomial and a fraction where the degree of the
numerator is less than the degree of the denominator
𝑥 5 + 5𝑥 4 + 6𝑥 3 + 𝑥 2 + 𝑥 + 1
𝑥 3 + 6𝑥 2 + 11𝑥 + 6

Factor 𝑥 3 + 6𝑥 2 + 11𝑥 + 6 using the rational roots, and factor theorems.

9
This can be proved using Algebra, though we will not do so here.
Get all the files at: https://bit.ly/azizhandouts

The possible roots are


±{1,2,3}

Substitute 𝑥 = −1 in 𝑥 3 + 6𝑥 2 + 11𝑥 + 6:
(−1)3 + 6(−1)2 + 11(−1) + 6 = 0

Use Polynomial Long Division:

Use partial fraction decomposition to write the fraction as the sum of fractions that only have a linear factor in
the denominator

Carry out the integration.

A. Repeated Linear Factors

Example 2.86
Show that the equality below is true and hence find the exact value of the definite integral:
5 5
2𝑥 + 3 1 4
∫ 2
𝑑𝑥 = ∫ ( + ) 𝑑𝑥
3 (2𝑥 − 1) 3 2𝑥 − 1 (2𝑥 − 1)2

5 5 5
2𝑥 − 1 + 4 2𝑥 − 1 + 4 1 4
∫ 2
= ∫ 2
= ∫ ( + ) 𝑑𝑥
3 (2𝑥 − 1) 3 (2𝑥 − 1) 3 2𝑥 − 1 (2𝑥 − 1)2

Example 2.87

B. Irreducible Quadratic Factors

Example 2.88

C. Repeated Irreducible Quadratic Factors

Example 2.89
Get all the files at: https://bit.ly/azizhandouts

2.5 Arc Length


A. Basics

2.90: Arc Length Formula (Informal Derivation)

𝐴𝐵 = √(Δ𝑥)2 + (Δ𝑦)2
Notice that line segment 𝐴𝐵 has a different length from 𝑎𝑟𝑐 𝐴𝐵.

To improve the approximation, we can reduce the distance between A and B. (𝑟𝑖𝑔ℎ𝑡 𝑑𝑖𝑎𝑔𝑟𝑎𝑚)
To make the approximation very, very close, take the limit as the distance between the points 𝐴 and 𝐵
becomes close to zero:
𝐴𝑠 𝐴𝐵 → 0, 𝑎𝑟𝑐 𝑙𝑒𝑛𝑔𝑡ℎ 𝐴𝐵 → 𝐴𝐵

Let 𝐴𝐵 = 𝑑𝑠, Δ𝑥 = 𝑑𝑥, Δ𝑦 = 𝑑𝑦 in 𝐴𝐵 = √(Δ𝑥)2 + (Δ𝑦)2 :


𝑑𝑠 = √(𝑑𝑥)2 + (𝑑𝑦)2

Multiply and divide by 𝑑𝑥:


1
𝑑𝑠 = √(𝑑𝑥)2 + (𝑑𝑦)2 ∙ ∙ 𝑑𝑥
𝑑𝑥
1
Move the inside the square root:
𝑑𝑥

(𝑑𝑥)2 (𝑑𝑦)2 𝑑𝑦 2
𝑑𝑠 = √ + ∙ 𝑑𝑥 = √1 + ( ) ∙ 𝑑𝑥
(𝑑𝑥)2 (𝑑𝑥)2 𝑑𝑥

Since we took the limit as 𝐴𝐵 = 𝑑𝑠 → 0, Δ𝑥 = 𝑑𝑥 → 0, Δ𝑦 = 𝑑𝑦 → 0. Integrate both sides:


𝑑𝑦 2
𝑠 = ∫ 𝑑𝑠 = ∫ √1 + ( ) ∙ 𝑑𝑥
𝑑𝑥
This is the formula for arc length.

Example 2.91
𝑦 = 𝑥2
Show that
𝑑2 𝑦 𝑑𝑦 2
𝑆𝑒𝑐𝑜𝑛𝑑 𝑑𝑒𝑟𝑖𝑣𝑎𝑡𝑖𝑣𝑒 = 2 ≠ ( ) = 𝑆𝑞𝑢𝑎𝑟𝑒 𝑜𝑓 𝑡ℎ𝑒 𝑑𝑒𝑟𝑖𝑣𝑎𝑡𝑖𝑣𝑒
𝑑𝑥 𝑑𝑥

Note that
𝑑𝑦 𝑑𝑦 2
= 2𝑥 ⇒ ( ) = 4𝑥 2
𝑑𝑥 𝑑𝑥
Get all the files at: https://bit.ly/azizhandouts

𝑑2 𝑦 𝑑 𝑑𝑦 𝑑
2
= ( )= (2𝑥) = 2
𝑑𝑥 𝑑𝑥 𝑑𝑥 𝑑𝑥

2.92: Arc Length of 𝒚 = 𝒇(𝒙), 𝒂 ≤ 𝒙 ≤ 𝒃


Arc length over the interval [𝑎, 𝑏] for a function 𝑦 = 𝑓(𝑥) is
𝑏
𝑑𝑦 2 𝑏
𝐿 = ∫ √1 + ( ) 𝑑𝑥 = ∫ √1 + [𝑓 ′ (𝑥)]2 𝑑𝑥
𝑎 𝑑𝑥 𝑎

➢ 𝑓′ must be continuous over the interval [𝑎, 𝑏]


➢ To be continuous, 𝑓′ must exist over the interval [𝑎, 𝑏]

When we say the interval [𝑎, 𝑏], we mean the points:


𝐴 = (𝑎, 𝑓(𝑎)), 𝐵 = (𝑏, 𝑓(𝑏))

Example 2.93
3
Find the arc length of the curve 𝑓(𝑥) = 𝑦 = 𝑥 2 − 4 over the interval 4 ≤ 𝑥 ≤ 8. Write your final answer in
exact radical form.

𝑑𝑦 3 1
𝑑𝑦 2 9
Make the substitutions 𝑎 = 4, 𝑏 = 8, 𝑑𝑥 = 2 𝑥 2 ⇒ (𝑑𝑥 ) = 4 𝑥 in
𝑏
𝑑𝑦 2 8
9
𝐿 = ∫ √1 + ( ) 𝑑𝑥 = ∫ √1 + 𝑥 𝑑𝑥
𝑎 𝑑𝑥 4 4

Use 𝑢 substitution with:


9 9 4
𝑢 = 1 + 𝑥 ⇒ 𝑑𝑢 = 𝑑𝑥 ⇒ 𝑑𝑥 = 𝑑𝑢
4 4 9

The limits of integration change to:


𝑥 = 4 ⇒ 𝑢 = 1 + 9 = 10
𝑥 = 8 ⇒ 𝑢 = 1 + 18 = 19

19
4
= ∫ √𝑢 ∙ 𝑑𝑢
10 9

Carry out the integration and move the constant outside:


4 2 3 19 8 3 19
= [ 𝑢2 ] = [𝑢2 ]
9 3 10 27 10

Substitute the limits of integration and simplify:


8 3 3 8
= (192 − 102 ) = (19√19 − 10√10)
27 27

Example 2.94
Concept Questions: Continuity

2.95: Perfect Square


𝑑𝑦 2
If the expression 1 + (𝑑𝑥 ) is a perfect square, then
Get all the files at: https://bit.ly/azizhandouts

𝑑𝑦 2
) √1 + (
𝑑𝑥
Will have the square and the square root cancel, and the integration will become simpler.

Example 2.96: Perfect Square


𝑒 𝑥 +𝑒 −𝑥
Find the arc length of 𝑦 = 2
from 𝑥 = 1 to 𝑥 = 2.

Find the first derivative:


𝑑𝑦 1
= ( ) (𝑒 𝑥 − 𝑒 −𝑥 )
𝑑𝑥 2

Square both sides:


𝑑𝑦 2 1
( ) = ( ) (𝑒 2𝑥 − 2 + 𝑒 −2𝑥 )
𝑑𝑥 4

Add 1 to both sides:


2
𝑑𝑦 2 1 𝑒 𝑥 + 𝑒 −𝑥
1+( ) = ( ) (𝑒 2𝑥 + 2 + 𝑒 −2𝑥 ) = ( )
𝑑𝑥 4 2

𝑑𝑦 2 𝑒 𝑥 +𝑒 −𝑥 𝑏 𝑑𝑦 2
Substitute √1 + (𝑑𝑥 ) = 2
,𝑎 = 1, 𝑏 = 2 in 𝐿 = ∫𝑎 √1 + (𝑑𝑥 ) 𝑑𝑥
2
𝑒 𝑥 + 𝑒 −𝑥 1
𝐿=∫ ( ) 𝑑𝑥 = [𝑒 𝑥 − 𝑒 −𝑥 ]12
1 2 2

1 1 1 1
= (𝑒 2 − 𝑒 −2 − 𝑒 + 𝑒 −1 ) = (𝑒 2 + 𝑒 − − 2 )
2 2 𝑒 𝑒

B. Integrating in terms of y
If the derivative does not exist, or the derivative is discontinuous, then we cannot use the arc length formula.
In such a case, we might still be able to find the arc length by integrating in terms of 𝑦 rather than 𝑥.

2.97: Arc Length of 𝒙 = 𝒈(𝒚), 𝒂 ≤ 𝒙 ≤ 𝒃


Arc length over the interval [𝑎, 𝑏] for a function 𝑥 = 𝑔(𝑥) is
𝑑
𝑑𝑥 2 𝑑
𝐿 = ∫ √1 + ( ) 𝑑𝑦 = ∫ √1 + [𝑔′ (𝑥)]2 𝑑𝑦
𝑐 𝑑𝑦 𝑐

➢ 𝑔′ must be continuous over the interval [𝑎, 𝑏]


➢ To be continuous, 𝑔′ must exist over the interval [𝑎, 𝑏]

When we say the interval [𝑐, 𝑑], we mean the points


𝐴 = (𝑔(𝑐), 𝑐), 𝐵 = (𝑔(𝑑), 𝑑)

Example 2.98
2
𝑦 = 𝑥3, 0 ≤ 𝑥 ≤ 1
A. Explain why the arc length formula that integrates in terms of 𝑥 will not work
B. Find the arc length by using the formula that integrates in terms of 𝑦.
Get all the files at: https://bit.ly/azizhandouts

Part A
𝑑𝑦 2 −1 2
= 𝑥 3= 1
𝑑𝑥 3
3𝑥 3
When
𝑑𝑦 2
𝑥=0⇒ = ⇒ 𝑁𝑜𝑡 𝑑𝑒𝑓𝑖𝑛𝑒𝑑
𝑑𝑥 0

Part B
3
𝑥 = 𝑦2
𝑑𝑥 3 1
= 𝑦2
𝑑𝑦 2
𝑑𝑥 2 9
1+( ) =1+ 𝑦
𝑑𝑦 4

𝑑
𝑑𝑥 2 1
9 8
𝐿 = ∫ √1 + ( ) 𝑑𝑦 = ∫ √1 + 𝑦 𝑑𝑦 = (13√13 − 1)
𝑐 𝑑𝑦 0 4 27

C. Differentials

2.99: Differential Formula

D. Arc Length Function

Example 2.100
Arc length of circle

2.6 Improper Integrals: Type I


A. Integrals tending to ∞
If you wish to calculate a definite integral which has limits of integration tending to infinity, you cannot do by
substitution, because infinity is a concept, and not a number.

2.101: Integrals: Upper Limit → ∞


If 𝑓(𝑥) is continuous on (𝑎, ∞) and the limit below exists and is finite, then:
∞ 𝑡
∫ 𝑓(𝑥) 𝑑𝑥 = lim ∫ 𝑓(𝑥) 𝑑𝑥
𝑎 𝑡→∞ 𝑎

➢ For the limit to exist, the integral must exist.

Example 2.102
𝑎 1
Evaluate∫1 𝑥2
𝑑𝑥 and use it to find the value of:
10 1
A. ∫1 𝑥 2 𝑑𝑥
100 1
B. ∫1 𝑥 2 𝑑𝑥
Get all the files at: https://bit.ly/azizhandouts

1000 1
C. ∫1 𝑑𝑥
𝑥2
∞ 1
D. ∫1 𝑥 2 𝑑𝑥

𝑎
1 1𝑎 1 1 1
∫ 2 𝑑𝑥 = [− ] = − − (− ) = 1 −
1 𝑥 𝑥1 𝑎 1 𝑎
Part A
1 9
1− = = 0.9
10 10
Part B
1 99
1− = = 0.99
100 100
Part C
1 999
1− = = 0.999
1000 1000
Part D
Determine the limit as the variable 𝑎 upper limit tends to infinity:
1
lim (1 − ) = 1 + 0 = 1
𝑎→∞ 𝑎

2.103: Convergence and Divergence


∞ 𝑡
∫ 𝑓(𝑥) 𝑑𝑥 = lim ∫ 𝑓(𝑥) 𝑑𝑥
𝑎 𝑡→∞ 𝑎
If the limit is:
➢ a finite number, the integral is convergent.
➢ infinite, or does not exist, the above integral is divergent.

Improper integrals can be evaluated only if they can be assigned a finite value (that is, are convergent).

For example, a geometric series:


𝑐𝑜𝑛𝑣𝑒𝑟𝑔𝑒𝑠 𝑓𝑜𝑟 − 1 < 𝑟 < 1
𝑎 + 𝑎𝑟 + ⋯ {
𝑑𝑖𝑣𝑒𝑟𝑔𝑒𝑠 𝑜𝑡ℎ𝑒𝑟𝑤𝑖𝑠𝑒

Example 2.104
𝜋
2
∫ sec 2 𝑥 𝑑𝑥
0

Replace the upper limit with a variable, and evaluate the integral:
𝑘
∫ sec 2 𝑥 𝑑𝑥 = [tan 𝑥]𝑘0 = tan 𝑘 − tan 0 = tan 𝑘
0
Determine the limit as the variable 𝑘 tends to infinity:
lim𝜋 tan 𝑘 = ∞
𝑘→
2

Example 2.105
Evaluate the integral below:

1
∫ 𝑑𝑥
1 𝑥4
Get all the files at: https://bit.ly/azizhandouts

Replace the upper limit with a variable, and evaluate the integral:
𝑡
1 1 1 𝑡 1 1 1 1 1
∫ 2 𝑑𝑥 = − [ 3 ] = − ( 3 − ) = (1 − 3 )
𝑎 𝑥 3 𝑥 1 3 𝑡 1 3 𝑡
1
As 𝑡 → ∞ ⇒ → 0 and hence:
𝑡3
1 1 1
(1 − 3 ) →
3 𝑡 3

Example 2.106
Evaluate the integral below:

1
∫ 𝑑𝑥
1 𝑥

Replace the upper limit with a variable, and evaluate the integral:
𝑡
1
∫ 𝑑𝑥 = [ln |𝑥|]1𝑡 = ln 𝑡 − ln 1 = ln 𝑡 − 0 = ln 𝑡
1 𝑥
As
𝑡 → ∞ ⇒ ln 𝑡 → ∞ ⇒ 𝐿𝑖𝑚𝑖𝑡 𝑑𝑜𝑒𝑠 𝑛𝑜𝑡 𝑒𝑥𝑖𝑠𝑡 ⇒ 𝐼𝑛𝑡𝑒𝑔𝑟𝑎𝑙 𝑑𝑖𝑣𝑒𝑟𝑔𝑒𝑠

Example 2.107
Evaluate the integral below:

1
∫ 𝑑𝑥
1 √𝑥

Replace the upper limit with a variable, and evaluate the integral:
𝑡 𝑡
1 1 1 1
∫ 𝑑𝑥 = [ √𝑥] = √𝑡 −
1 √𝑥 2 1 2 2

1 1
lim ( √𝑡 − ) 𝑑𝑜𝑒𝑠 𝑛𝑜𝑡 𝑒𝑥𝑖𝑠𝑡 ⇒ 𝐼𝑛𝑡𝑒𝑔𝑟𝑎𝑙 𝑑𝑖𝑣𝑒𝑟𝑔𝑒𝑠
𝑡→∞ 2 2

𝟏
2.108: Convergence of
𝒙𝒑

1
∫ 𝑑𝑥
1 𝑥𝑝

𝐶𝑜𝑛𝑣𝑒𝑟𝑔𝑒𝑠 𝑓𝑜𝑟 𝑝 > 1


𝐷𝑖𝑣𝑒𝑟𝑔𝑒𝑠 𝑓𝑜𝑟 𝑝 ≤ 1

Case I: For 𝒑 ≠ 𝟏
Evaluate the proper integral first (by replacing ∞ with 𝑡):
𝑡 𝑡 𝑡
1 1 1 1 1 1
∫ 𝑝 𝑑𝑥 = ∫ 𝑥 −𝑝 𝑑𝑥 = [ 𝑝−1 ] = ( 𝑝−1 − 1) = (1 − 𝑝−1 )
1 𝑥 1 (−𝑝 + 1)(𝑥 ) 1 −𝑝 + 1 𝑡 𝑝−1 𝑡

1 1 1 1
lim (1 − 𝑝−1 ) = lim (1 − 𝑝−1 )
𝑡→∞ 𝑝−1 𝑡 𝑝 − 1 𝑡→∞ 𝑡

If 𝑝 > 1
Get all the files at: https://bit.ly/azizhandouts

1
lim =0
𝑡→∞ 𝑡 𝑝−1
If 𝑝 < 1
1
lim = lim 𝑡1−𝑝
𝑡→∞ 𝑡 𝑝−1 𝑡→∞
1−0.5
𝑝 = 0.5 ⇒ lim 𝑡 = lim 𝑡 0.5 = lim √𝑡
𝑡→∞ 𝑡→∞ 𝑡→∞
1−(−1) 2
𝑝 = −1 ⇒ lim 𝑡 = lim 𝑡
𝑡→∞ 𝑡→∞

Case II: For 𝒑 = 𝟏:


1 1 1
When 𝑝 = 1 ⇒ 𝑥 𝑝 = 𝑥 1 = 𝑥

Replace the infinity with 𝑡:


𝑡
1
∫ 𝑑𝑥 = [ln |𝑥|]1𝑡 = ln 𝑡 − ln 1 = ln 𝑡
1 𝑥

The improper integral is when the upper limit 𝑡 tends to infinity:


𝑡 → ∞ ⇒ ln 𝑡 → ∞ ⇒ 𝐿𝑖𝑚𝑖𝑡 𝑑𝑜𝑒𝑠 𝑛𝑜𝑡 𝑒𝑥𝑖𝑠𝑡 ⇒ 𝐼𝑛𝑡𝑒𝑔𝑟𝑎𝑙 𝑑𝑖𝑣𝑒𝑟𝑔𝑒𝑠

Example 2.109
Without evaluating the entire integral, check whether it converges:

1
∫ 𝜋 𝑑𝑥
0.2 𝑥 3

𝜋 3.14
≈ > 1 ⇒ 𝐶𝑜𝑛𝑣𝑒𝑟𝑔𝑒𝑠
3 3

2.110: Improper Integrals as Area


One interpretation of a definite integral is as area under the curve.
Improper integrals can be used to calculate the area under a curve, if it has a finite value.

➢ This has applications in probability distributions.

Example 2.111
Determine the area to the right of the 𝑦 axis, above the 𝑥 axis and under the curve 𝑦 = 𝑒 −𝑥 .

We want the integral:



∫ 𝑒 −𝑥 𝑑𝑥
0

Replace the upper limit with a variable, and evaluate the integral:
𝑘
1 𝑘 1 1 1
∫ 𝑒 −𝑥 𝑑𝑥 = [− 𝑥 ] = − 𝑘 − (− 0 ) = 1 − 𝑘
0 𝑒 0 𝑒 𝑒 𝑒

Determine the limit as the variable 𝑘 tends to infinity:


=1−0=1

2.112: Shortcut Method


Get all the files at: https://bit.ly/azizhandouts

If you understand the concept, you can do the process less formally by using ∞ in the integral, and taking the
limit at the appropriate step.

2.113: Integration by Parts


Improper integrals can require the use of integration techniques such as integration by parts.

Example 2.114
Determine the area to the right of the 𝑦 axis and under the curve10 𝑦 = 𝑥𝑒 −𝑥 .


∫ 𝑥𝑒 −𝑥 𝑑𝑥
0

Find the associated indefinite integral using integration by parts:


𝑢 = 𝑥, 𝑑𝑣 = 𝑒 −𝑥 𝑑𝑥
𝑑𝑢 = 𝑑𝑥, 𝑣 = −𝑒 −𝑥

Apply ∫ 𝑢 𝑑𝑣 = 𝑢𝑣 − ∫ 𝑣 𝑑𝑢:
𝑥+1
∫ 𝑥𝑒 −𝑥 𝑑𝑥 = 𝑥(−𝑒 −𝑥 ) − ∫ −𝑒 −𝑥 𝑑𝑥 = −𝑥𝑒 −𝑥 − 𝑒 −𝑥 + 𝐶 = − +𝐶
𝑒𝑥


𝑥+1 ∞ 𝑥+1 1−0
∫ 𝑥𝑒 −𝑥 𝑑𝑥 = [ 𝑥 ] = ( lim 𝑥
) − (− 0 ) = 0 + 1 = 1
0 𝑒 0 𝑥→∞ 𝑒 𝑒

Example 2.115
Determine the area to the right of the 𝑦 axis and under the curve11 𝑦 = 𝑥 2 𝑒 −𝑥 .


∫ 𝑥 2 𝑒 −𝑥 𝑑𝑥
0
The associated indefinite integral is:
𝑥 2 + 2𝑥 + 2
∫ 𝑥 2 𝑒 −𝑥 𝑑𝑥 = −(𝑥 2 𝑒 −𝑥 + 2𝑥𝑒 −𝑥 + 2𝑒 −𝑥 ) = − ( )
𝑒𝑥

The improper integral becomes:


∞ ∞
𝑥 2 + 2𝑥 + 2
∫ 𝑥 2 𝑒 −𝑥 𝑑𝑥 = [− ( )] = 0 − (−2) = 2
0 𝑒𝑥 0

Where we evaluated the upper and lower limit as:


𝑥 2 + 2𝑥 + 2
lim (− )=0
𝑥→∞ 𝑒𝑥
02 + 2(0) + 2
𝑥 = 0 ⇒ (− ) = −2
𝑒0

2.116: Integrals: Lower Limit → −∞

10
This is an example of the gamma function, an important function in higher mathematics.
∞ ∞ ∞
11
As you might have noticed, ∫0 𝑒 −𝑥 𝑑𝑥 = 0! = 1, ∫0 𝑥𝑒 −𝑥 𝑑𝑥 = 1! = 1, ∫0 𝑥 2 𝑒 −𝑥 𝑑𝑥 = 2! = 2. The gamma function is a
generalization of the factorial function.
Get all the files at: https://bit.ly/azizhandouts

𝑎 𝑎
∫ 𝑓(𝑥) 𝑑𝑥 = lim ∫ 𝑓(𝑥) 𝑑𝑥
−∞ 𝑡→−∞ 𝑡
There are some important conditions:
𝑎
➢ ∫𝑡 𝑓(𝑥) 𝑑𝑥 exists for every number 𝑡 ≤ 𝑎
➢ The limit above exists and is finite

Example 2.117

2.118: Doubly Improper Integrals: Limits → ±∞


To handle improper integrals which range from −∞ to ∞, we need to divide (cut) the integral into two
integrals, each of which is a (singly) improper integral:
−∞ 𝑐 ∞
∫ 𝑓(𝑥) 𝑑𝑥 = ∫ 𝑓(𝑥) 𝑑𝑥 + ∫ 𝑓(𝑥) 𝑑𝑥
−∞ −∞ 𝑐
Where
𝑐 𝑖𝑠 𝑎𝑛𝑦 𝑟𝑒𝑎𝑙 𝑛𝑢𝑚𝑏𝑒𝑟

Note:
➢ 𝑓(𝑥) must be continuous over (−∞, ∞)

Example 2.119: Arc Length


Determine the length of the semicircle 𝑦 = √1 − 𝑥 2

𝑑𝑦 1 𝑥
= ∙ 2𝑥 =
𝑑𝑥 2√1 − 𝑥 2 √1 − 𝑥 2
2 2
𝑑𝑦 𝑥
( ) =
𝑑𝑥 1 − 𝑥2

𝑑𝑦 2 𝑥2 1 − 𝑥2 + 𝑥2 1
1+( ) =1+ 2
= 2
=
𝑑𝑥 1−𝑥 1−𝑥 1 − 𝑥2

𝑏
𝑑𝑦 2 1
1 1
1
𝐿 = ∫ √1 + ( ) 𝑑𝑥 = ∫ √ 2
𝑑𝑥 = ∫ 𝑑𝑥
𝑎 𝑑𝑥 −1 1 − 𝑥 −1 √1 − 𝑥
2

1
This integral is doubly improper since is not defined at 𝑥 ∈ {−1,1}.
√1−𝑥 2
To calculate the integral, we split it in between the limits of integration:
0 1
1 1
=∫ 𝑑𝑥 + ∫ 𝑑𝑥
2 2
−1 √1 − 𝑥 0 √1 − 𝑥

Replace the −1 with an 𝑎, and 1 with a 𝑏:


0 𝑏
1 1
=∫ 𝑑𝑥 + ∫ 𝑑𝑥
𝑎 √1 − 𝑥2 0 √1 − 𝑥 2

1
Since ∫ 𝑑𝑥 = sin−1 𝑥 + 𝐶:
√1−𝑥 2
= [sin−1 𝑥]0𝑎 𝑑𝑥 + [sin−1 𝑥]𝑏0 𝑑𝑥

Substitute the limits of integration:


Get all the files at: https://bit.ly/azizhandouts

= (sin−1 0 − sin−1 𝑎) − (sin−1 𝑏 − sin−1 1)

Take the
= lim (sin−1 0 − sin−1 𝑎) − lim (sin−1 𝑏 − sin−1 0)
𝑎→−1 𝑏→1
𝜋 𝜋 𝜋 𝜋
= [0 − (− )] + [ − 0] = + = 𝜋
2 2 2 2

2.120: 𝒖 Substitution
When using a 𝑢 substitution, the limits of integration need to be changed as well.
In particular, the infinite limits also need to be changed.

Example 2.121

𝑥
∫ 𝑑𝑥
−∞ (𝑥 2 + 5)2

Split the integral at a suitable value (𝑐 = 0):


0 ∞
𝑥 𝑥
∫ 2 2
𝑑𝑥 + ∫ 2 2
𝑑𝑥
−∞ (𝑥 + 5) 0 (𝑥 + 5)

Notice that the numerator is a constant multiple of the derivative of the expression in the square.
Use 𝑢 substitution with
𝑢 = 𝑥 2 + 5 ⇒ 𝑑𝑢 = 2𝑥 𝑑𝑥

The limits of integration are:


𝑥 = 0 ⇒ 𝑢 = 02 + 5 = 5
𝑥 → −∞ ⇒ 𝑢 → ∞
𝑥→∞⇒𝑢→∞

Substitute the values calculated above:


1 5 1 1 ∞ 1 1 1
∫ 2 𝑑𝑢 + ∫ 2 𝑑𝑢 = − + =0
2 ∞𝑢 2 5 𝑢 10 10

Where we evaluated the integrals above using:


1 5 1 1 1 5 1 1 1 1 1 1
∫ 2 𝑑𝑢 = lim [− ] = − lim ( − ) = − ∙ = −
2 ∞𝑢 𝑝→∞ 2 𝑢 𝑝 2 𝑝→∞ 5 𝑝 2 5 10
1 ∞ 1 1 1 𝑞 1 1 1 1 1 1
∫ 𝑑𝑢 = lim [− ] = − lim ( − ) = ∙ =
2 5 𝑢2 𝑞→∞ 2 𝑢 5 2 𝑞→∞ 𝑞 5 2 5 10

Example 2.122

𝑥3
∫ 4
𝑑𝑥
−∞ 𝑥 + 1

B. Volumes with Improper Integrals

Example 2.123
2
Find the volume between 𝑧 = 𝑒 −𝑥 is revolved around the 𝑧 axis and the 𝑥𝑦 plane.
Get all the files at: https://bit.ly/azizhandouts

Use a change of variables. Let 𝑌 be the 𝑧 axis, 𝑋 be the 𝑥 axis, 𝑍 be the 𝑦 axis.

2
𝑌 = 𝑒 −𝑋 is revolved around the 𝑌 axis, find the volume between this and the 𝑋𝑍 plane

𝑆ℎ𝑒𝑙𝑙 𝑆ℎ𝑒𝑙𝑙
𝑉 = 2𝜋 ∫ ( )( )
𝑅𝑎𝑑𝑖𝑢𝑠 𝐻𝑒𝑖𝑔ℎ𝑡

𝑆ℎ𝑒𝑙𝑙 𝑅𝑎𝑑𝑖𝑢𝑠 = 𝑥
2
𝑆ℎ𝑒𝑙𝑙 𝐻𝑒𝑖𝑔ℎ𝑡 = 𝑒 −𝑥


2
2𝜋 ∫ 𝑥𝑒 −𝑥 𝑑𝑥
0

2 𝑡 2 2
−𝜋[𝑒 −𝑥 ]0 = −𝜋(𝑒 −𝑡 − 𝑒 0 ) = 𝜋(𝑒 0 − 𝑒 −𝑡 )

2
lim 𝜋(𝑒 0 − 𝑒 −𝑡 ) = 𝜋(𝑒 0 ) = 𝜋
𝑡→∞

2.7 Improper Integrals: Type II


A. Type II Improper Integrals: Tending to Infinity on the 𝒚-axis
These integrals tend on infinity on the 𝑦 axis because there on an infinite discontinuity.

2.124: Types of Discontinuity


➢ Lower limit has a discontinuity
➢ Upper limit has a discontinuity
➢ Discontinuity exists between the limits

Example 2.125
1
1
∫ 3 𝑑𝑥
−1 √𝑥

Split the integral:


𝑡
1 3 2 𝑡 3 2 2 3 2 3 2 3
𝐴= ∫ 𝑑𝑥 = [𝑥 3] = (𝑡 3 − (−1)3 ) = (𝑡 3 − 1) ⇒ lim (𝑡 3 − 1) = −
3
−1 √𝑥 2 −1 2 2 𝑡→0 2 2
1 1
1 3 2 3 2 2 3 2 2 3
𝐵 = ∫ 3 𝑑𝑥 = [ 𝑥 3 ] = (13 − 𝑡 3 ) ⇒ lim (13 − 𝑡 3 ) =
𝑡 √𝑥 2 𝑡 2 𝑡→0 2 2

1
1 3 3
∫ 𝑑𝑥 = 𝐴 + 𝐵 = − + = 0
3
−1 √𝑥 2 2

Example 2.126
Determine the 𝑥 values for which the integral below converges:
𝑥
𝑢
∫ 𝑑𝑢
√1 − 𝑢
0
Get all the files at: https://bit.ly/azizhandouts

𝑡 = 1 − 𝑢, 𝑑𝑡 = −𝑑𝑢, 𝑢 = 1 − 𝑡

𝑢 𝑡−1 2 3 2 3
∫ 𝑑𝑢 = ∫ 𝑑𝑡 = 𝑡 2 − 2√𝑡 = (1 − 𝑢)2 − 2√1 − 𝑢 + 𝐶
√1 − 𝑢 √𝑡 3 3

𝑥
𝑥
𝑢 2 3 2 3 2
∫ 𝑑𝑢 = [ (1 − 𝑢)2 − 2√1 − 𝑢] = ( (1 − 𝑥)2 − 2√1 − 𝑥) − ( − 2)
√1 − 𝑢 3 0 3 3
0
2 3 4
= ( (1 − 𝑥)2 − 2√1 − 𝑥) +
3 3

For the expression above to be defined:


1−𝑥 ≥0⇒𝑥 ≤1

At 𝑥 = 1, the original function is undefined. Hence, take the limit of the expression as 𝑥 → 1:
2 3 4
lim ( (1 − 𝑥)2 − 2√1 − 𝑥) +
𝑥→1 3 3

Evaluate the limit by substitution:


2 3 4 4
( (1 − 1)2 − 2√1 − 1) + =
3 3 3

Hence, the interval of convergence of the integral is:


𝑥≤1
B. Applications

Example 2.127
Area of a Region
Combining Type I and Type II Integrals
Inequalities
Volume
Laplace Transforms
Escape Velocity
Get all the files at: https://bit.ly/azizhandouts

3. FURTHER TOPICS
3.1 Hyperbolic Integrals
A. Antiderivatives

Example 3.1
1
∫ 𝜋 sinh 𝑥 + cosh 𝑥 𝑑𝑥
𝜋

Write as the sum of two integrals, and then integrate:


1 1
𝜋 ∫ sinh 𝑥 + ∫ cosh 𝑥 𝑑𝑥 = 𝜋 cosh 𝑥 + sinh 𝑥 + 𝐶
𝜋 𝜋

B. u-substitution

Example 3.2
sinh 𝜋𝑥
∫ 𝑑𝑥
𝜋

sinh 𝜋𝑥 1
∫ 𝑑𝑥 = ∫ sinh 𝜋𝑥 𝑑𝑥
𝜋 𝜋
Substitute 𝑢 = 𝜋𝑥 ⇒ 𝑑𝑢 = 𝜋 𝑑𝑥:
1 1 cosh 𝜋𝑥
2
∫ sinh 𝑢 𝑑𝑢 = 2 cosh 𝑢 =
𝜋 𝜋 𝜋2

C. Integration by Parts

Example 3.3
∫ 𝑥 sinh 𝑒𝑥 𝑑𝑥

We use integration by parts:


𝑢 = 𝑥, 𝑑𝑣 = sinh 𝑒𝑥 𝑑𝑥
cosh 𝑒𝑥
𝑑𝑢 = 𝑑𝑥, 𝑣=
𝑒
Apply ∫ 𝑢 𝑑𝑣 = 𝑢𝑣 − ∫ 𝑣 𝑑𝑢:
cosh 𝑒𝑥 cosh 𝑒𝑥 cosh 𝑒𝑥 1
∫ 𝑥 sinh 𝑒𝑥 𝑑𝑥 = 𝑥 ∙ −∫ 𝑑𝑢 = 𝑥 ∙ − 2 sinh 𝑒𝑥 + 𝐶
𝑒 𝑒 𝑒 𝑒

D. Inverse Trigonometric Functions

Example 3.4
Evaluate the integral below by using 𝑢-substitution, followed by integration by parts.
1
∫ sin−1 ( ) 𝑑𝑥
𝑥

Integration by parts
Get all the files at: https://bit.ly/azizhandouts

1
𝑢 = sin−1 ( ) , 𝑑𝑣 = 𝑑𝑥
𝑥
1 1 1
𝑑𝑢 = − 𝑑𝑥 = − 𝑑𝑥 = − 𝑑𝑥, 𝑣=𝑥
1 √𝑥 2−1 √𝑥 2−1∙𝑥
√1 − 2 ∙ 𝑥 2
𝑥 ∙ 𝑥2
𝑥
Apply ∫ 𝑢 𝑑𝑣 = 𝑢𝑣 − ∫ 𝑣 𝑑𝑢:
1 1 1
∫ sin−1 ( ) 𝑑𝑥 = 𝑥 ∙ sin−1 − ∫ 𝑥 × − 𝑑𝑥
𝑥 𝑥 √𝑥 2 − 1 ∙ 𝑥

Focus on the last integral. Start by simplifying:


1 1
−∫𝑥 × − 𝑑𝑥 = ∫ 𝑑𝑥
√𝑥 2 − 1 ∙ 𝑥 √𝑥 2 − 1
𝒖-substitution
You can recognize the above as the derivative of cosh−1 𝑥 or understand the method better by substituting:
𝑥 = cosh 𝑢 ⇒ 𝑑𝑥 = sinh 𝑢 𝑑𝑢
To get:
1 1
∫ 𝑑𝑥 = ∫ sinh 𝑢 𝑑𝑢
2
√𝑥 − 1 √cosh2 𝑢 − 1

Use the identity cosh2 𝑢 − sinh2 𝑢 = 1 ⇒ sinh2 𝑢 = 1 − cosh2 𝑢:


1 1
=∫ sinh 𝑢 𝑑𝑢 = ∫ sinh 𝑢 𝑑𝑢 = ∫ 𝑑𝑢 = 𝑢 + 𝑐
2
√sinh 𝑢 sinh 𝑢

And now 𝑥 = cosh 𝑢 ⇒ 𝑢 = cosh−1 𝑥:


𝑢 + 𝑐 = cosh−1 𝑥 + 𝐶

And, hence, the final answer is:


1 1
∫ sin−1 ( ) 𝑑𝑥 = 𝑥 ∙ sin−1 ( ) + cosh−1 𝑥 + 𝐶
𝑥 𝑥
E. Using equations to solve for the Integral

3.2 Leibniz Rule


A. Basics

3.5: Leibniz Rule


Proof

Example 3.6
Explanation

Example 3.7
Practice examples

Example 3.8
JEE 2024 question
Get all the files at: https://bit.ly/azizhandouts

3.3 Walli’s Theorem


Example 3.9
Statement and Basic Examples

10 Examples

You might also like